Allowed Publications
Slot System
Featured Buckets
Featured Buckets Admin
Reverse Chronological Sort
Allow Teaser Image

Is menopausal hormone therapy safe when your patient carries a BRCA mutation?

Article Type
Changed
Tue, 08/28/2018 - 11:06
Display Headline
Is menopausal hormone therapy safe when your patient carries a BRCA mutation?

Case: Disabling vasomotor symptoms in a BRCA1 mutation carrier
Christine is a 39-year-old mother of 2 who underwent risk-reducing, minimally invasive bilateral salpingo-oophorectomy with hysterectomy 4 months ago for a BRCA1 mutation (with benign findings on pathology). Eighteen months before that surgery, she had risk-reducing bilateral mastectomy with reconstruction (implants) and was advised by her surgeon that she no longer needs breast imaging.

Today she reports disabling hot flashes, insomnia, vaginal dryness, and painful sex. Her previous ObGyn, who performed the hysterectomy, was unwilling to prescribe hormone therapy (HT) due to safety concerns. Christine tried venlafaxine at 37 to 75 mg but noted little relief of her vasomotor symptoms.

In discussing her symptoms with you during this initial visit, Christine, a practicing accountant, also reveals that she does not feel as intellectually “sharp” as she did before her gynecologic surgery.

What can you offer for relief of her symptoms?

More BRCA mutation carriers are being identified and choosing to undergo risk-reducing salpingo-oophorectomy and bilateral mastectomy. Accordingly, clinicians are likely to face more questions about the use of systemic HT in this population. Because mutation carriers may worry about the safety of HT, given their BRCA status, some may delay or avoid salpingo-oophorectomy—a surgery that not only reduces the risk of ovarian, fallopian tube, and peritoneal cancer by 80% but also decreases the risk of breast cancer by 48%.1

Surgically menopausal women in their 30s or 40s who are not treated with HT appear to have an elevated risk for dementia and Parkinsonism.2 In addition, vasomotor symptoms are often more severe, and the risks for osteoporosis and, likely, cardiovascular disease are elevated in women with early menopause who are not treated with HT. For these reasons, systemic HT is recommended for women with early menopause, and generally should be continued at least until the normal age of menopause unless specific contraindications are present.3

Because Christine not only has had risk-reducing gynecologic surgery but also risk-reducing bilateral mastectomy, her current risk for breast cancer is very low whether or not she uses HT. Because she does not have a uterus, her symptoms can effectively and safely be treated with systemic estrogen-only therapy.

Among clinicians with special expertise in the management of BRCA mutation carriers, the use of systemic HT would be considered appropriate—and not controversial—in this setting.4

Angelina Jolie details her surgeries
In March 2015, 39-year-old Oscar-winning actress and filmmaker Angelina Jolie Pitt published an opinion piece in the New York Times detailing her recent laparoscopic salpingo-oophorectomy and initiation of HT.5 Ms. Jolie Pitt, who carries the BRCA1 mutation, lost her mother, grandmother, and aunt to hereditary breast/ovarian cancer. Two years earlier, Ms. Jolie Pitt made news by describing her decision to move ahead with risk-reducing bilateral mastectomy.

Following her risk-reducing salpingo-oophorectomy, she initiated systemic HT using transdermal estradiol and off-label use of a levonorgestrel-releasing intrauterine system for endometrial protection.

Her courageous decision to publicly describe her surgery and subsequent initiation of systemic HT will likely encourage women with ominous family histories to seek out genetic counseling and testing. Her decision to “go public” regarding surgery should help mutation carriers without a history of cancer (known in the BRCA community as “previvors”) who have completed their families to move forward with risk-reducing gynecologic surgery and, when appropriate, use of systemic HT.6

The outlook for previvors with intact breasts
Three studies address the risk of breast cancer with use of systemic HT among previvors with intact breasts. A 2005 study followed a cohort of BRCA1 and BRCA2 carriers with intact breasts, 155 of whom had undergone risk-reducing salpingo-oophorectomy, for a mean of 3.6 years. Of these women, 60% and 7%, respectively, of those who had and had not undergone salpingo-oophorectomy used HT. The authors noted that bilateral salpingo-oophorectomy reduced the risk of breast cancer by some 60%, whether or not women used HT.7

A 2008 case-control study focused on 472 menopausal BRCA1 carriers, half of whom had been diagnosed with breast cancer (cases); the other half had not received this diagnosis (controls). A 43% reduction in the risk of breast cancer was associated with prior use of HT.8

A 2011 presentation described a cohort study in which 1,299 BRCA1 and BRCA2 carriers with intact breasts who had undergone salpingo-oophorectomy were followed for a mean of 5.4 years postoperatively. In this population, use of HT was not associated with an increased risk of breast cancer. Among women with BRCA1 mutations, use of systemic HT was associated with a reduced risk of breast cancer.9

Viewed in aggregate, these studies reassure us that short-term use of systemic HT does not increase breast cancer risk in women with BRCA1 or BRCA2 mutations and intact breasts.

 

 

Dr. Simon
Nevertheless, I think it is important to point out that a properly powered study to assess actual risk in this setting is not available in the literature.

When a patient refuses HT
Dr. Pinkerton
Some BRCA mutation carriers may refuse HT despite reassurance that it is safe. Nonhormonal therapies are not as effective at relieving severe menopausal symptoms. Almost all selective serotonin reuptake inhibitors (SSRIs) and serotonin-norepinephrine reuptake inhibitors (SNRIs) can be offered, although only low-dose paroxetine salt is approved for treatment of postmenopausal hot flashes.10,11 Gabapentin also has shown efficacy in relieving hot flashes.

For genitourinary syndrome of menopause (GSM; formerly known as vulvovaginal atrophy), lubricants and moisturizers may provide some benefit, but they don’t improve the vaginal superficial cells and, therefore, are not as effective as hormonal options. There is now a selective estrogen receptor modulator (SERM) approved to treat GSM—ospemifene. However, in clinical trials, ospemifene has been shown to increase hot flashes, so it would not be a good option for our patient.12

Case: Continued
Christine follows up 3 months after initiating estrogen therapy (oral estradiol 2 mg daily). She reports significant improvement in her hot flashes, with improved sleep and fewer sleep disruptions. In addition, she feels that her “mental sharpness” has returned.

Dr. Pinkerton
What if this patient had an intact uterus? Then she would not be a candidate for estrogen-only therapy because she would need continued endometrial protection. Options then would include low-dose continuous or cyclic progestogen therapy, often starting with micronized progesterone, as the E3N Study suggested it has a less negative effect on the uterus.13 Or she could use a levonorgestrel-releasing intrauterine system off label, as Ms. Jolie Pitt elected to do.

Another option would be combining estrogen with a SERM. The only estrogen/SERM combination currently approved by the US Food and Drug Administration (FDA) is conjugated estrogen/bazedoxefine, which showed no increase in breast tenderness, breast density, or bleeding rates, compared with placebo, in multiple trials up to 5 years in duration.14

Case: Resolved
Christine says she would like to continue HT, although she still experiences dryness and discomfort when sexually active with her husband, despite use of a vaginal lubricant. A pelvic examination is consistent with early changes of GSM.15

You discuss GSM with Christine and suggest that she consider 1 of 2 strategies:

 

  • Switch from daily use of oral estradiol to the 3-month systemic 0.1-mg estradiol ring (Femring), which would address both her vasomotor symptoms and her GSM.
  • Continue oral estradiol and add low-dose vaginal estrogen (cream, tablets, or Estring 2 mg).

Christine chooses Option 2. When she returns 6 months later for her well-woman visit, she reports that all of her menopausal symptoms have resolved, and a pelvic examination no longer reveals changes of GSM.

 

Share your thoughts on this article! Send your Letter to the Editor to rbarbieri@frontlinemedcom.com. Please include your name and the city and state in which you practice.

References


1. Finch AP, Lubinski J, Moller P, et al. Impact of oophorectomy on cancer incidence and mortality in women with a BRCA1 or BRCA2 mutation. J Clin Oncol. 2014;32(15):1547–1553.
2. Rocca WA, Bower JH, Maraganore DM, et al. Increased risk of cognitive impairment or dementia in women who underwent oophorectomy before menopause. Neurology. 2007;69(11):1074–1083.
3. North American Menopause Society. The 2012 hormone therapy position statement of the North American Menopause Society. Menopause. 2012;19(3):257–271.
4. Finch AP, Evans G, Narod SA. BRCA carriers, prophylactic salpingo-oophorectomy and menopause: clinical management considerations and recommendations. Womens Health (Lond Engl). 2012;8(5):543–555.
5. Pitt AJ. Angelina Jolie Pitt: Diary of a Surgery. New York Times. http://www.nytimes.com/2015/03/24/opinion/angelina-jolie-pitt-diary-of-a-surgery.html. Published March 24, 2015. Accessed July 9, 2015.
6. Holman L, Brandt A, Daniels M, et al. Risk-reducing salpingo-oophorectomy and prophylactic mastectomy among BRCA mutation “previvors.” Gynecol Oncol. 2012;127(1 suppl):S17.
7. Rebbeck TR, Friebel T, Wagner T, et al; PROSE Study Group. Effect of short-term hormone replacement therapy on breast cancer risk reduction after bilateral prophylactic oophorectomy in BRCA1 and BRCA2 mutation carriers: the PROSE Study Group. J Clin Oncol. 2005;23(31):7804–7810.
8. Eisen A, Lubinski J, Gronwald J, et al; Hereditary Breast Cancer Clinical Study Group. Hormone therapy and the risk of breast cancer in BRCA1 mutation carriers. J Natl Cancer Inst. 2008;100(19):1361–1367.
9. Domchek SM, Mitchell G, Lindeman GJ, et al. Challenges to the development of new agents for molecularly defined patient subsets: lessons from BRCA1/2-associated breast cancer. J Clin Oncol. 2011;29(32):4224–4226.
10. Krause MS, Nakajima ST. Hormonal and nonhormonal treatment of vasomotor symptoms. Obstet Gynecol Clin North Am. 2015;42(1):163–179.
11. Simon JA, Portman DJ, Kaunitz AM, et al. Low-dose paroxetine 7.5 mg for menopausal vasomotor symptoms: two randomized controlled trials. Menopause. 2013;20(10):1027–1030.
12. Bachmann GA, Komi JO; Ospemifene Study Group. Ospemifene effectively treats vulvovaginal atrophy in postmenopausal women: results from a pivotal phase 3 study. Menopause. 2010;17(3):480–486.
13. Fournier A, Fabre A, Mesrine S, Boutron-Ruault MC, Berrino F, Clavel-Chapelon F. Use of different postmenopausal hormone therapies and risk of histology- and hormone receptor–defined invasive breast cancer. J Clin Oncol. 2008;26(8):1260–1268.
14. Pinkerton JV, Pickar JH, Racketa J, Mirkin S. Bazedoxifene/conjugated estrogens for menopausal symptom treatment and osteoporosis prevention. Climacteric. 2012;15(5):411–418.
15. Rahn DD, Carberry C, Sanses TV, et al; Society of Gynecologic Surgeons Systematic Review Group. Vaginal estrogen for genitourinary syndrome of menopause: a systematic review. Obstet Gynecol. 2014;124(6):1147–1156.

Article PDF
Author and Disclosure Information
CASES IN MENOPAUSEBrought to you by the menopause experts


Dr. Kaunitz is University of Florida Research Foundation Professor and Associate Chairman, Department of Obstetrics and Gynecology, University of Florida College of Medicine–Jacksonville, and Director of Menopause and Gynecologic Ultrasound Services, UF Women’s Health Specialists–Emerson. Dr. Kaunitz serves on the OBG Management Board of Editors.

Dr. Pinkerton is Professor, Department of Obstetrics and Gynecology, and Director, Division of Midlife Women’s Health, at the University of Virginia in Charlottesville. Dr. Pinkerton is a North American Menopause Society (NAMS) past president and certified menopause practitioner. She also serves on the OBG Management Board of Editors.

Dr. Simon is Clinical Professor, Department of Obstetrics and Gynecology, George Washington University, and Medical Director, Women’s Health & Research Consultants, Washington, DC. Dr. Simon is a NAMS past president, a certified menopause practitioner, and clinical densitometrist. He also serves on the OBG Management Board of Editors.

Dr. Pinkerton and Dr. Simon provided peer review and comments for Dr. Kaunitz’s case study.

 

Disclosures

Dr. Kaunitz reports that within the past 36 months, he has received or is currently receiving grant or research support from Bayer, Teva, and TherapeuticsMD, and has served or is currently serving as a consultant to Actavis, Bayer, and Teva.

Dr. Pinkerton reports that within the past 36 months, she has received or is currently receiving grant or research support from TherapeuticsMD and has served or is currently serving as a consultant to Noven, Inc., Pfizer, Shionogi, and TherapeuticsMD.

Dr. Simon reports that within the past 36 months, he has been a consultant to or served on the advisory boards of AbbVie, Actavis, Amgen, Amneal, Apotex, Ascend ­Therapeutics, BioSante, Depomed, Dr. ­Reddy Laboratories, Everett Laboratories, Intimina by Lelo, Lupin, Meda, Merck, Novartis, ­Noven, Novo Nordisk, Nuelle, Pfizer, Regeneron, Sanofi SA, Sermonix, Shionogi, Shippan Point Advisors, Sprout, Teva, TherapeuticsMD, Warner Chilcott, and Watson.

In the past 36 months, Dr. Simon has received grant/research support from ­AbbVie, ­Actavis, Agile Therapeutics, Bayer Healthcare, Endo­Ceutics, New England Research Institute, Novo Nordisk, Palatin Technologies, Teva, and ­TherapeuticsMD.

Within the past 36 months, Dr. Simon has also served on the speaker’s bureaus of Amgen, Eisai, Merck, Novartis, ­Noven, Novo Nordisk, Shionogi, Teva, and Warner Chilcott.

Dr. Simon served as Chief Medical Officer of Sprout Pharmaceuticals until April 1, 2013.

Issue
OBG Management - 27(8)
Publications
Topics
Page Number
24–26
Legacy Keywords
Andrew M. Kaunitz MD, JoAnn V. Pinkerton MD, James A. Simon MD, menopause, hormone therapy, HT, patient carries a BRCA mutation, BRCA mutation, breast cancer, BRCA1, BRCA2, mutation carrier, intact breasts, risk-reducing bilateral mastectomy, systemic hormone therapy, risk of breast cancer, vasomotor symptoms, hot flashes, insomnia, vaginal dryness, painful sex, hysterectomy, salpingo-oophorectomy, ovarian cancer, fallopian tube cancer, peritoneal cancer, surgical menopause, dementia, Parkinsonism,
Angelina Jolie, previvors, systemic HT, nonhormonal therapy, selective serotonin reuptake inhibitors, SSRIs, serotonin-norepinephrine reuptake inhibitors, SNRIs, paroxetine salt, gabapentin, genitourinary syndrome of menopause, GSM, vaginal lubricants, vaginal moisturizers, selective estrogen receptor modulator, SERM, ospemifene, estrogen therapy, oral estradiol, endometrial protection, progestogen therapy, micronized progesterone,
Sections
Author and Disclosure Information
CASES IN MENOPAUSEBrought to you by the menopause experts


Dr. Kaunitz is University of Florida Research Foundation Professor and Associate Chairman, Department of Obstetrics and Gynecology, University of Florida College of Medicine–Jacksonville, and Director of Menopause and Gynecologic Ultrasound Services, UF Women’s Health Specialists–Emerson. Dr. Kaunitz serves on the OBG Management Board of Editors.

Dr. Pinkerton is Professor, Department of Obstetrics and Gynecology, and Director, Division of Midlife Women’s Health, at the University of Virginia in Charlottesville. Dr. Pinkerton is a North American Menopause Society (NAMS) past president and certified menopause practitioner. She also serves on the OBG Management Board of Editors.

Dr. Simon is Clinical Professor, Department of Obstetrics and Gynecology, George Washington University, and Medical Director, Women’s Health & Research Consultants, Washington, DC. Dr. Simon is a NAMS past president, a certified menopause practitioner, and clinical densitometrist. He also serves on the OBG Management Board of Editors.

Dr. Pinkerton and Dr. Simon provided peer review and comments for Dr. Kaunitz’s case study.

 

Disclosures

Dr. Kaunitz reports that within the past 36 months, he has received or is currently receiving grant or research support from Bayer, Teva, and TherapeuticsMD, and has served or is currently serving as a consultant to Actavis, Bayer, and Teva.

Dr. Pinkerton reports that within the past 36 months, she has received or is currently receiving grant or research support from TherapeuticsMD and has served or is currently serving as a consultant to Noven, Inc., Pfizer, Shionogi, and TherapeuticsMD.

Dr. Simon reports that within the past 36 months, he has been a consultant to or served on the advisory boards of AbbVie, Actavis, Amgen, Amneal, Apotex, Ascend ­Therapeutics, BioSante, Depomed, Dr. ­Reddy Laboratories, Everett Laboratories, Intimina by Lelo, Lupin, Meda, Merck, Novartis, ­Noven, Novo Nordisk, Nuelle, Pfizer, Regeneron, Sanofi SA, Sermonix, Shionogi, Shippan Point Advisors, Sprout, Teva, TherapeuticsMD, Warner Chilcott, and Watson.

In the past 36 months, Dr. Simon has received grant/research support from ­AbbVie, ­Actavis, Agile Therapeutics, Bayer Healthcare, Endo­Ceutics, New England Research Institute, Novo Nordisk, Palatin Technologies, Teva, and ­TherapeuticsMD.

Within the past 36 months, Dr. Simon has also served on the speaker’s bureaus of Amgen, Eisai, Merck, Novartis, ­Noven, Novo Nordisk, Shionogi, Teva, and Warner Chilcott.

Dr. Simon served as Chief Medical Officer of Sprout Pharmaceuticals until April 1, 2013.

Author and Disclosure Information
CASES IN MENOPAUSEBrought to you by the menopause experts


Dr. Kaunitz is University of Florida Research Foundation Professor and Associate Chairman, Department of Obstetrics and Gynecology, University of Florida College of Medicine–Jacksonville, and Director of Menopause and Gynecologic Ultrasound Services, UF Women’s Health Specialists–Emerson. Dr. Kaunitz serves on the OBG Management Board of Editors.

Dr. Pinkerton is Professor, Department of Obstetrics and Gynecology, and Director, Division of Midlife Women’s Health, at the University of Virginia in Charlottesville. Dr. Pinkerton is a North American Menopause Society (NAMS) past president and certified menopause practitioner. She also serves on the OBG Management Board of Editors.

Dr. Simon is Clinical Professor, Department of Obstetrics and Gynecology, George Washington University, and Medical Director, Women’s Health & Research Consultants, Washington, DC. Dr. Simon is a NAMS past president, a certified menopause practitioner, and clinical densitometrist. He also serves on the OBG Management Board of Editors.

Dr. Pinkerton and Dr. Simon provided peer review and comments for Dr. Kaunitz’s case study.

 

Disclosures

Dr. Kaunitz reports that within the past 36 months, he has received or is currently receiving grant or research support from Bayer, Teva, and TherapeuticsMD, and has served or is currently serving as a consultant to Actavis, Bayer, and Teva.

Dr. Pinkerton reports that within the past 36 months, she has received or is currently receiving grant or research support from TherapeuticsMD and has served or is currently serving as a consultant to Noven, Inc., Pfizer, Shionogi, and TherapeuticsMD.

Dr. Simon reports that within the past 36 months, he has been a consultant to or served on the advisory boards of AbbVie, Actavis, Amgen, Amneal, Apotex, Ascend ­Therapeutics, BioSante, Depomed, Dr. ­Reddy Laboratories, Everett Laboratories, Intimina by Lelo, Lupin, Meda, Merck, Novartis, ­Noven, Novo Nordisk, Nuelle, Pfizer, Regeneron, Sanofi SA, Sermonix, Shionogi, Shippan Point Advisors, Sprout, Teva, TherapeuticsMD, Warner Chilcott, and Watson.

In the past 36 months, Dr. Simon has received grant/research support from ­AbbVie, ­Actavis, Agile Therapeutics, Bayer Healthcare, Endo­Ceutics, New England Research Institute, Novo Nordisk, Palatin Technologies, Teva, and ­TherapeuticsMD.

Within the past 36 months, Dr. Simon has also served on the speaker’s bureaus of Amgen, Eisai, Merck, Novartis, ­Noven, Novo Nordisk, Shionogi, Teva, and Warner Chilcott.

Dr. Simon served as Chief Medical Officer of Sprout Pharmaceuticals until April 1, 2013.

Article PDF
Article PDF
Related Articles

Case: Disabling vasomotor symptoms in a BRCA1 mutation carrier
Christine is a 39-year-old mother of 2 who underwent risk-reducing, minimally invasive bilateral salpingo-oophorectomy with hysterectomy 4 months ago for a BRCA1 mutation (with benign findings on pathology). Eighteen months before that surgery, she had risk-reducing bilateral mastectomy with reconstruction (implants) and was advised by her surgeon that she no longer needs breast imaging.

Today she reports disabling hot flashes, insomnia, vaginal dryness, and painful sex. Her previous ObGyn, who performed the hysterectomy, was unwilling to prescribe hormone therapy (HT) due to safety concerns. Christine tried venlafaxine at 37 to 75 mg but noted little relief of her vasomotor symptoms.

In discussing her symptoms with you during this initial visit, Christine, a practicing accountant, also reveals that she does not feel as intellectually “sharp” as she did before her gynecologic surgery.

What can you offer for relief of her symptoms?

More BRCA mutation carriers are being identified and choosing to undergo risk-reducing salpingo-oophorectomy and bilateral mastectomy. Accordingly, clinicians are likely to face more questions about the use of systemic HT in this population. Because mutation carriers may worry about the safety of HT, given their BRCA status, some may delay or avoid salpingo-oophorectomy—a surgery that not only reduces the risk of ovarian, fallopian tube, and peritoneal cancer by 80% but also decreases the risk of breast cancer by 48%.1

Surgically menopausal women in their 30s or 40s who are not treated with HT appear to have an elevated risk for dementia and Parkinsonism.2 In addition, vasomotor symptoms are often more severe, and the risks for osteoporosis and, likely, cardiovascular disease are elevated in women with early menopause who are not treated with HT. For these reasons, systemic HT is recommended for women with early menopause, and generally should be continued at least until the normal age of menopause unless specific contraindications are present.3

Because Christine not only has had risk-reducing gynecologic surgery but also risk-reducing bilateral mastectomy, her current risk for breast cancer is very low whether or not she uses HT. Because she does not have a uterus, her symptoms can effectively and safely be treated with systemic estrogen-only therapy.

Among clinicians with special expertise in the management of BRCA mutation carriers, the use of systemic HT would be considered appropriate—and not controversial—in this setting.4

Angelina Jolie details her surgeries
In March 2015, 39-year-old Oscar-winning actress and filmmaker Angelina Jolie Pitt published an opinion piece in the New York Times detailing her recent laparoscopic salpingo-oophorectomy and initiation of HT.5 Ms. Jolie Pitt, who carries the BRCA1 mutation, lost her mother, grandmother, and aunt to hereditary breast/ovarian cancer. Two years earlier, Ms. Jolie Pitt made news by describing her decision to move ahead with risk-reducing bilateral mastectomy.

Following her risk-reducing salpingo-oophorectomy, she initiated systemic HT using transdermal estradiol and off-label use of a levonorgestrel-releasing intrauterine system for endometrial protection.

Her courageous decision to publicly describe her surgery and subsequent initiation of systemic HT will likely encourage women with ominous family histories to seek out genetic counseling and testing. Her decision to “go public” regarding surgery should help mutation carriers without a history of cancer (known in the BRCA community as “previvors”) who have completed their families to move forward with risk-reducing gynecologic surgery and, when appropriate, use of systemic HT.6

The outlook for previvors with intact breasts
Three studies address the risk of breast cancer with use of systemic HT among previvors with intact breasts. A 2005 study followed a cohort of BRCA1 and BRCA2 carriers with intact breasts, 155 of whom had undergone risk-reducing salpingo-oophorectomy, for a mean of 3.6 years. Of these women, 60% and 7%, respectively, of those who had and had not undergone salpingo-oophorectomy used HT. The authors noted that bilateral salpingo-oophorectomy reduced the risk of breast cancer by some 60%, whether or not women used HT.7

A 2008 case-control study focused on 472 menopausal BRCA1 carriers, half of whom had been diagnosed with breast cancer (cases); the other half had not received this diagnosis (controls). A 43% reduction in the risk of breast cancer was associated with prior use of HT.8

A 2011 presentation described a cohort study in which 1,299 BRCA1 and BRCA2 carriers with intact breasts who had undergone salpingo-oophorectomy were followed for a mean of 5.4 years postoperatively. In this population, use of HT was not associated with an increased risk of breast cancer. Among women with BRCA1 mutations, use of systemic HT was associated with a reduced risk of breast cancer.9

Viewed in aggregate, these studies reassure us that short-term use of systemic HT does not increase breast cancer risk in women with BRCA1 or BRCA2 mutations and intact breasts.

 

 

Dr. Simon
Nevertheless, I think it is important to point out that a properly powered study to assess actual risk in this setting is not available in the literature.

When a patient refuses HT
Dr. Pinkerton
Some BRCA mutation carriers may refuse HT despite reassurance that it is safe. Nonhormonal therapies are not as effective at relieving severe menopausal symptoms. Almost all selective serotonin reuptake inhibitors (SSRIs) and serotonin-norepinephrine reuptake inhibitors (SNRIs) can be offered, although only low-dose paroxetine salt is approved for treatment of postmenopausal hot flashes.10,11 Gabapentin also has shown efficacy in relieving hot flashes.

For genitourinary syndrome of menopause (GSM; formerly known as vulvovaginal atrophy), lubricants and moisturizers may provide some benefit, but they don’t improve the vaginal superficial cells and, therefore, are not as effective as hormonal options. There is now a selective estrogen receptor modulator (SERM) approved to treat GSM—ospemifene. However, in clinical trials, ospemifene has been shown to increase hot flashes, so it would not be a good option for our patient.12

Case: Continued
Christine follows up 3 months after initiating estrogen therapy (oral estradiol 2 mg daily). She reports significant improvement in her hot flashes, with improved sleep and fewer sleep disruptions. In addition, she feels that her “mental sharpness” has returned.

Dr. Pinkerton
What if this patient had an intact uterus? Then she would not be a candidate for estrogen-only therapy because she would need continued endometrial protection. Options then would include low-dose continuous or cyclic progestogen therapy, often starting with micronized progesterone, as the E3N Study suggested it has a less negative effect on the uterus.13 Or she could use a levonorgestrel-releasing intrauterine system off label, as Ms. Jolie Pitt elected to do.

Another option would be combining estrogen with a SERM. The only estrogen/SERM combination currently approved by the US Food and Drug Administration (FDA) is conjugated estrogen/bazedoxefine, which showed no increase in breast tenderness, breast density, or bleeding rates, compared with placebo, in multiple trials up to 5 years in duration.14

Case: Resolved
Christine says she would like to continue HT, although she still experiences dryness and discomfort when sexually active with her husband, despite use of a vaginal lubricant. A pelvic examination is consistent with early changes of GSM.15

You discuss GSM with Christine and suggest that she consider 1 of 2 strategies:

 

  • Switch from daily use of oral estradiol to the 3-month systemic 0.1-mg estradiol ring (Femring), which would address both her vasomotor symptoms and her GSM.
  • Continue oral estradiol and add low-dose vaginal estrogen (cream, tablets, or Estring 2 mg).

Christine chooses Option 2. When she returns 6 months later for her well-woman visit, she reports that all of her menopausal symptoms have resolved, and a pelvic examination no longer reveals changes of GSM.

 

Share your thoughts on this article! Send your Letter to the Editor to rbarbieri@frontlinemedcom.com. Please include your name and the city and state in which you practice.

Case: Disabling vasomotor symptoms in a BRCA1 mutation carrier
Christine is a 39-year-old mother of 2 who underwent risk-reducing, minimally invasive bilateral salpingo-oophorectomy with hysterectomy 4 months ago for a BRCA1 mutation (with benign findings on pathology). Eighteen months before that surgery, she had risk-reducing bilateral mastectomy with reconstruction (implants) and was advised by her surgeon that she no longer needs breast imaging.

Today she reports disabling hot flashes, insomnia, vaginal dryness, and painful sex. Her previous ObGyn, who performed the hysterectomy, was unwilling to prescribe hormone therapy (HT) due to safety concerns. Christine tried venlafaxine at 37 to 75 mg but noted little relief of her vasomotor symptoms.

In discussing her symptoms with you during this initial visit, Christine, a practicing accountant, also reveals that she does not feel as intellectually “sharp” as she did before her gynecologic surgery.

What can you offer for relief of her symptoms?

More BRCA mutation carriers are being identified and choosing to undergo risk-reducing salpingo-oophorectomy and bilateral mastectomy. Accordingly, clinicians are likely to face more questions about the use of systemic HT in this population. Because mutation carriers may worry about the safety of HT, given their BRCA status, some may delay or avoid salpingo-oophorectomy—a surgery that not only reduces the risk of ovarian, fallopian tube, and peritoneal cancer by 80% but also decreases the risk of breast cancer by 48%.1

Surgically menopausal women in their 30s or 40s who are not treated with HT appear to have an elevated risk for dementia and Parkinsonism.2 In addition, vasomotor symptoms are often more severe, and the risks for osteoporosis and, likely, cardiovascular disease are elevated in women with early menopause who are not treated with HT. For these reasons, systemic HT is recommended for women with early menopause, and generally should be continued at least until the normal age of menopause unless specific contraindications are present.3

Because Christine not only has had risk-reducing gynecologic surgery but also risk-reducing bilateral mastectomy, her current risk for breast cancer is very low whether or not she uses HT. Because she does not have a uterus, her symptoms can effectively and safely be treated with systemic estrogen-only therapy.

Among clinicians with special expertise in the management of BRCA mutation carriers, the use of systemic HT would be considered appropriate—and not controversial—in this setting.4

Angelina Jolie details her surgeries
In March 2015, 39-year-old Oscar-winning actress and filmmaker Angelina Jolie Pitt published an opinion piece in the New York Times detailing her recent laparoscopic salpingo-oophorectomy and initiation of HT.5 Ms. Jolie Pitt, who carries the BRCA1 mutation, lost her mother, grandmother, and aunt to hereditary breast/ovarian cancer. Two years earlier, Ms. Jolie Pitt made news by describing her decision to move ahead with risk-reducing bilateral mastectomy.

Following her risk-reducing salpingo-oophorectomy, she initiated systemic HT using transdermal estradiol and off-label use of a levonorgestrel-releasing intrauterine system for endometrial protection.

Her courageous decision to publicly describe her surgery and subsequent initiation of systemic HT will likely encourage women with ominous family histories to seek out genetic counseling and testing. Her decision to “go public” regarding surgery should help mutation carriers without a history of cancer (known in the BRCA community as “previvors”) who have completed their families to move forward with risk-reducing gynecologic surgery and, when appropriate, use of systemic HT.6

The outlook for previvors with intact breasts
Three studies address the risk of breast cancer with use of systemic HT among previvors with intact breasts. A 2005 study followed a cohort of BRCA1 and BRCA2 carriers with intact breasts, 155 of whom had undergone risk-reducing salpingo-oophorectomy, for a mean of 3.6 years. Of these women, 60% and 7%, respectively, of those who had and had not undergone salpingo-oophorectomy used HT. The authors noted that bilateral salpingo-oophorectomy reduced the risk of breast cancer by some 60%, whether or not women used HT.7

A 2008 case-control study focused on 472 menopausal BRCA1 carriers, half of whom had been diagnosed with breast cancer (cases); the other half had not received this diagnosis (controls). A 43% reduction in the risk of breast cancer was associated with prior use of HT.8

A 2011 presentation described a cohort study in which 1,299 BRCA1 and BRCA2 carriers with intact breasts who had undergone salpingo-oophorectomy were followed for a mean of 5.4 years postoperatively. In this population, use of HT was not associated with an increased risk of breast cancer. Among women with BRCA1 mutations, use of systemic HT was associated with a reduced risk of breast cancer.9

Viewed in aggregate, these studies reassure us that short-term use of systemic HT does not increase breast cancer risk in women with BRCA1 or BRCA2 mutations and intact breasts.

 

 

Dr. Simon
Nevertheless, I think it is important to point out that a properly powered study to assess actual risk in this setting is not available in the literature.

When a patient refuses HT
Dr. Pinkerton
Some BRCA mutation carriers may refuse HT despite reassurance that it is safe. Nonhormonal therapies are not as effective at relieving severe menopausal symptoms. Almost all selective serotonin reuptake inhibitors (SSRIs) and serotonin-norepinephrine reuptake inhibitors (SNRIs) can be offered, although only low-dose paroxetine salt is approved for treatment of postmenopausal hot flashes.10,11 Gabapentin also has shown efficacy in relieving hot flashes.

For genitourinary syndrome of menopause (GSM; formerly known as vulvovaginal atrophy), lubricants and moisturizers may provide some benefit, but they don’t improve the vaginal superficial cells and, therefore, are not as effective as hormonal options. There is now a selective estrogen receptor modulator (SERM) approved to treat GSM—ospemifene. However, in clinical trials, ospemifene has been shown to increase hot flashes, so it would not be a good option for our patient.12

Case: Continued
Christine follows up 3 months after initiating estrogen therapy (oral estradiol 2 mg daily). She reports significant improvement in her hot flashes, with improved sleep and fewer sleep disruptions. In addition, she feels that her “mental sharpness” has returned.

Dr. Pinkerton
What if this patient had an intact uterus? Then she would not be a candidate for estrogen-only therapy because she would need continued endometrial protection. Options then would include low-dose continuous or cyclic progestogen therapy, often starting with micronized progesterone, as the E3N Study suggested it has a less negative effect on the uterus.13 Or she could use a levonorgestrel-releasing intrauterine system off label, as Ms. Jolie Pitt elected to do.

Another option would be combining estrogen with a SERM. The only estrogen/SERM combination currently approved by the US Food and Drug Administration (FDA) is conjugated estrogen/bazedoxefine, which showed no increase in breast tenderness, breast density, or bleeding rates, compared with placebo, in multiple trials up to 5 years in duration.14

Case: Resolved
Christine says she would like to continue HT, although she still experiences dryness and discomfort when sexually active with her husband, despite use of a vaginal lubricant. A pelvic examination is consistent with early changes of GSM.15

You discuss GSM with Christine and suggest that she consider 1 of 2 strategies:

 

  • Switch from daily use of oral estradiol to the 3-month systemic 0.1-mg estradiol ring (Femring), which would address both her vasomotor symptoms and her GSM.
  • Continue oral estradiol and add low-dose vaginal estrogen (cream, tablets, or Estring 2 mg).

Christine chooses Option 2. When she returns 6 months later for her well-woman visit, she reports that all of her menopausal symptoms have resolved, and a pelvic examination no longer reveals changes of GSM.

 

Share your thoughts on this article! Send your Letter to the Editor to rbarbieri@frontlinemedcom.com. Please include your name and the city and state in which you practice.

References


1. Finch AP, Lubinski J, Moller P, et al. Impact of oophorectomy on cancer incidence and mortality in women with a BRCA1 or BRCA2 mutation. J Clin Oncol. 2014;32(15):1547–1553.
2. Rocca WA, Bower JH, Maraganore DM, et al. Increased risk of cognitive impairment or dementia in women who underwent oophorectomy before menopause. Neurology. 2007;69(11):1074–1083.
3. North American Menopause Society. The 2012 hormone therapy position statement of the North American Menopause Society. Menopause. 2012;19(3):257–271.
4. Finch AP, Evans G, Narod SA. BRCA carriers, prophylactic salpingo-oophorectomy and menopause: clinical management considerations and recommendations. Womens Health (Lond Engl). 2012;8(5):543–555.
5. Pitt AJ. Angelina Jolie Pitt: Diary of a Surgery. New York Times. http://www.nytimes.com/2015/03/24/opinion/angelina-jolie-pitt-diary-of-a-surgery.html. Published March 24, 2015. Accessed July 9, 2015.
6. Holman L, Brandt A, Daniels M, et al. Risk-reducing salpingo-oophorectomy and prophylactic mastectomy among BRCA mutation “previvors.” Gynecol Oncol. 2012;127(1 suppl):S17.
7. Rebbeck TR, Friebel T, Wagner T, et al; PROSE Study Group. Effect of short-term hormone replacement therapy on breast cancer risk reduction after bilateral prophylactic oophorectomy in BRCA1 and BRCA2 mutation carriers: the PROSE Study Group. J Clin Oncol. 2005;23(31):7804–7810.
8. Eisen A, Lubinski J, Gronwald J, et al; Hereditary Breast Cancer Clinical Study Group. Hormone therapy and the risk of breast cancer in BRCA1 mutation carriers. J Natl Cancer Inst. 2008;100(19):1361–1367.
9. Domchek SM, Mitchell G, Lindeman GJ, et al. Challenges to the development of new agents for molecularly defined patient subsets: lessons from BRCA1/2-associated breast cancer. J Clin Oncol. 2011;29(32):4224–4226.
10. Krause MS, Nakajima ST. Hormonal and nonhormonal treatment of vasomotor symptoms. Obstet Gynecol Clin North Am. 2015;42(1):163–179.
11. Simon JA, Portman DJ, Kaunitz AM, et al. Low-dose paroxetine 7.5 mg for menopausal vasomotor symptoms: two randomized controlled trials. Menopause. 2013;20(10):1027–1030.
12. Bachmann GA, Komi JO; Ospemifene Study Group. Ospemifene effectively treats vulvovaginal atrophy in postmenopausal women: results from a pivotal phase 3 study. Menopause. 2010;17(3):480–486.
13. Fournier A, Fabre A, Mesrine S, Boutron-Ruault MC, Berrino F, Clavel-Chapelon F. Use of different postmenopausal hormone therapies and risk of histology- and hormone receptor–defined invasive breast cancer. J Clin Oncol. 2008;26(8):1260–1268.
14. Pinkerton JV, Pickar JH, Racketa J, Mirkin S. Bazedoxifene/conjugated estrogens for menopausal symptom treatment and osteoporosis prevention. Climacteric. 2012;15(5):411–418.
15. Rahn DD, Carberry C, Sanses TV, et al; Society of Gynecologic Surgeons Systematic Review Group. Vaginal estrogen for genitourinary syndrome of menopause: a systematic review. Obstet Gynecol. 2014;124(6):1147–1156.

References


1. Finch AP, Lubinski J, Moller P, et al. Impact of oophorectomy on cancer incidence and mortality in women with a BRCA1 or BRCA2 mutation. J Clin Oncol. 2014;32(15):1547–1553.
2. Rocca WA, Bower JH, Maraganore DM, et al. Increased risk of cognitive impairment or dementia in women who underwent oophorectomy before menopause. Neurology. 2007;69(11):1074–1083.
3. North American Menopause Society. The 2012 hormone therapy position statement of the North American Menopause Society. Menopause. 2012;19(3):257–271.
4. Finch AP, Evans G, Narod SA. BRCA carriers, prophylactic salpingo-oophorectomy and menopause: clinical management considerations and recommendations. Womens Health (Lond Engl). 2012;8(5):543–555.
5. Pitt AJ. Angelina Jolie Pitt: Diary of a Surgery. New York Times. http://www.nytimes.com/2015/03/24/opinion/angelina-jolie-pitt-diary-of-a-surgery.html. Published March 24, 2015. Accessed July 9, 2015.
6. Holman L, Brandt A, Daniels M, et al. Risk-reducing salpingo-oophorectomy and prophylactic mastectomy among BRCA mutation “previvors.” Gynecol Oncol. 2012;127(1 suppl):S17.
7. Rebbeck TR, Friebel T, Wagner T, et al; PROSE Study Group. Effect of short-term hormone replacement therapy on breast cancer risk reduction after bilateral prophylactic oophorectomy in BRCA1 and BRCA2 mutation carriers: the PROSE Study Group. J Clin Oncol. 2005;23(31):7804–7810.
8. Eisen A, Lubinski J, Gronwald J, et al; Hereditary Breast Cancer Clinical Study Group. Hormone therapy and the risk of breast cancer in BRCA1 mutation carriers. J Natl Cancer Inst. 2008;100(19):1361–1367.
9. Domchek SM, Mitchell G, Lindeman GJ, et al. Challenges to the development of new agents for molecularly defined patient subsets: lessons from BRCA1/2-associated breast cancer. J Clin Oncol. 2011;29(32):4224–4226.
10. Krause MS, Nakajima ST. Hormonal and nonhormonal treatment of vasomotor symptoms. Obstet Gynecol Clin North Am. 2015;42(1):163–179.
11. Simon JA, Portman DJ, Kaunitz AM, et al. Low-dose paroxetine 7.5 mg for menopausal vasomotor symptoms: two randomized controlled trials. Menopause. 2013;20(10):1027–1030.
12. Bachmann GA, Komi JO; Ospemifene Study Group. Ospemifene effectively treats vulvovaginal atrophy in postmenopausal women: results from a pivotal phase 3 study. Menopause. 2010;17(3):480–486.
13. Fournier A, Fabre A, Mesrine S, Boutron-Ruault MC, Berrino F, Clavel-Chapelon F. Use of different postmenopausal hormone therapies and risk of histology- and hormone receptor–defined invasive breast cancer. J Clin Oncol. 2008;26(8):1260–1268.
14. Pinkerton JV, Pickar JH, Racketa J, Mirkin S. Bazedoxifene/conjugated estrogens for menopausal symptom treatment and osteoporosis prevention. Climacteric. 2012;15(5):411–418.
15. Rahn DD, Carberry C, Sanses TV, et al; Society of Gynecologic Surgeons Systematic Review Group. Vaginal estrogen for genitourinary syndrome of menopause: a systematic review. Obstet Gynecol. 2014;124(6):1147–1156.

Issue
OBG Management - 27(8)
Issue
OBG Management - 27(8)
Page Number
24–26
Page Number
24–26
Publications
Publications
Topics
Article Type
Display Headline
Is menopausal hormone therapy safe when your patient carries a BRCA mutation?
Display Headline
Is menopausal hormone therapy safe when your patient carries a BRCA mutation?
Legacy Keywords
Andrew M. Kaunitz MD, JoAnn V. Pinkerton MD, James A. Simon MD, menopause, hormone therapy, HT, patient carries a BRCA mutation, BRCA mutation, breast cancer, BRCA1, BRCA2, mutation carrier, intact breasts, risk-reducing bilateral mastectomy, systemic hormone therapy, risk of breast cancer, vasomotor symptoms, hot flashes, insomnia, vaginal dryness, painful sex, hysterectomy, salpingo-oophorectomy, ovarian cancer, fallopian tube cancer, peritoneal cancer, surgical menopause, dementia, Parkinsonism,
Angelina Jolie, previvors, systemic HT, nonhormonal therapy, selective serotonin reuptake inhibitors, SSRIs, serotonin-norepinephrine reuptake inhibitors, SNRIs, paroxetine salt, gabapentin, genitourinary syndrome of menopause, GSM, vaginal lubricants, vaginal moisturizers, selective estrogen receptor modulator, SERM, ospemifene, estrogen therapy, oral estradiol, endometrial protection, progestogen therapy, micronized progesterone,
Legacy Keywords
Andrew M. Kaunitz MD, JoAnn V. Pinkerton MD, James A. Simon MD, menopause, hormone therapy, HT, patient carries a BRCA mutation, BRCA mutation, breast cancer, BRCA1, BRCA2, mutation carrier, intact breasts, risk-reducing bilateral mastectomy, systemic hormone therapy, risk of breast cancer, vasomotor symptoms, hot flashes, insomnia, vaginal dryness, painful sex, hysterectomy, salpingo-oophorectomy, ovarian cancer, fallopian tube cancer, peritoneal cancer, surgical menopause, dementia, Parkinsonism,
Angelina Jolie, previvors, systemic HT, nonhormonal therapy, selective serotonin reuptake inhibitors, SSRIs, serotonin-norepinephrine reuptake inhibitors, SNRIs, paroxetine salt, gabapentin, genitourinary syndrome of menopause, GSM, vaginal lubricants, vaginal moisturizers, selective estrogen receptor modulator, SERM, ospemifene, estrogen therapy, oral estradiol, endometrial protection, progestogen therapy, micronized progesterone,
Sections
Inside the Article

    In This Article

 

  • Angelina Jolie describes her surgeries
  • Hormone therapy for previvors with intact breasts?
  • When a patient refuses hormone therapy
Article PDF Media

Conjugated estrogen plus bazedoxifene—a new approach to estrogen therapy

Article Type
Changed
Tue, 08/28/2018 - 11:04
Display Headline
Conjugated estrogen plus bazedoxifene—a new approach to estrogen therapy

In this special installment of Cases in Menopause, I interview series contributor and menopause expert JoAnn V. Pinkerton, MD. We discuss a fairly new therapy: the combination conjugated estrogen and bazedoxifene (CE/BZA; Duavee) for the treatment of moderate to severe hot flashes due to menopause and the prevention of menopausal osteoporosis.

Much of my practice has focused on the treatment of menopausal women, but which of my patients can benefit from this particular combination of CE 0.45 mg plus BZA 20 mg? I asked Dr. Pinkerton this question, and more.

Which patients can benefit most?
Dr. Pinkerton CE/BZA was tested in healthy postmenopausal women with a uterus at risk for bone loss who were reporting 50 or more moderate to severe hot flashes per week. The combination of CE and BZA is a good choice for women who have bothersome menopausal symptoms: hot flashes, night sweats, and sleep disruption or symptomatic vulvovaginal atrophy (VVA)—although it’s not approved for VVA.

Efficacy and safety data show that compared with placebo:

 

  • CE/BZA decreases the frequency and severity of hot flashes at 12 weeks, and those decreases are maintained at 12 months.1,2
  • Women taking CE/BZA have greater improvements in sleep, with both decreased sleep disturbance and time to fall asleep.3
  • CE/BZA maintained or prevented lumbar spine and hip bone loss in postmenopausal women at risk for osteoporosis. 1,4,5

Although fracture data were not captured and the drug was not tested in osteoporotic women, study results showed bone loss prevention at 12 months, which was sustained at 24 months. The improvement in bone mineral density from baseline was about 1% to 1.5%. This was compared with a bone loss of 1.8% in women taking placebo (P<.01).

In clinical studies, women taking CE/BZA versus placebo also have reported a lower incidence of painful intercourse,6 and some improvement in health-related quality of life and treatment satisfaction.7,8

In short, CE/BZA is a good option for symptomatic menopausal women with a uterus who have bothersome hot flashes, night sweats, and sleep disruptions and want to prevent bone loss.

What about adverse effects?
Dr. Pinkerton In general, CE/BZA has a favorable safety and tolerability profile, with an overall incidence of adverse events similar to placebo. The rates of cardiovascular and cerebrovascular events, cancers (breast, endometrial, and ovarian), and mortality are comparable to placebo in 2-year trials. These data are limited; studies have been conducted in healthy postmenopausal women. Future studies need to define the full risk profile, particularly among overweight or obese women and different ethnic groups and for longer-term use.

Is there a role among women with breast cancer?
Dr. Pinkerton CE/BZA has not been tested in women at risk for or with prior breast cancer. In preclinical trials of up to 2 years, involving healthy postmenopausal women, the rates for breast cancer with CE/BZA were similar to placebo. There are no long-term data, however, and there are no data in women at risk for breast cancer. I recommend that women who have or are at high risk for breast cancer consider nonhormonal treatment options.9–11

Has there been an associated increase in breast density with CE/BZA?
Dr. Pinkerton No. Data from two randomized clinical trials showed that the breast density changes with 12-month CE/BZA treatment was similar to placebo—which is markedly different from comparisons of placebo and combination estrogen-progestin therapy (EPT), where EPT increased breast density. If indeed this lack of an association translates into fewer breast cancers, it would be wonderful, but we do not have long-term data. We can tell our patients that using CE/BZA has not been shown to increase the risk of breast cancer, at least up to 2 years.

What makes CE/BZA different from traditional EPT?
Dr. Pinkerton There are two exciting differences:

 

  1. The incidences of breast pain and tenderness were found to be similar to placebo, and were significantly less than those with the comparator EPT (conjugated estrogens 0.45 mg plus medroxyprogesterone acetate [CE/MPA] 1.5 mg).9,10,12
  2. Bleeding and spotting rates were significantly less than those found with CE/MPA.13

In addition, high rates of amenorrhea have been found—comparable to placebo.13

CE/BZA is similar to traditional EPT in several ways. For instance, compared with placebo, at 2 years, CE/BZA was not found to increase the incidence of endometrial hyperplasia, endometrial thickness (increase from baseline was <1 mm and comparable to placebo), or endometrial cancers.14 Lastly, similar to EPT, there is probably a twofold risk of venous thromboembolism (VTE) with BZA 20 mg alone.15 Importantly, there has been no additive effect on VTE risk when combining CE with BZA; however, we will need longer studies, in older women, to fully evaluate this risk.1

 

 

Overall, in symptomatic postmenopausal women with a uterus, randomized controlled data show the same improvement with CE/BZA as that seen with traditional oral EPTs, with improvements in hot flashes; night sweats, with fewer sleep disruptions; and prevention of bone loss. In addition, the changes in cholesterol (an increase in triglyceride levels) and effect on the vagina are the same. Yet, CE/BZA appears to have a neutral effect on the breast and protects against endometrial hyperplasia and endometrial cancer without causing bleeding.9,10 CE/BZA’s VTE and stroke risks are expected to be similar to traditional oral EPT.

Therefore, the major benefit of CE/BZA for women who have a uterus is the lack of significant breast tenderness, lack of changes in breast density, and lack of vaginal bleeding that is often seen with traditional EPT.12

Then, is progestogen the harmful agent in traditional HT options?
Dr. Pinkerton There is evidence that estrogen plus progestogen therapy has more risk for breast cancer than estrogen alone. But in women who have a uterus, you need to protect against uterine cancer so, up until now, the only option was to add progestogen. Some studies suggest the risk of breast cancer may differ depending on the type of progestogen. So it’s a laudable goal to try to protect the endometrium without using a progestogen.

Given its safety profile, do you see CE/BZA being indicated for women without a uterus?
Dr. Pinkerton CE/BZA has been tested only in women with a uterus; there is no indication for using it in hysterectomized women. In the future, unless trial data show a benefit to hysterectomized women—by a reduction in breast cancer compared with estrogen alone—there would be no reason to add BZA to the CE for these women. You would just use CE or another type of estrogen alone.

Do you anticipate BZA being used alone?
Dr. Pinkerton For treating osteoporosis in postmenopausal women at increased fracture risk, BZA alone has greater benefits than risks. It is approved in other countries to prevent or treat osteoporosis. In 2008, Wyeth received an approval letter from the US Food and Drug Administration for BZA alone but, for whatever reason, the drug was not brought to market. BZA reduces the number of new lumbar spine fractures by 4% (vs 2% for placebo), with efficacy better in those with a higher risk of fractures. Like raloxifene, it has not been shown effective at reducing nonvertebral fractures, although it maintains spinal bone density.16

BZA available as monotherapy could tempt clinicians to pair it with other estrogens. We must recognize that the combination of the specific estrogen and BZA dose and type need to be balanced to provide endometrial hyperplasia protection. It would not be safe or effective to take BZA as a selective estrogen-receptor modulator and pair it with any other untested systemic estrogen. I do not anticipate, in this country, that BZA will become available as monotherapy.

New options are welcome
Dr. Moore Novel strategies for clinicians to optimally treat menopausal symptoms are always welcome. I look forward to more data from the SMART trials on CE/BZA and to moving forward as we gain experience with using this new treatment option.

Share your thoughts on this article! Send your Letter to the Editor to rbarbieri@frontlinemedcom.com. Please include your name and the city and state in which you practice.

References

 

1. Lobo RA, Pinkerton JV, Gass ML, et al. Evaluation of bazedoxifene/conjugated estrogens for the treatment of menopausal symptoms and effects on metabolic bone parameters and overall safety profile. Fertil Steril. 2009;92(3):1025–1038.
2. Pinkerton JV, Utian WH, Constantine GD, Olivier S, Pickar JH. Relief of vasomotor symptoms with the tissue-selective estrogen complex containing bazedoxifene/conjugated estrogens. Menopause. 2009;16:(6)1116–1124.
3. Pinkerton JV, Pan K, Abraham L, et al. Sleep parameters and health-related quality of life with bazedoxifene/conjugated estrogens. Menopause. 2014;21(3):252–259.
4. Lindsay R, Gallagher JC, Kagan R, Pickar JH, Constantine G. Efficacy of tissue-selective estrogen complex (TSEC) of bazedoxifene/conjugated estrogens (BZA/CE) for osteoporosis prevention in at-risk postmenopausal women. Fertil Steril. 2009;92(3):1045–1052.
5. Pinkerton JV, Harvey JA, Lindsay R, et al; SMART-5 Investigators. Effects of bazedoxifene/conjugated estrogens on the endometrium and bone. J Clin Endocrinol Metab. 2014;99(2):E189–E198.
6. Kagan R, Williams RS, Pan K, Mirkin S, Pickar JH. A randomized, placebo- and active-controlled trial of bazedoxifene/conjugated estrogens (BZA/CE) for treatment of moderate to severe vulvar/vaginal atrophy in postmenopausal women. Menopause. 2010;17(2):281–289.
7. Utian W, Yu H, Bobula J, Mirkin S, Olivier S, Pickar JH. Bazedoxifene/conjugated estrogens and quality of life in postmenopausal women. Maturitas. 2009;63:(4)329–335.
8. Abraham L, Pinkerton JV, Messig M, Ryan KA, Komm BS, Mirkin S. Menopause-specific quality of life across varying menopausal populations with conjugated estrogens/bazedoxifene. Maturitas. 2014;78(3):212–218.
9. Harvey JA, Pinkerton JV, Baracat EC, Shi H, Chines AA, Mirkin S. Breast density changes in a randomized controlled trial evaluating bazedoxifene/conjugated estrogens. Menopause. 2013;20:(2)138–145.
10. Pinkerton JV, Harvey JA, Pan K, et al. Breast effects of bazedoxifene-conjugated estrogens. Obstet Gynecol. 2013;121(5):959–968.
11. Kaunitz AM. When should a menopausal woman discontinue hormone therapy? OBG Manag. 2014;26(2):59–65.
12. Pinkerton JV, Pickar JH,  Racketa J, Mirkin S. Bazedoxifene/conjugated estrogens for menopausal symptom treatment and osteoporosis prevention. Climacteric. 2012;15:(5)411–418.
13. Archer DF, Lewis V, Carr BR, Olivier S, Pickar JH. Bazedoxifene/conjugated estrogens (BZA/CE): incidence of uterine bleeding in postmenopausal women. Fertil Steril. 2009;92:1039–1044.
14. Pickar JH, Yeh I-T, Bachmann G, Speroff L. Endometrial effects of a tissue selective estrogen complex (TSEC) containing bazedoxifene/conjugated estrogens as a menopausal therapy. Fertil Steril. 2009; 92(3):1018–1024.
15. Mirkin S, Komm BS. Tissue-selective estrogen complexes for postmenopausal women. Maturitas. 2013;76(3):213–220.
16. Ellis AG, Reginster JY, Luo X, et al. Bazedoxifene versus oral bisphosphonates for the prevention of nonvertebral fractures in postmenopausal women with osteoporosis at higher risk of fracture. Value Health. 2014;17(4):424–432.

Article PDF
Author and Disclosure Information

Anne A. Moore, DNP, APN

CASES IN MENOPAUSE
Brought to you by the menopause experts

Dr. Moore is Women’s Health Clinical Trainer, Tennessee Department of Health, Nashville, Tennessee. She serves on the OBG Management Board of Editors.

Dr. Pinkerton is Professor, Department of Obstetrics and Gynecology, and Director, Division of Midlife Women’s Health, at the University of Virginia in Charlottesville. She is a NAMS past president and certified menopause practitioner and serves on the OBG Management Board of ­Editors.

Disclosures
Dr. Moore reports no financial relationships relevant to this article. Dr. Pinkerton reports that her institution receives consulting fees from DepoMed, Noven, NovoNordisk, Pfizer, and Shionogi; current grant or research support from Therapeutics MD, prior support from DepoMed, Bionova, and Endoceutics, and, several years ago, support from Pfizer; and travel funds from DepoMed, Noven, NovoNordisk, Pfizer, Therapeutics MD, and Shionogi.

Issue
OBG Management - 26(10)
Publications
Topics
Page Number
52,53,54
Legacy Keywords
Anne A. Moore DNP,JoAnn V. Pinkerton MD,conjugated estrogen,CE,bazedoxifene,BZA,Duavee,menopause experts,Cases in Menopause,CE/BZA,postmenopausal women,hot flashes,menopausal symptoms,night sweats,sleep disruption,symptomatic vulvovaginal atrophy,VVA,osteoporosis,bone loss prevention,bone mineral density,painful intercourse,cardiovascular events,breast cancer,cerebrovascular events,endometrial cancer,ovarian cancer,obesity,estrogen-progestin therapy,EPT,breast density,medroxyprogesterone acetate,MPA,CE/MPA,amenorrhea,breast tenderness,vaginal bleeding,stroke risk,VTE,venous thromboembolism,cholesterol,hysterectomy,monotherapy,SMART
Sections
Author and Disclosure Information

Anne A. Moore, DNP, APN

CASES IN MENOPAUSE
Brought to you by the menopause experts

Dr. Moore is Women’s Health Clinical Trainer, Tennessee Department of Health, Nashville, Tennessee. She serves on the OBG Management Board of Editors.

Dr. Pinkerton is Professor, Department of Obstetrics and Gynecology, and Director, Division of Midlife Women’s Health, at the University of Virginia in Charlottesville. She is a NAMS past president and certified menopause practitioner and serves on the OBG Management Board of ­Editors.

Disclosures
Dr. Moore reports no financial relationships relevant to this article. Dr. Pinkerton reports that her institution receives consulting fees from DepoMed, Noven, NovoNordisk, Pfizer, and Shionogi; current grant or research support from Therapeutics MD, prior support from DepoMed, Bionova, and Endoceutics, and, several years ago, support from Pfizer; and travel funds from DepoMed, Noven, NovoNordisk, Pfizer, Therapeutics MD, and Shionogi.

Author and Disclosure Information

Anne A. Moore, DNP, APN

CASES IN MENOPAUSE
Brought to you by the menopause experts

Dr. Moore is Women’s Health Clinical Trainer, Tennessee Department of Health, Nashville, Tennessee. She serves on the OBG Management Board of Editors.

Dr. Pinkerton is Professor, Department of Obstetrics and Gynecology, and Director, Division of Midlife Women’s Health, at the University of Virginia in Charlottesville. She is a NAMS past president and certified menopause practitioner and serves on the OBG Management Board of ­Editors.

Disclosures
Dr. Moore reports no financial relationships relevant to this article. Dr. Pinkerton reports that her institution receives consulting fees from DepoMed, Noven, NovoNordisk, Pfizer, and Shionogi; current grant or research support from Therapeutics MD, prior support from DepoMed, Bionova, and Endoceutics, and, several years ago, support from Pfizer; and travel funds from DepoMed, Noven, NovoNordisk, Pfizer, Therapeutics MD, and Shionogi.

Article PDF
Article PDF
Related Articles

In this special installment of Cases in Menopause, I interview series contributor and menopause expert JoAnn V. Pinkerton, MD. We discuss a fairly new therapy: the combination conjugated estrogen and bazedoxifene (CE/BZA; Duavee) for the treatment of moderate to severe hot flashes due to menopause and the prevention of menopausal osteoporosis.

Much of my practice has focused on the treatment of menopausal women, but which of my patients can benefit from this particular combination of CE 0.45 mg plus BZA 20 mg? I asked Dr. Pinkerton this question, and more.

Which patients can benefit most?
Dr. Pinkerton CE/BZA was tested in healthy postmenopausal women with a uterus at risk for bone loss who were reporting 50 or more moderate to severe hot flashes per week. The combination of CE and BZA is a good choice for women who have bothersome menopausal symptoms: hot flashes, night sweats, and sleep disruption or symptomatic vulvovaginal atrophy (VVA)—although it’s not approved for VVA.

Efficacy and safety data show that compared with placebo:

 

  • CE/BZA decreases the frequency and severity of hot flashes at 12 weeks, and those decreases are maintained at 12 months.1,2
  • Women taking CE/BZA have greater improvements in sleep, with both decreased sleep disturbance and time to fall asleep.3
  • CE/BZA maintained or prevented lumbar spine and hip bone loss in postmenopausal women at risk for osteoporosis. 1,4,5

Although fracture data were not captured and the drug was not tested in osteoporotic women, study results showed bone loss prevention at 12 months, which was sustained at 24 months. The improvement in bone mineral density from baseline was about 1% to 1.5%. This was compared with a bone loss of 1.8% in women taking placebo (P<.01).

In clinical studies, women taking CE/BZA versus placebo also have reported a lower incidence of painful intercourse,6 and some improvement in health-related quality of life and treatment satisfaction.7,8

In short, CE/BZA is a good option for symptomatic menopausal women with a uterus who have bothersome hot flashes, night sweats, and sleep disruptions and want to prevent bone loss.

What about adverse effects?
Dr. Pinkerton In general, CE/BZA has a favorable safety and tolerability profile, with an overall incidence of adverse events similar to placebo. The rates of cardiovascular and cerebrovascular events, cancers (breast, endometrial, and ovarian), and mortality are comparable to placebo in 2-year trials. These data are limited; studies have been conducted in healthy postmenopausal women. Future studies need to define the full risk profile, particularly among overweight or obese women and different ethnic groups and for longer-term use.

Is there a role among women with breast cancer?
Dr. Pinkerton CE/BZA has not been tested in women at risk for or with prior breast cancer. In preclinical trials of up to 2 years, involving healthy postmenopausal women, the rates for breast cancer with CE/BZA were similar to placebo. There are no long-term data, however, and there are no data in women at risk for breast cancer. I recommend that women who have or are at high risk for breast cancer consider nonhormonal treatment options.9–11

Has there been an associated increase in breast density with CE/BZA?
Dr. Pinkerton No. Data from two randomized clinical trials showed that the breast density changes with 12-month CE/BZA treatment was similar to placebo—which is markedly different from comparisons of placebo and combination estrogen-progestin therapy (EPT), where EPT increased breast density. If indeed this lack of an association translates into fewer breast cancers, it would be wonderful, but we do not have long-term data. We can tell our patients that using CE/BZA has not been shown to increase the risk of breast cancer, at least up to 2 years.

What makes CE/BZA different from traditional EPT?
Dr. Pinkerton There are two exciting differences:

 

  1. The incidences of breast pain and tenderness were found to be similar to placebo, and were significantly less than those with the comparator EPT (conjugated estrogens 0.45 mg plus medroxyprogesterone acetate [CE/MPA] 1.5 mg).9,10,12
  2. Bleeding and spotting rates were significantly less than those found with CE/MPA.13

In addition, high rates of amenorrhea have been found—comparable to placebo.13

CE/BZA is similar to traditional EPT in several ways. For instance, compared with placebo, at 2 years, CE/BZA was not found to increase the incidence of endometrial hyperplasia, endometrial thickness (increase from baseline was <1 mm and comparable to placebo), or endometrial cancers.14 Lastly, similar to EPT, there is probably a twofold risk of venous thromboembolism (VTE) with BZA 20 mg alone.15 Importantly, there has been no additive effect on VTE risk when combining CE with BZA; however, we will need longer studies, in older women, to fully evaluate this risk.1

 

 

Overall, in symptomatic postmenopausal women with a uterus, randomized controlled data show the same improvement with CE/BZA as that seen with traditional oral EPTs, with improvements in hot flashes; night sweats, with fewer sleep disruptions; and prevention of bone loss. In addition, the changes in cholesterol (an increase in triglyceride levels) and effect on the vagina are the same. Yet, CE/BZA appears to have a neutral effect on the breast and protects against endometrial hyperplasia and endometrial cancer without causing bleeding.9,10 CE/BZA’s VTE and stroke risks are expected to be similar to traditional oral EPT.

Therefore, the major benefit of CE/BZA for women who have a uterus is the lack of significant breast tenderness, lack of changes in breast density, and lack of vaginal bleeding that is often seen with traditional EPT.12

Then, is progestogen the harmful agent in traditional HT options?
Dr. Pinkerton There is evidence that estrogen plus progestogen therapy has more risk for breast cancer than estrogen alone. But in women who have a uterus, you need to protect against uterine cancer so, up until now, the only option was to add progestogen. Some studies suggest the risk of breast cancer may differ depending on the type of progestogen. So it’s a laudable goal to try to protect the endometrium without using a progestogen.

Given its safety profile, do you see CE/BZA being indicated for women without a uterus?
Dr. Pinkerton CE/BZA has been tested only in women with a uterus; there is no indication for using it in hysterectomized women. In the future, unless trial data show a benefit to hysterectomized women—by a reduction in breast cancer compared with estrogen alone—there would be no reason to add BZA to the CE for these women. You would just use CE or another type of estrogen alone.

Do you anticipate BZA being used alone?
Dr. Pinkerton For treating osteoporosis in postmenopausal women at increased fracture risk, BZA alone has greater benefits than risks. It is approved in other countries to prevent or treat osteoporosis. In 2008, Wyeth received an approval letter from the US Food and Drug Administration for BZA alone but, for whatever reason, the drug was not brought to market. BZA reduces the number of new lumbar spine fractures by 4% (vs 2% for placebo), with efficacy better in those with a higher risk of fractures. Like raloxifene, it has not been shown effective at reducing nonvertebral fractures, although it maintains spinal bone density.16

BZA available as monotherapy could tempt clinicians to pair it with other estrogens. We must recognize that the combination of the specific estrogen and BZA dose and type need to be balanced to provide endometrial hyperplasia protection. It would not be safe or effective to take BZA as a selective estrogen-receptor modulator and pair it with any other untested systemic estrogen. I do not anticipate, in this country, that BZA will become available as monotherapy.

New options are welcome
Dr. Moore Novel strategies for clinicians to optimally treat menopausal symptoms are always welcome. I look forward to more data from the SMART trials on CE/BZA and to moving forward as we gain experience with using this new treatment option.

Share your thoughts on this article! Send your Letter to the Editor to rbarbieri@frontlinemedcom.com. Please include your name and the city and state in which you practice.

In this special installment of Cases in Menopause, I interview series contributor and menopause expert JoAnn V. Pinkerton, MD. We discuss a fairly new therapy: the combination conjugated estrogen and bazedoxifene (CE/BZA; Duavee) for the treatment of moderate to severe hot flashes due to menopause and the prevention of menopausal osteoporosis.

Much of my practice has focused on the treatment of menopausal women, but which of my patients can benefit from this particular combination of CE 0.45 mg plus BZA 20 mg? I asked Dr. Pinkerton this question, and more.

Which patients can benefit most?
Dr. Pinkerton CE/BZA was tested in healthy postmenopausal women with a uterus at risk for bone loss who were reporting 50 or more moderate to severe hot flashes per week. The combination of CE and BZA is a good choice for women who have bothersome menopausal symptoms: hot flashes, night sweats, and sleep disruption or symptomatic vulvovaginal atrophy (VVA)—although it’s not approved for VVA.

Efficacy and safety data show that compared with placebo:

 

  • CE/BZA decreases the frequency and severity of hot flashes at 12 weeks, and those decreases are maintained at 12 months.1,2
  • Women taking CE/BZA have greater improvements in sleep, with both decreased sleep disturbance and time to fall asleep.3
  • CE/BZA maintained or prevented lumbar spine and hip bone loss in postmenopausal women at risk for osteoporosis. 1,4,5

Although fracture data were not captured and the drug was not tested in osteoporotic women, study results showed bone loss prevention at 12 months, which was sustained at 24 months. The improvement in bone mineral density from baseline was about 1% to 1.5%. This was compared with a bone loss of 1.8% in women taking placebo (P<.01).

In clinical studies, women taking CE/BZA versus placebo also have reported a lower incidence of painful intercourse,6 and some improvement in health-related quality of life and treatment satisfaction.7,8

In short, CE/BZA is a good option for symptomatic menopausal women with a uterus who have bothersome hot flashes, night sweats, and sleep disruptions and want to prevent bone loss.

What about adverse effects?
Dr. Pinkerton In general, CE/BZA has a favorable safety and tolerability profile, with an overall incidence of adverse events similar to placebo. The rates of cardiovascular and cerebrovascular events, cancers (breast, endometrial, and ovarian), and mortality are comparable to placebo in 2-year trials. These data are limited; studies have been conducted in healthy postmenopausal women. Future studies need to define the full risk profile, particularly among overweight or obese women and different ethnic groups and for longer-term use.

Is there a role among women with breast cancer?
Dr. Pinkerton CE/BZA has not been tested in women at risk for or with prior breast cancer. In preclinical trials of up to 2 years, involving healthy postmenopausal women, the rates for breast cancer with CE/BZA were similar to placebo. There are no long-term data, however, and there are no data in women at risk for breast cancer. I recommend that women who have or are at high risk for breast cancer consider nonhormonal treatment options.9–11

Has there been an associated increase in breast density with CE/BZA?
Dr. Pinkerton No. Data from two randomized clinical trials showed that the breast density changes with 12-month CE/BZA treatment was similar to placebo—which is markedly different from comparisons of placebo and combination estrogen-progestin therapy (EPT), where EPT increased breast density. If indeed this lack of an association translates into fewer breast cancers, it would be wonderful, but we do not have long-term data. We can tell our patients that using CE/BZA has not been shown to increase the risk of breast cancer, at least up to 2 years.

What makes CE/BZA different from traditional EPT?
Dr. Pinkerton There are two exciting differences:

 

  1. The incidences of breast pain and tenderness were found to be similar to placebo, and were significantly less than those with the comparator EPT (conjugated estrogens 0.45 mg plus medroxyprogesterone acetate [CE/MPA] 1.5 mg).9,10,12
  2. Bleeding and spotting rates were significantly less than those found with CE/MPA.13

In addition, high rates of amenorrhea have been found—comparable to placebo.13

CE/BZA is similar to traditional EPT in several ways. For instance, compared with placebo, at 2 years, CE/BZA was not found to increase the incidence of endometrial hyperplasia, endometrial thickness (increase from baseline was <1 mm and comparable to placebo), or endometrial cancers.14 Lastly, similar to EPT, there is probably a twofold risk of venous thromboembolism (VTE) with BZA 20 mg alone.15 Importantly, there has been no additive effect on VTE risk when combining CE with BZA; however, we will need longer studies, in older women, to fully evaluate this risk.1

 

 

Overall, in symptomatic postmenopausal women with a uterus, randomized controlled data show the same improvement with CE/BZA as that seen with traditional oral EPTs, with improvements in hot flashes; night sweats, with fewer sleep disruptions; and prevention of bone loss. In addition, the changes in cholesterol (an increase in triglyceride levels) and effect on the vagina are the same. Yet, CE/BZA appears to have a neutral effect on the breast and protects against endometrial hyperplasia and endometrial cancer without causing bleeding.9,10 CE/BZA’s VTE and stroke risks are expected to be similar to traditional oral EPT.

Therefore, the major benefit of CE/BZA for women who have a uterus is the lack of significant breast tenderness, lack of changes in breast density, and lack of vaginal bleeding that is often seen with traditional EPT.12

Then, is progestogen the harmful agent in traditional HT options?
Dr. Pinkerton There is evidence that estrogen plus progestogen therapy has more risk for breast cancer than estrogen alone. But in women who have a uterus, you need to protect against uterine cancer so, up until now, the only option was to add progestogen. Some studies suggest the risk of breast cancer may differ depending on the type of progestogen. So it’s a laudable goal to try to protect the endometrium without using a progestogen.

Given its safety profile, do you see CE/BZA being indicated for women without a uterus?
Dr. Pinkerton CE/BZA has been tested only in women with a uterus; there is no indication for using it in hysterectomized women. In the future, unless trial data show a benefit to hysterectomized women—by a reduction in breast cancer compared with estrogen alone—there would be no reason to add BZA to the CE for these women. You would just use CE or another type of estrogen alone.

Do you anticipate BZA being used alone?
Dr. Pinkerton For treating osteoporosis in postmenopausal women at increased fracture risk, BZA alone has greater benefits than risks. It is approved in other countries to prevent or treat osteoporosis. In 2008, Wyeth received an approval letter from the US Food and Drug Administration for BZA alone but, for whatever reason, the drug was not brought to market. BZA reduces the number of new lumbar spine fractures by 4% (vs 2% for placebo), with efficacy better in those with a higher risk of fractures. Like raloxifene, it has not been shown effective at reducing nonvertebral fractures, although it maintains spinal bone density.16

BZA available as monotherapy could tempt clinicians to pair it with other estrogens. We must recognize that the combination of the specific estrogen and BZA dose and type need to be balanced to provide endometrial hyperplasia protection. It would not be safe or effective to take BZA as a selective estrogen-receptor modulator and pair it with any other untested systemic estrogen. I do not anticipate, in this country, that BZA will become available as monotherapy.

New options are welcome
Dr. Moore Novel strategies for clinicians to optimally treat menopausal symptoms are always welcome. I look forward to more data from the SMART trials on CE/BZA and to moving forward as we gain experience with using this new treatment option.

Share your thoughts on this article! Send your Letter to the Editor to rbarbieri@frontlinemedcom.com. Please include your name and the city and state in which you practice.

References

 

1. Lobo RA, Pinkerton JV, Gass ML, et al. Evaluation of bazedoxifene/conjugated estrogens for the treatment of menopausal symptoms and effects on metabolic bone parameters and overall safety profile. Fertil Steril. 2009;92(3):1025–1038.
2. Pinkerton JV, Utian WH, Constantine GD, Olivier S, Pickar JH. Relief of vasomotor symptoms with the tissue-selective estrogen complex containing bazedoxifene/conjugated estrogens. Menopause. 2009;16:(6)1116–1124.
3. Pinkerton JV, Pan K, Abraham L, et al. Sleep parameters and health-related quality of life with bazedoxifene/conjugated estrogens. Menopause. 2014;21(3):252–259.
4. Lindsay R, Gallagher JC, Kagan R, Pickar JH, Constantine G. Efficacy of tissue-selective estrogen complex (TSEC) of bazedoxifene/conjugated estrogens (BZA/CE) for osteoporosis prevention in at-risk postmenopausal women. Fertil Steril. 2009;92(3):1045–1052.
5. Pinkerton JV, Harvey JA, Lindsay R, et al; SMART-5 Investigators. Effects of bazedoxifene/conjugated estrogens on the endometrium and bone. J Clin Endocrinol Metab. 2014;99(2):E189–E198.
6. Kagan R, Williams RS, Pan K, Mirkin S, Pickar JH. A randomized, placebo- and active-controlled trial of bazedoxifene/conjugated estrogens (BZA/CE) for treatment of moderate to severe vulvar/vaginal atrophy in postmenopausal women. Menopause. 2010;17(2):281–289.
7. Utian W, Yu H, Bobula J, Mirkin S, Olivier S, Pickar JH. Bazedoxifene/conjugated estrogens and quality of life in postmenopausal women. Maturitas. 2009;63:(4)329–335.
8. Abraham L, Pinkerton JV, Messig M, Ryan KA, Komm BS, Mirkin S. Menopause-specific quality of life across varying menopausal populations with conjugated estrogens/bazedoxifene. Maturitas. 2014;78(3):212–218.
9. Harvey JA, Pinkerton JV, Baracat EC, Shi H, Chines AA, Mirkin S. Breast density changes in a randomized controlled trial evaluating bazedoxifene/conjugated estrogens. Menopause. 2013;20:(2)138–145.
10. Pinkerton JV, Harvey JA, Pan K, et al. Breast effects of bazedoxifene-conjugated estrogens. Obstet Gynecol. 2013;121(5):959–968.
11. Kaunitz AM. When should a menopausal woman discontinue hormone therapy? OBG Manag. 2014;26(2):59–65.
12. Pinkerton JV, Pickar JH,  Racketa J, Mirkin S. Bazedoxifene/conjugated estrogens for menopausal symptom treatment and osteoporosis prevention. Climacteric. 2012;15:(5)411–418.
13. Archer DF, Lewis V, Carr BR, Olivier S, Pickar JH. Bazedoxifene/conjugated estrogens (BZA/CE): incidence of uterine bleeding in postmenopausal women. Fertil Steril. 2009;92:1039–1044.
14. Pickar JH, Yeh I-T, Bachmann G, Speroff L. Endometrial effects of a tissue selective estrogen complex (TSEC) containing bazedoxifene/conjugated estrogens as a menopausal therapy. Fertil Steril. 2009; 92(3):1018–1024.
15. Mirkin S, Komm BS. Tissue-selective estrogen complexes for postmenopausal women. Maturitas. 2013;76(3):213–220.
16. Ellis AG, Reginster JY, Luo X, et al. Bazedoxifene versus oral bisphosphonates for the prevention of nonvertebral fractures in postmenopausal women with osteoporosis at higher risk of fracture. Value Health. 2014;17(4):424–432.

References

 

1. Lobo RA, Pinkerton JV, Gass ML, et al. Evaluation of bazedoxifene/conjugated estrogens for the treatment of menopausal symptoms and effects on metabolic bone parameters and overall safety profile. Fertil Steril. 2009;92(3):1025–1038.
2. Pinkerton JV, Utian WH, Constantine GD, Olivier S, Pickar JH. Relief of vasomotor symptoms with the tissue-selective estrogen complex containing bazedoxifene/conjugated estrogens. Menopause. 2009;16:(6)1116–1124.
3. Pinkerton JV, Pan K, Abraham L, et al. Sleep parameters and health-related quality of life with bazedoxifene/conjugated estrogens. Menopause. 2014;21(3):252–259.
4. Lindsay R, Gallagher JC, Kagan R, Pickar JH, Constantine G. Efficacy of tissue-selective estrogen complex (TSEC) of bazedoxifene/conjugated estrogens (BZA/CE) for osteoporosis prevention in at-risk postmenopausal women. Fertil Steril. 2009;92(3):1045–1052.
5. Pinkerton JV, Harvey JA, Lindsay R, et al; SMART-5 Investigators. Effects of bazedoxifene/conjugated estrogens on the endometrium and bone. J Clin Endocrinol Metab. 2014;99(2):E189–E198.
6. Kagan R, Williams RS, Pan K, Mirkin S, Pickar JH. A randomized, placebo- and active-controlled trial of bazedoxifene/conjugated estrogens (BZA/CE) for treatment of moderate to severe vulvar/vaginal atrophy in postmenopausal women. Menopause. 2010;17(2):281–289.
7. Utian W, Yu H, Bobula J, Mirkin S, Olivier S, Pickar JH. Bazedoxifene/conjugated estrogens and quality of life in postmenopausal women. Maturitas. 2009;63:(4)329–335.
8. Abraham L, Pinkerton JV, Messig M, Ryan KA, Komm BS, Mirkin S. Menopause-specific quality of life across varying menopausal populations with conjugated estrogens/bazedoxifene. Maturitas. 2014;78(3):212–218.
9. Harvey JA, Pinkerton JV, Baracat EC, Shi H, Chines AA, Mirkin S. Breast density changes in a randomized controlled trial evaluating bazedoxifene/conjugated estrogens. Menopause. 2013;20:(2)138–145.
10. Pinkerton JV, Harvey JA, Pan K, et al. Breast effects of bazedoxifene-conjugated estrogens. Obstet Gynecol. 2013;121(5):959–968.
11. Kaunitz AM. When should a menopausal woman discontinue hormone therapy? OBG Manag. 2014;26(2):59–65.
12. Pinkerton JV, Pickar JH,  Racketa J, Mirkin S. Bazedoxifene/conjugated estrogens for menopausal symptom treatment and osteoporosis prevention. Climacteric. 2012;15:(5)411–418.
13. Archer DF, Lewis V, Carr BR, Olivier S, Pickar JH. Bazedoxifene/conjugated estrogens (BZA/CE): incidence of uterine bleeding in postmenopausal women. Fertil Steril. 2009;92:1039–1044.
14. Pickar JH, Yeh I-T, Bachmann G, Speroff L. Endometrial effects of a tissue selective estrogen complex (TSEC) containing bazedoxifene/conjugated estrogens as a menopausal therapy. Fertil Steril. 2009; 92(3):1018–1024.
15. Mirkin S, Komm BS. Tissue-selective estrogen complexes for postmenopausal women. Maturitas. 2013;76(3):213–220.
16. Ellis AG, Reginster JY, Luo X, et al. Bazedoxifene versus oral bisphosphonates for the prevention of nonvertebral fractures in postmenopausal women with osteoporosis at higher risk of fracture. Value Health. 2014;17(4):424–432.

Issue
OBG Management - 26(10)
Issue
OBG Management - 26(10)
Page Number
52,53,54
Page Number
52,53,54
Publications
Publications
Topics
Article Type
Display Headline
Conjugated estrogen plus bazedoxifene—a new approach to estrogen therapy
Display Headline
Conjugated estrogen plus bazedoxifene—a new approach to estrogen therapy
Legacy Keywords
Anne A. Moore DNP,JoAnn V. Pinkerton MD,conjugated estrogen,CE,bazedoxifene,BZA,Duavee,menopause experts,Cases in Menopause,CE/BZA,postmenopausal women,hot flashes,menopausal symptoms,night sweats,sleep disruption,symptomatic vulvovaginal atrophy,VVA,osteoporosis,bone loss prevention,bone mineral density,painful intercourse,cardiovascular events,breast cancer,cerebrovascular events,endometrial cancer,ovarian cancer,obesity,estrogen-progestin therapy,EPT,breast density,medroxyprogesterone acetate,MPA,CE/MPA,amenorrhea,breast tenderness,vaginal bleeding,stroke risk,VTE,venous thromboembolism,cholesterol,hysterectomy,monotherapy,SMART
Legacy Keywords
Anne A. Moore DNP,JoAnn V. Pinkerton MD,conjugated estrogen,CE,bazedoxifene,BZA,Duavee,menopause experts,Cases in Menopause,CE/BZA,postmenopausal women,hot flashes,menopausal symptoms,night sweats,sleep disruption,symptomatic vulvovaginal atrophy,VVA,osteoporosis,bone loss prevention,bone mineral density,painful intercourse,cardiovascular events,breast cancer,cerebrovascular events,endometrial cancer,ovarian cancer,obesity,estrogen-progestin therapy,EPT,breast density,medroxyprogesterone acetate,MPA,CE/MPA,amenorrhea,breast tenderness,vaginal bleeding,stroke risk,VTE,venous thromboembolism,cholesterol,hysterectomy,monotherapy,SMART
Sections
Article PDF Media

When should a menopausal woman discontinue hormone therapy?

Article Type
Changed
Tue, 08/28/2018 - 11:03
Display Headline
When should a menopausal woman discontinue hormone therapy?

CASE: AFTER 3 YEARS OF HT, A PATIENT ASKS WHETHER IT’S TIME TO QUIT

My menopausal patient is a 57-year-old woman with a body mass index (BMI) of 21 kg/m2. Her mother, who also was slender, suffered a hip fracture at age 74.

When this patient was 53, approximately 8 months after her last menstrual period, she scheduled a problem visit to discuss bothersome hot flushes, which occurred primarily at night. These symptoms were associated with sleep disruption and irritability. At that problem visit, the patient and I discussed the benefits and risks of menopausal hormone therapy (HT), and she elected to initiate it, choosing transdermal estradiol using an 0.05-mg patch, combined with oral micronized progesterone 100-mg (one capsule) at bedtime. Two months later, she telephoned my office to report that she was experiencing only moderate relief of her symptoms. I increased the dose of estradiol to 0.075 mg. On her next well-woman visit, the patient remarked that her symptoms were largely resolved and said that she wished to continue the regimen.

Now, as she presents for her well-woman visit 3 years later, she asks how long she should continue the HT.

How would you counsel such a patient?

Although the duration of HT is still marked by controversy, clinicians often encounter the issue in practice. As the North American Menopause Society (NAMS) notes in a recent Practice Pearl and in its 2012 Position Statement on hormone therapy, the determination of the optimal duration of HT can be a challenge for clinicians and patients.1,2

In this article, I discuss indications for HT and consider variables that may influence its duration. I also offer practical guidance on therapeutic options for women who elect to use HT for an extended duration.

HOT FLUSHES CAN BE A LONG-TERM CONCERN
Moderate to severe vasomotor symptoms (VMS) are the most common indication for systemic combination estrogen-progestin or estrogen-only HT—and HT is the most effective treatment for VMS.2

Some experts have cautioned that “it remains prudent to keep the…duration of treatment short” or that HT “may serve a useful role in short-term symptom management.”3,4 However, for many menopausal women, VMS are a long-term concern. The Penn Ovarian Aging Study was conducted to estimate the duration of moderate-to-severe VMS and found a median duration of such symptoms of more than 10 years. In this landmark cohort study, the median duration of VMS, which began near the time of the menopausal transition, was almost 12 years.5

In a study of older menopausal women (mean age, 67 years; mean time since menopause, 19 years), 11.8% reported “clinically significant” hot flushes and “more than half of these women who complained of significant hot flushes at baseline continued to report bothersome symptoms after 3 years.”6

These observations underscore the fact that, in many women, short-term use (3–5 years) of HT will not be sufficient to control bothersome VMS.

SYSTEMIC HT ALSO BENEFITS BONE
The standard daily dose of HT (eg, conjugated equine estrogens [CEE], 0.625 mg; micronized estradiol, 1.0 mg; or transdermal estradiol, 0.05 mg) for relief of VMS also prevents osteoporosis,2 with many HT formulations approved for prevention of this condition. Randomized trial data from the Women’s Health Initiative (WHI) also have confirmed that a standard dose of HT prevents fractures in menopausal women.2

However, in menopausal women, doses of estrogen therapy substantially lower than those commonly used to treat VMS can still maintain or improve bone mineral density (BMD). For example, serum estradiol levels remain in the menopausal range during use of the weekly estradiol ultra-low-dose patch (0.014 mg).7 In a clinical trial of women (mean age, 66 years) with an intact uterus, use of this ultra-low-dose estradiol patch for 2 years without progestin did not increase the risk of endometrial hyperplasia7—although this patch does appear to increase the incidence of endometrial proliferation.8 For this reason, periodic endometrial monitoring may be appropriate in women using the 0.014-mg estradiol patch over the long term, including vaginal ultrasound assessment of endometrial thickness. Package labeling for this patch recommends that women with an intact uterus be given a progestogen for 14 days every 6 to 12 months.9

Related Article: Update on Osteoporosis Steven R. Goldstein, MD (December 2013)

Although the ultra-low-dose estradiol patch is approved for the prevention of osteoporosis, its efficacy in treating VMS is uncertain. For instance, in a study of this patch in women aged 60 to 80 years, with skeletal health and safety as the primary outcomes, 16% of participants reported VMS at baseline. The 0.014-mg estradiol patch did not reduce VMS more than placebo.10 However, in a trial of the 0.014-mg estradiol patch in which impact on VMS was the primary outcome, the ultra-low-dose patch did relieve VMS.11 (The ultra-low-dose patch currently is not approved to treat VMS.) Low-dose CEE (0.3 mg, 0.45 mg) and low-dose oral estradiol (0.5 mg) have been found to be effective in the treatment of VMS.12,13

 

 

Data on the risk of osteoporotic fractures among women using the ultra-low-dose estradiol patch are not available.

Use of HT to prevent osteoporosis is appropriate for women who have other indications for HT, such as VMS. For women using HT who no longer experience VMS, long-term use of HT for osteoporosis is controversial. However, it may be considered for women at elevated risk for osteoporosis when skeleton-specific treatments (eg, bisphosphonates) are not tolerated or when such women prefer not to use skeleton-specific therapy.

FDA package labeling for systemic HT indicates that, “When prescribing solely for the prevention of postmenopausal osteoporosis, therapy only should be considered for women at significant risk of osteoporosis, and non-estrogen medications should be carefully considered.”14

The NAMS 2012 Position Statement on HT states: “Provided that the woman is well aware of the potential benefits and risks and has clinical supervision, extending [estrogen-progestin therapy] use with the lowest effective dose is acceptable under some circumstances, including 1) for the woman who has determined that the benefits of menopause symptom relief outweigh risks, notably after failing an attempt to stop [estrogen-progestin therapy], and 2) for the woman at high risk of fracture for whom alternate therapies are not appropriate or cause unacceptable adverse effects.”2

A 2014 Practice Bulletin from the American College of Obstetricians and Gynecologists (ACOG) on the management of menopausal symptoms states: “The decision to continue HT should be individualized and be based on a woman’s symptoms and the risk–benefit ratio, regardless of age. Because some women aged 65 years and older may continue to need systemic HT for the management of vasomotor symptoms, ACOG recommends against routine discontinuation of systemic estrogen at age 65. As with younger women, use of HT and estrogen therapy should be individualized based on each woman’s risk–benefit ratio and clinical presentation.”15

As I have detailed, doses of HT that are lower than those used to treat VMS can prevent loss of BMD. Accordingly, clinicians prescribing HT for the sole indication of osteoporosis prevention should use doses lower than those for standard HT. Moreover, clinicians prescribing HT specifically to prevent osteoporosis should recognize the elevated risk of breast cancer with estrogen-progestin therapy. Extended use of estrogen-only therapy is more appropriate for this indication.

Related Article: Your menopausal patient's breast biopsy reveals atypical hyperplasia JoAnn V. Pinkerton, MD (Cases in Menopause, May 2013)

While estrogen-only therapy is common in women following hysterectomy, ultra-low-dose estrogen therapy with regular endometrial monitoring also can be considered in women with an intact uterus.

Also be aware that BMD declines rapidly after discontinuation of HT (in contrast with bisphosphonates), so alternative agents to maintain BMD should be considered when HT is stopped.16

HOW SAFE IS EXTENDED USE OF SYSTEMIC HT?
The incidence of breast cancer and mortality from breast cancer increase after 3 to 5 years of estrogen-progestin therapy, and the risk of stroke remains elevated throughout use of combination as well as estrogen-only HT.2 Women with an intact uterus who choose to extend the duration of combination therapy beyond 5 years for control of VMS or protection against osteoporosis, or both, need to be candidly counseled about these concerns. No increase in the risk of breast cancer was observed in the estrogen-only arm of the WHI randomized, clinical trial (mean duration of CEE therapy of 7.1 years).17

James A. Simon, MD:
Not only was there no increase in the risk of breast cancer in the estrogen-only arm, but the therapy was associated with a decrease in risk that persisted even following discontinuation of the therapy.

 JoAnn V. Pinkerton, MD:
Yes, after 7 years of follow-up, women taking CEE (0.625 mg daily) had a reduction in the risk of breast cancer that translated into a decrease in mortality.17

Related Article: Stop enforcing a 5-year rule for menopausal hormone therapy Robert L. Reid, MD (Stop/Start, December 2013)

Long-term risks of oral estrogen
Among women who initiate HT at the time of menopause, long-term use does not appear to increase the risk of coronary heart disease (CHD), although follow-up in clinical trials has not extended beyond 7 years for estrogen-progestin therapy, and midlife may bring increases in a woman’s baseline cardiovascular risk.2 However, in the WHI, women who initiated oral estrogen-only or estrogen-progestin therapy later in menopause experienced an increased risk of CHD,18 underscoring the need for caution and individualization in this patient population.

Oral HT increases the risk of venous thromboembolism (VTE) and stroke.2 In addition, age is an independent risk factor for these two outcomes. Observational studies suggest that, in contrast with oral estrogen, transdermal HT does not increase the risk of VTE.19–24 Randomized trial data are lacking.

 

 

Similarly, one observational study suggests that low-dose (≤0.05 mg) transdermal estradiol does not appear to increase the risk of stroke,25 but, again, clinical trial data are unavailable.

Given these apparent safety advantages, transdermal estrogen therapy would appear to be preferable to oral estrogen in older, long-term users, a perspective supported by ACOG.26

Related Article: Is one oral estrogen formulation safer than another for menopausal women? Andrew M. Kaunitz, MD (Examining the Evidence, January 2014)

In regard to VTE, oral estradiol appears to be safer than CEE.27 Accordingly, oral estradiol may be preferable for older long-term users who don’t tolerate transdermal estradiol due to local skin reactions or costs. Oral estradiol also is less expensive than CEE.

What to expect when your patient discontinues systemic HT
VMS may recur is as many as 50% of women after they discontinue HT. The likelihood of recurring VMS does not appear to vary between abrupt and tapered discontinuation.2

Some HT users may be reluctant to reduce their dose or discontinue HT, particularly those who experienced severe VMS originally. In my clinical experience, many of these women are receptive to a trial of lower-dose HT, especially when I advise them that they can resume their original (higher) dose should bothersome VMS recur.

JoAnn V. Pinkerton, MD:
I use a similar approach with my patients, advising them to try 3 months off HT with the understanding that they can resume the therapy if they develop bothersome symptoms.

Individualized assessment of HT benefits and risks and shared decision-making play important roles in the management of these patients. As the dose of HT declines, or systemic HT is discontinued, symptoms of genital atrophy may become more prominent and, in the absence of indications for systemic HT (bothersome VMS or prevention of osteoporosis), may best be addressed with vaginal estrogen therapy or ospemifene.

Extended use of vaginal estrogen
Unlike VMS, untreated genital atrophy may continue to progress as women age, ­sometimes necessitating use of vaginal estrogen. Because the clinical trials that served as the basis for FDA approval of vaginal estrogen formulations did not find an elevated risk of endometrial hyperplasia, routine use of a progestin to prevent endometrial proliferation in women with an intact uterus is not recommended.28 However, these trials were too limited in duration to assure long-term endometrial safety. All postmenopausal women using vaginal ET should be advised to report any vaginal bleeding, and that bleeding should be evaluated appropriately.

JoAnn V. Pinkerton, MD:
I recommend transvaginal ultrasound and endometrial biopsy for women using vaginal estrogen who report spotting or bleeding.

Although low-dose local or vaginal estrogen therapy has not been studied in clinical trials beyond 1 year, it is thought to carry significantly fewer risks than systemic HT.28 Several studies have confirmed that serum estrogen levels remain in the postmenopausal range in women using low-dose vaginal estrogen, specifically the 3-month estradiol ring (2 mg) or twice-weekly estradiol tablets (10 µg).28

Besides relieving vaginal dryness and dyspareunia, low-dose vaginal estrogen also may improve overactive bladder and reduce the incidence of recurrent urinary tract infection.29,30

CASE: RESOLVED
The 57-year-old patient has been essentially symptom-free for the past 3 years using an estradiol patch (0.075 mg) with progesterone (100 mg nightly). Now she asks how long she should continue HT, and I explain that the duration of bothersome VMS is different in each woman. I also counsel her that hot flushes do resolve over time in almost all women. When she asks how likely it is that bothersome VMS will recur if she simply stops HT, I explain that bothersome symptoms often last 10 years or longer, and I remind her that her VMS began some 4 years earlier.

I also briefly review HT benefits (treatment of VMS as well as prevention of vulvovaginal atrophy and osteoporosis) and risks (small increased risk of breast cancer and stroke). I suggest a reduction in her HT dose as a reasonable method to determine her ongoing need for HT, telling her that she should know within about 1 month how she feels on the lower dose (0.05-mg patch). I also advise her to call my office if bothersome VMS recur on the lower dose so that I can increase the dose back to its original level.

After her estradiol dose is reduced, the patient reports only minimal VMS, and she opts to continue the estradiol patch (0.05 mg) with nightly progesterone (100 mg) for another 2 years.

At age 59, during her well-woman visit, she decides to lower the estradiol further, transitioning to a 0.0375-mg patch but maintaining the nightly progesterone (100 mg). She reports no VMS on this new regimen.

 

 

At age 61, because of her maternal history of osteoporosis and her own low BMI, the patient undergoes BMD assessment with dual x-ray absorptiometry (DXA). In average-risk women, NAMS recommends that BMD assessment be performed at age 65.31 The results of BMD assessment are normal.

After further discussion, the patient agrees to an even lower dose of estradiol, switching to an 0.025-mg patch, with progesterone (100 mg nightly) administered for 2 weeks in every 3-month interval. She reports no VMS or vaginal bleeding on this lower-dose HT regimen.

After 12 months on this new regimen, the patient undergoes vaginal sonography, revealing an endometrial thickness of 3 mm. She continues this regimen, including annual vaginal ultrasound assessment of the endometrium, without problems until her well-woman visit at age 65.

At that visit, I explain that discontinuation of HT is unlikely to trigger recurred VMS but may cause her to lose BMD rapidly for several years, and may also result in unpleasant symptoms from vulvovaginal atrophy including sexual discomfort. She decides to switch to an 0.014-mg estradiol patch without progesterone, and to undergo ultrasound assessment of her endometrium every 1 to 2 years.

Do you have a troubling case in menopause?
Suggest it to our expert panel. They may address your management dilemma in a future issue!
Email us at obg@frontlinemedcom.com

BOTTOM LINE: INDIVIDUALIZE THE DURATION OF HT
Although published data on extended use of HT are few, many clinicians caring for menopausal women are asked to make a recommendation. Because extended use of estrogen-progestin HT increases the risk of breast cancer, estrogen-only HT has a more favorable benefit-risk ratio. If a patient uses estrogen-progestin HT for an extended duration, periodic discussions about the elevated risk of breast cancer are appropriate.

JoAnn V. Pinkerton, MD:
The risk of breast cancer associated with extended use of estrogen-progestin HT likely is reduced if lower doses are given. Overall, however, the risk appears to be both dose- and duration-dependent.

We lack randomized trial data on CHD and other risks in women who begin HT at the time of menopause and continue it for decades. In older women who use HT for an extended duration, transdermal estrogen may be safer in regard to the risk of VTE and stroke.

As the systemic estrogen dose is lowered, it is possible to reduce the dose of the progestin (the sole function of which is to protect the endometrium from estrogen stimulation). Intermittent dosing can be used, although we lack long-term safety data, and periodic endometrial evaluation should be considered.

Remember also that, with intermittent or daily dosing of a progestin, you are relying on the patient to take this medication to protect the endometrium.

Extended use of low-dose vaginal estrogen HT may be necessary to treat symptoms of vulvovaginal atrophy, which tend to worsen over time. Administration of a progestin is not currently recommended with use of vaginal estrogen, but long-term use may increase the risk of endometrial stimulation.

Read other CASES IN MENOPAUSE
Your postmenopausal patient reports a history of migraine
James A. Simon, MD (October 2013)
Your menopausal patient's breast biopsy reveals atypical hyperplasia
JoAnn V. Pinkerton, MD (May 2013)

WE WANT TO HEAR FROM YOU!
Drop us a line and let us know what you think about current articles, which topics you'd like to see covered in future issues, and what challenges you face in daily practice. Tell us what you think by emailing us at: obg@frontlinemedcom.com

References

  1. Kaunitz AM. NAMS Practice Pearl: Extended duration use of menopausal hormone therapy [published online ahead of print January 6, 2014]. Menopause.
  2. North American Menopause Society. Position Statement: The 2012 hormone therapy position statement of the North American Menopause Society. Menopause. 2012;19(3):257–271.
  3. Nelson HD. Postmenopausal estrogen for treatment of hot flashes: clinical applications. JAMA. 2004;291(13):1621–1625.
  4. Hulley SB, Grady D. The WHI estrogen-alone trial—do things look any better? JAMA. 2004;291(14):1769–1771.
  5. Freeman EW, Sammel MD, Lin H, Liu Z, Gracia CR. Duration of menopausal hot flushes and associated risk factors. Obstet Gynecol. 2011;117(5):1095–1104.
  6. Huang AJ, Grady D, Jacoby VL, Blackwell TL, Bauer DC, Sawaya GF. Persistent hot flushes in older postmenopausal women. Arch Intern Med. 2008;168(8):840–846.
  7. Ettinger B, Ensrud KE, Wallace R, Johnson KC, Cummings SR, Yankov V, Vittinghoff  E, Grady D. Effects of ultralow-dose transdermal estradiol on bone mineral density: A randomized clinical trial. Obstet Gynecol. 2004;104(3):443–451.
  8. Johnson SR, Ettinger B, Macer JL, Ensrud KE, Quan J, Grady D. Uterine and vaginal effects of unopposed ultralow-dose transdermal estradiol. Obstet Gynecol. 2005;105(4):779–787.
  9. Menostar [package insert]. Wayne, NJ: Bayer; 2007.
  10. Diem S, Grady D, Quan J, Vittinghoff  E, Wallace R, Hanes V, Ensrud K. Effects of ultralow-dose transdermal estradiol on postmenopausal symptoms in women aged 60 to 80 years. Menopause. 2006;13(1):130–138.
  11. Bachmann GA, Schaefers M, Uddin A, Utian WH. Lowest effective transdermal 17beta-estradiol dose for relief of hot flushes in postmenopausal women: A randomized controlled trial. Obstet Gynecol. 2007;110(4):771–779.
  12. Honjo H, Taketani Y. Low-dose estradiol for climacteric symptoms in Japanese women: A randomized, controlled trial. Climacteric. 2009;12(4):319–328. 
  13. Utian WH, Shoupe D, Bachmann G, Pinkerton JV, Pickar JH. Relief of vasomotor symptoms and vaginal atrophy with lower doses of conjugated equine estrogens and medroxyprogesterone acetate. Fertil Steril. 2001;75(6):1065–1079.
  14. Premarin [package insert]. Philadelphia, PA: Pfizer; 2012.
  15. American College of Obstetricians and Gynecologists. Practice Bulletin No. 141: Management of menopausal symptoms. Obstet Gynecol. 2014;123(1):202–216.
  16. National Osteoporosis Foundation. Clinician’s Guide to Prevention and Treatment of Osteoporosis. Washington, DC: National Osteoporosis Foundation; 2013.
  17. Anderson GL, Chlebowski RT, Aragaki AK, et al. Conjugated equine oestrogen and breast cancer incidence and mortality in postmenopausal women with hysterectomy: Extended follow-up of the Women’s Health Initiative randomised placebo-controlled trial. Lancet Oncol. 2012;13(5):476–486.
  18. Rossouw JE, Prentice RL, Manson JE, et al. Postmenopausal hormone therapy and risk of cardiovascular disease by age and years since menopause. JAMA. 2007;4:297(13):1465–1477.
  19. Canonico M, Fournier A, Carcaillon L, et al. Postmenopausal hormone therapy and risk of idiopathic venous thromboembolism: results from the E3N cohort study. Arterioscler Thromb Vasc Biol. 2010;30(2):340–345.
  20. Sweetland S, Beral V, Balkwill A, et al; The Million Women Study Collaborators. Venous thromboembolism risk in relation to use of different types of postmenopausal hormone therapy in a large prospective study [published online ahead of print September 10, 2012]. J Thromb Haemost. 2012. doi:10.1111/j.1538-7836.2012.04919.x.
  21. Roach RE, Lijfering WM, Helmerhorst FM, Cannegieter SC, Rosendaal FR, van Hylckama Vlieg A. The risk of venous thrombosis in women over 50 years old using oral contraception or postmenopausal hormone therapy. J Thromb Haemost. 2013;11(1):124–131.
  22. Renoux C, Dell’Aniello S, Suissa S. Hormone replacement therapy and the risk of venous thromboembolism: A population-based study. J Thromb Haemost. 2010;8(5):979–986.
  23. Laliberté F, Dea K, Duh MS, Kahler KH, Rolli M, Lefebvre P. Does the route of administration for estrogen hormone therapy impact the risk of venous thromboembolism? Estradiol transdermal system versus oral estrogen-only hormone therapy. Menopause. 2011;18(10):1052–1059.
  24. Scarabin PY, Oger E, Plu-Bureau G; EStrogen and THromboEmbolism Risk Study Group. Differential association of oral and transdermal oestrogen-replacement therapy with venous thromboembolism risk. Lancet. 2003;362(9382):428–432.
  25. Renoux C, Dell’Aniello S, Garbe E, Suissa S. Transdermal and oral hormone replacement therapy and the risk of stroke: A nested case-control study. BMJ. 2010;340:c2519. doi:10.1136/bmj.c2519.
  26. American College of Obstetricians and Gynecologists. Committee Opinion No. 556: Postmenopausal estrogen therapy: Route of administration and risk of venous thromboembolism. Obstet Gynecol. 2013;121(4):887–890.
  27. Smith NL, Blondon M, Wiggins KL, Harrington LB, van Hylckama Vlieg A, Floyd JS. Lower risk of cardiovascular events in postmenopausal women taking oral estradiol compared with oral conjugated equine estrogens. JAMA Intern Med. 2014;14(1):25–31.
  28. North American Menopause Society. 2013 Position Statement: Management of symptomatic vulvovaginal atrophy. Menopause. 2013;20(9):888–902.
  29. Nelken RS, Ozel BZ, Leegant AR, Felix JC, Mishell DR. Randomized trial of estradiol vaginal ring versus oral oxybutynin for the treatment of overactive bladder. Menopause. 2011;18(9):962–966.
  30. Eriksen BC. A randomized, open, parallel-group study on the preventive effect of an estradiol-releasing vaginal ring (Estring) on recurrent urinary tract infections in postmenopausal women. Am J Obstet Gynecol. 1999;180(5):1072–1079.
  31. North American Menopause Society. 2010 Position Statement: Management of osteoporosis in postmenopausal women. Menopause. 2010;17(1):25–54.
Article PDF
Author and Disclosure Information

Andrew M. Kaunitz, MD

Dr. JoAnn V. Pinkerton and Dr. James A. Simon provided peer review and comments for Dr. Kaunitz's case study.

CASES IN MENOPAUSE
Brought to you by the menopause experts

Andrew M. Kaunitz, MD
Professor and Associate Chairman, Department of Obstetrics and Gynecology, University of Florida College of Medicine–Jacksonville. Dr. Kaunitz is a NAMS Board member and certified menopause practitioner. He also serves on the OBG Management Board of Editors.

JoAnn V. Pinkerton, MD
Professor, Department of Obstetrics and Gynecology, and Director, Division of Midlife Women’s Health, University of Virginia. Dr. Pinkerton is a North American Menopause Society (NAMS) past president and certified menopause practitioner. She also serves on the OBG Management Board of Editors.

James A. Simon, MD
Clinical Professor, Department of Obstetrics and Gynecology, George Washington University, and Medical Director, Women’s Health & Research Consultants, Washington, DC. Dr. Simon is a NAMS past president, a certified menopause practitioner, and a certified clinical densitometrist. He also serves on the OBG Management Board of Editors.

Disclosures
Dr. Kaunitz reports that his institution receives grant or research support from Bayer, Endoceutics, Noven, and Teva, and that he is a consultant to Actavis, Bayer, DepoMed, and Teva.

Dr. Pinkerton reports that her institution receives consulting fees from DepoMed, Noven, NovoNordisk, Pfizer, and Shionogi; grant or research support from DepoMed, Bionova, and Endoceutics; and travel funds from DepoMed, Noven, NovoNordisk, Pfizer, and Shionogi.

Dr. Simon reports being a consultant to or on the advisory boards of Abbott Laboratories, Amgen, Ascend Therapeutics, DepoMed, Lelo, MD Therapeutics, Meda Pharmaceuticals, Merck, Noven, NovoNordisk, Novogyne, Pfizer, Shionogi, Shippan Point Advisors LLC, Sprout Pharmaceuticals, Teva, Warner Chilcott, and Watson. He also reports receiving (currently or in the past year) grant/research support from NovoNordisk, Novogyne, Palatin Technologies, Teva, and Warner Chilcott. He reports serving on the speakers bureaus of Noven, NovoNordisk, Teva, and Warner Chilcott. Dr. Simon was the Chief Medical Officer for Sprout Pharmaceuticals until April 1, 2013.

Issue
OBG Management - 26(2)
Publications
Topics
Page Number
59-65
Legacy Keywords
Andrew Kaunitz,James Simon,JoAnn Pinkerton,menopause,hormone therapy,menopausal symptoms,hot flushes,HT,transdermal estradiol,oral micronized progesterone,North American Menopause Society,NAMS,optimal duration of HT,vasomotor symptoms,estrogen-only,estrogen-progestin,Women's Health Initiative,WHI,VMS,osteoporosis,hysterectomy,bone mineral density,BMD,
Sections
Author and Disclosure Information

Andrew M. Kaunitz, MD

Dr. JoAnn V. Pinkerton and Dr. James A. Simon provided peer review and comments for Dr. Kaunitz's case study.

CASES IN MENOPAUSE
Brought to you by the menopause experts

Andrew M. Kaunitz, MD
Professor and Associate Chairman, Department of Obstetrics and Gynecology, University of Florida College of Medicine–Jacksonville. Dr. Kaunitz is a NAMS Board member and certified menopause practitioner. He also serves on the OBG Management Board of Editors.

JoAnn V. Pinkerton, MD
Professor, Department of Obstetrics and Gynecology, and Director, Division of Midlife Women’s Health, University of Virginia. Dr. Pinkerton is a North American Menopause Society (NAMS) past president and certified menopause practitioner. She also serves on the OBG Management Board of Editors.

James A. Simon, MD
Clinical Professor, Department of Obstetrics and Gynecology, George Washington University, and Medical Director, Women’s Health & Research Consultants, Washington, DC. Dr. Simon is a NAMS past president, a certified menopause practitioner, and a certified clinical densitometrist. He also serves on the OBG Management Board of Editors.

Disclosures
Dr. Kaunitz reports that his institution receives grant or research support from Bayer, Endoceutics, Noven, and Teva, and that he is a consultant to Actavis, Bayer, DepoMed, and Teva.

Dr. Pinkerton reports that her institution receives consulting fees from DepoMed, Noven, NovoNordisk, Pfizer, and Shionogi; grant or research support from DepoMed, Bionova, and Endoceutics; and travel funds from DepoMed, Noven, NovoNordisk, Pfizer, and Shionogi.

Dr. Simon reports being a consultant to or on the advisory boards of Abbott Laboratories, Amgen, Ascend Therapeutics, DepoMed, Lelo, MD Therapeutics, Meda Pharmaceuticals, Merck, Noven, NovoNordisk, Novogyne, Pfizer, Shionogi, Shippan Point Advisors LLC, Sprout Pharmaceuticals, Teva, Warner Chilcott, and Watson. He also reports receiving (currently or in the past year) grant/research support from NovoNordisk, Novogyne, Palatin Technologies, Teva, and Warner Chilcott. He reports serving on the speakers bureaus of Noven, NovoNordisk, Teva, and Warner Chilcott. Dr. Simon was the Chief Medical Officer for Sprout Pharmaceuticals until April 1, 2013.

Author and Disclosure Information

Andrew M. Kaunitz, MD

Dr. JoAnn V. Pinkerton and Dr. James A. Simon provided peer review and comments for Dr. Kaunitz's case study.

CASES IN MENOPAUSE
Brought to you by the menopause experts

Andrew M. Kaunitz, MD
Professor and Associate Chairman, Department of Obstetrics and Gynecology, University of Florida College of Medicine–Jacksonville. Dr. Kaunitz is a NAMS Board member and certified menopause practitioner. He also serves on the OBG Management Board of Editors.

JoAnn V. Pinkerton, MD
Professor, Department of Obstetrics and Gynecology, and Director, Division of Midlife Women’s Health, University of Virginia. Dr. Pinkerton is a North American Menopause Society (NAMS) past president and certified menopause practitioner. She also serves on the OBG Management Board of Editors.

James A. Simon, MD
Clinical Professor, Department of Obstetrics and Gynecology, George Washington University, and Medical Director, Women’s Health & Research Consultants, Washington, DC. Dr. Simon is a NAMS past president, a certified menopause practitioner, and a certified clinical densitometrist. He also serves on the OBG Management Board of Editors.

Disclosures
Dr. Kaunitz reports that his institution receives grant or research support from Bayer, Endoceutics, Noven, and Teva, and that he is a consultant to Actavis, Bayer, DepoMed, and Teva.

Dr. Pinkerton reports that her institution receives consulting fees from DepoMed, Noven, NovoNordisk, Pfizer, and Shionogi; grant or research support from DepoMed, Bionova, and Endoceutics; and travel funds from DepoMed, Noven, NovoNordisk, Pfizer, and Shionogi.

Dr. Simon reports being a consultant to or on the advisory boards of Abbott Laboratories, Amgen, Ascend Therapeutics, DepoMed, Lelo, MD Therapeutics, Meda Pharmaceuticals, Merck, Noven, NovoNordisk, Novogyne, Pfizer, Shionogi, Shippan Point Advisors LLC, Sprout Pharmaceuticals, Teva, Warner Chilcott, and Watson. He also reports receiving (currently or in the past year) grant/research support from NovoNordisk, Novogyne, Palatin Technologies, Teva, and Warner Chilcott. He reports serving on the speakers bureaus of Noven, NovoNordisk, Teva, and Warner Chilcott. Dr. Simon was the Chief Medical Officer for Sprout Pharmaceuticals until April 1, 2013.

Article PDF
Article PDF
Related Articles

CASE: AFTER 3 YEARS OF HT, A PATIENT ASKS WHETHER IT’S TIME TO QUIT

My menopausal patient is a 57-year-old woman with a body mass index (BMI) of 21 kg/m2. Her mother, who also was slender, suffered a hip fracture at age 74.

When this patient was 53, approximately 8 months after her last menstrual period, she scheduled a problem visit to discuss bothersome hot flushes, which occurred primarily at night. These symptoms were associated with sleep disruption and irritability. At that problem visit, the patient and I discussed the benefits and risks of menopausal hormone therapy (HT), and she elected to initiate it, choosing transdermal estradiol using an 0.05-mg patch, combined with oral micronized progesterone 100-mg (one capsule) at bedtime. Two months later, she telephoned my office to report that she was experiencing only moderate relief of her symptoms. I increased the dose of estradiol to 0.075 mg. On her next well-woman visit, the patient remarked that her symptoms were largely resolved and said that she wished to continue the regimen.

Now, as she presents for her well-woman visit 3 years later, she asks how long she should continue the HT.

How would you counsel such a patient?

Although the duration of HT is still marked by controversy, clinicians often encounter the issue in practice. As the North American Menopause Society (NAMS) notes in a recent Practice Pearl and in its 2012 Position Statement on hormone therapy, the determination of the optimal duration of HT can be a challenge for clinicians and patients.1,2

In this article, I discuss indications for HT and consider variables that may influence its duration. I also offer practical guidance on therapeutic options for women who elect to use HT for an extended duration.

HOT FLUSHES CAN BE A LONG-TERM CONCERN
Moderate to severe vasomotor symptoms (VMS) are the most common indication for systemic combination estrogen-progestin or estrogen-only HT—and HT is the most effective treatment for VMS.2

Some experts have cautioned that “it remains prudent to keep the…duration of treatment short” or that HT “may serve a useful role in short-term symptom management.”3,4 However, for many menopausal women, VMS are a long-term concern. The Penn Ovarian Aging Study was conducted to estimate the duration of moderate-to-severe VMS and found a median duration of such symptoms of more than 10 years. In this landmark cohort study, the median duration of VMS, which began near the time of the menopausal transition, was almost 12 years.5

In a study of older menopausal women (mean age, 67 years; mean time since menopause, 19 years), 11.8% reported “clinically significant” hot flushes and “more than half of these women who complained of significant hot flushes at baseline continued to report bothersome symptoms after 3 years.”6

These observations underscore the fact that, in many women, short-term use (3–5 years) of HT will not be sufficient to control bothersome VMS.

SYSTEMIC HT ALSO BENEFITS BONE
The standard daily dose of HT (eg, conjugated equine estrogens [CEE], 0.625 mg; micronized estradiol, 1.0 mg; or transdermal estradiol, 0.05 mg) for relief of VMS also prevents osteoporosis,2 with many HT formulations approved for prevention of this condition. Randomized trial data from the Women’s Health Initiative (WHI) also have confirmed that a standard dose of HT prevents fractures in menopausal women.2

However, in menopausal women, doses of estrogen therapy substantially lower than those commonly used to treat VMS can still maintain or improve bone mineral density (BMD). For example, serum estradiol levels remain in the menopausal range during use of the weekly estradiol ultra-low-dose patch (0.014 mg).7 In a clinical trial of women (mean age, 66 years) with an intact uterus, use of this ultra-low-dose estradiol patch for 2 years without progestin did not increase the risk of endometrial hyperplasia7—although this patch does appear to increase the incidence of endometrial proliferation.8 For this reason, periodic endometrial monitoring may be appropriate in women using the 0.014-mg estradiol patch over the long term, including vaginal ultrasound assessment of endometrial thickness. Package labeling for this patch recommends that women with an intact uterus be given a progestogen for 14 days every 6 to 12 months.9

Related Article: Update on Osteoporosis Steven R. Goldstein, MD (December 2013)

Although the ultra-low-dose estradiol patch is approved for the prevention of osteoporosis, its efficacy in treating VMS is uncertain. For instance, in a study of this patch in women aged 60 to 80 years, with skeletal health and safety as the primary outcomes, 16% of participants reported VMS at baseline. The 0.014-mg estradiol patch did not reduce VMS more than placebo.10 However, in a trial of the 0.014-mg estradiol patch in which impact on VMS was the primary outcome, the ultra-low-dose patch did relieve VMS.11 (The ultra-low-dose patch currently is not approved to treat VMS.) Low-dose CEE (0.3 mg, 0.45 mg) and low-dose oral estradiol (0.5 mg) have been found to be effective in the treatment of VMS.12,13

 

 

Data on the risk of osteoporotic fractures among women using the ultra-low-dose estradiol patch are not available.

Use of HT to prevent osteoporosis is appropriate for women who have other indications for HT, such as VMS. For women using HT who no longer experience VMS, long-term use of HT for osteoporosis is controversial. However, it may be considered for women at elevated risk for osteoporosis when skeleton-specific treatments (eg, bisphosphonates) are not tolerated or when such women prefer not to use skeleton-specific therapy.

FDA package labeling for systemic HT indicates that, “When prescribing solely for the prevention of postmenopausal osteoporosis, therapy only should be considered for women at significant risk of osteoporosis, and non-estrogen medications should be carefully considered.”14

The NAMS 2012 Position Statement on HT states: “Provided that the woman is well aware of the potential benefits and risks and has clinical supervision, extending [estrogen-progestin therapy] use with the lowest effective dose is acceptable under some circumstances, including 1) for the woman who has determined that the benefits of menopause symptom relief outweigh risks, notably after failing an attempt to stop [estrogen-progestin therapy], and 2) for the woman at high risk of fracture for whom alternate therapies are not appropriate or cause unacceptable adverse effects.”2

A 2014 Practice Bulletin from the American College of Obstetricians and Gynecologists (ACOG) on the management of menopausal symptoms states: “The decision to continue HT should be individualized and be based on a woman’s symptoms and the risk–benefit ratio, regardless of age. Because some women aged 65 years and older may continue to need systemic HT for the management of vasomotor symptoms, ACOG recommends against routine discontinuation of systemic estrogen at age 65. As with younger women, use of HT and estrogen therapy should be individualized based on each woman’s risk–benefit ratio and clinical presentation.”15

As I have detailed, doses of HT that are lower than those used to treat VMS can prevent loss of BMD. Accordingly, clinicians prescribing HT for the sole indication of osteoporosis prevention should use doses lower than those for standard HT. Moreover, clinicians prescribing HT specifically to prevent osteoporosis should recognize the elevated risk of breast cancer with estrogen-progestin therapy. Extended use of estrogen-only therapy is more appropriate for this indication.

Related Article: Your menopausal patient's breast biopsy reveals atypical hyperplasia JoAnn V. Pinkerton, MD (Cases in Menopause, May 2013)

While estrogen-only therapy is common in women following hysterectomy, ultra-low-dose estrogen therapy with regular endometrial monitoring also can be considered in women with an intact uterus.

Also be aware that BMD declines rapidly after discontinuation of HT (in contrast with bisphosphonates), so alternative agents to maintain BMD should be considered when HT is stopped.16

HOW SAFE IS EXTENDED USE OF SYSTEMIC HT?
The incidence of breast cancer and mortality from breast cancer increase after 3 to 5 years of estrogen-progestin therapy, and the risk of stroke remains elevated throughout use of combination as well as estrogen-only HT.2 Women with an intact uterus who choose to extend the duration of combination therapy beyond 5 years for control of VMS or protection against osteoporosis, or both, need to be candidly counseled about these concerns. No increase in the risk of breast cancer was observed in the estrogen-only arm of the WHI randomized, clinical trial (mean duration of CEE therapy of 7.1 years).17

James A. Simon, MD:
Not only was there no increase in the risk of breast cancer in the estrogen-only arm, but the therapy was associated with a decrease in risk that persisted even following discontinuation of the therapy.

 JoAnn V. Pinkerton, MD:
Yes, after 7 years of follow-up, women taking CEE (0.625 mg daily) had a reduction in the risk of breast cancer that translated into a decrease in mortality.17

Related Article: Stop enforcing a 5-year rule for menopausal hormone therapy Robert L. Reid, MD (Stop/Start, December 2013)

Long-term risks of oral estrogen
Among women who initiate HT at the time of menopause, long-term use does not appear to increase the risk of coronary heart disease (CHD), although follow-up in clinical trials has not extended beyond 7 years for estrogen-progestin therapy, and midlife may bring increases in a woman’s baseline cardiovascular risk.2 However, in the WHI, women who initiated oral estrogen-only or estrogen-progestin therapy later in menopause experienced an increased risk of CHD,18 underscoring the need for caution and individualization in this patient population.

Oral HT increases the risk of venous thromboembolism (VTE) and stroke.2 In addition, age is an independent risk factor for these two outcomes. Observational studies suggest that, in contrast with oral estrogen, transdermal HT does not increase the risk of VTE.19–24 Randomized trial data are lacking.

 

 

Similarly, one observational study suggests that low-dose (≤0.05 mg) transdermal estradiol does not appear to increase the risk of stroke,25 but, again, clinical trial data are unavailable.

Given these apparent safety advantages, transdermal estrogen therapy would appear to be preferable to oral estrogen in older, long-term users, a perspective supported by ACOG.26

Related Article: Is one oral estrogen formulation safer than another for menopausal women? Andrew M. Kaunitz, MD (Examining the Evidence, January 2014)

In regard to VTE, oral estradiol appears to be safer than CEE.27 Accordingly, oral estradiol may be preferable for older long-term users who don’t tolerate transdermal estradiol due to local skin reactions or costs. Oral estradiol also is less expensive than CEE.

What to expect when your patient discontinues systemic HT
VMS may recur is as many as 50% of women after they discontinue HT. The likelihood of recurring VMS does not appear to vary between abrupt and tapered discontinuation.2

Some HT users may be reluctant to reduce their dose or discontinue HT, particularly those who experienced severe VMS originally. In my clinical experience, many of these women are receptive to a trial of lower-dose HT, especially when I advise them that they can resume their original (higher) dose should bothersome VMS recur.

JoAnn V. Pinkerton, MD:
I use a similar approach with my patients, advising them to try 3 months off HT with the understanding that they can resume the therapy if they develop bothersome symptoms.

Individualized assessment of HT benefits and risks and shared decision-making play important roles in the management of these patients. As the dose of HT declines, or systemic HT is discontinued, symptoms of genital atrophy may become more prominent and, in the absence of indications for systemic HT (bothersome VMS or prevention of osteoporosis), may best be addressed with vaginal estrogen therapy or ospemifene.

Extended use of vaginal estrogen
Unlike VMS, untreated genital atrophy may continue to progress as women age, ­sometimes necessitating use of vaginal estrogen. Because the clinical trials that served as the basis for FDA approval of vaginal estrogen formulations did not find an elevated risk of endometrial hyperplasia, routine use of a progestin to prevent endometrial proliferation in women with an intact uterus is not recommended.28 However, these trials were too limited in duration to assure long-term endometrial safety. All postmenopausal women using vaginal ET should be advised to report any vaginal bleeding, and that bleeding should be evaluated appropriately.

JoAnn V. Pinkerton, MD:
I recommend transvaginal ultrasound and endometrial biopsy for women using vaginal estrogen who report spotting or bleeding.

Although low-dose local or vaginal estrogen therapy has not been studied in clinical trials beyond 1 year, it is thought to carry significantly fewer risks than systemic HT.28 Several studies have confirmed that serum estrogen levels remain in the postmenopausal range in women using low-dose vaginal estrogen, specifically the 3-month estradiol ring (2 mg) or twice-weekly estradiol tablets (10 µg).28

Besides relieving vaginal dryness and dyspareunia, low-dose vaginal estrogen also may improve overactive bladder and reduce the incidence of recurrent urinary tract infection.29,30

CASE: RESOLVED
The 57-year-old patient has been essentially symptom-free for the past 3 years using an estradiol patch (0.075 mg) with progesterone (100 mg nightly). Now she asks how long she should continue HT, and I explain that the duration of bothersome VMS is different in each woman. I also counsel her that hot flushes do resolve over time in almost all women. When she asks how likely it is that bothersome VMS will recur if she simply stops HT, I explain that bothersome symptoms often last 10 years or longer, and I remind her that her VMS began some 4 years earlier.

I also briefly review HT benefits (treatment of VMS as well as prevention of vulvovaginal atrophy and osteoporosis) and risks (small increased risk of breast cancer and stroke). I suggest a reduction in her HT dose as a reasonable method to determine her ongoing need for HT, telling her that she should know within about 1 month how she feels on the lower dose (0.05-mg patch). I also advise her to call my office if bothersome VMS recur on the lower dose so that I can increase the dose back to its original level.

After her estradiol dose is reduced, the patient reports only minimal VMS, and she opts to continue the estradiol patch (0.05 mg) with nightly progesterone (100 mg) for another 2 years.

At age 59, during her well-woman visit, she decides to lower the estradiol further, transitioning to a 0.0375-mg patch but maintaining the nightly progesterone (100 mg). She reports no VMS on this new regimen.

 

 

At age 61, because of her maternal history of osteoporosis and her own low BMI, the patient undergoes BMD assessment with dual x-ray absorptiometry (DXA). In average-risk women, NAMS recommends that BMD assessment be performed at age 65.31 The results of BMD assessment are normal.

After further discussion, the patient agrees to an even lower dose of estradiol, switching to an 0.025-mg patch, with progesterone (100 mg nightly) administered for 2 weeks in every 3-month interval. She reports no VMS or vaginal bleeding on this lower-dose HT regimen.

After 12 months on this new regimen, the patient undergoes vaginal sonography, revealing an endometrial thickness of 3 mm. She continues this regimen, including annual vaginal ultrasound assessment of the endometrium, without problems until her well-woman visit at age 65.

At that visit, I explain that discontinuation of HT is unlikely to trigger recurred VMS but may cause her to lose BMD rapidly for several years, and may also result in unpleasant symptoms from vulvovaginal atrophy including sexual discomfort. She decides to switch to an 0.014-mg estradiol patch without progesterone, and to undergo ultrasound assessment of her endometrium every 1 to 2 years.

Do you have a troubling case in menopause?
Suggest it to our expert panel. They may address your management dilemma in a future issue!
Email us at obg@frontlinemedcom.com

BOTTOM LINE: INDIVIDUALIZE THE DURATION OF HT
Although published data on extended use of HT are few, many clinicians caring for menopausal women are asked to make a recommendation. Because extended use of estrogen-progestin HT increases the risk of breast cancer, estrogen-only HT has a more favorable benefit-risk ratio. If a patient uses estrogen-progestin HT for an extended duration, periodic discussions about the elevated risk of breast cancer are appropriate.

JoAnn V. Pinkerton, MD:
The risk of breast cancer associated with extended use of estrogen-progestin HT likely is reduced if lower doses are given. Overall, however, the risk appears to be both dose- and duration-dependent.

We lack randomized trial data on CHD and other risks in women who begin HT at the time of menopause and continue it for decades. In older women who use HT for an extended duration, transdermal estrogen may be safer in regard to the risk of VTE and stroke.

As the systemic estrogen dose is lowered, it is possible to reduce the dose of the progestin (the sole function of which is to protect the endometrium from estrogen stimulation). Intermittent dosing can be used, although we lack long-term safety data, and periodic endometrial evaluation should be considered.

Remember also that, with intermittent or daily dosing of a progestin, you are relying on the patient to take this medication to protect the endometrium.

Extended use of low-dose vaginal estrogen HT may be necessary to treat symptoms of vulvovaginal atrophy, which tend to worsen over time. Administration of a progestin is not currently recommended with use of vaginal estrogen, but long-term use may increase the risk of endometrial stimulation.

Read other CASES IN MENOPAUSE
Your postmenopausal patient reports a history of migraine
James A. Simon, MD (October 2013)
Your menopausal patient's breast biopsy reveals atypical hyperplasia
JoAnn V. Pinkerton, MD (May 2013)

WE WANT TO HEAR FROM YOU!
Drop us a line and let us know what you think about current articles, which topics you'd like to see covered in future issues, and what challenges you face in daily practice. Tell us what you think by emailing us at: obg@frontlinemedcom.com

CASE: AFTER 3 YEARS OF HT, A PATIENT ASKS WHETHER IT’S TIME TO QUIT

My menopausal patient is a 57-year-old woman with a body mass index (BMI) of 21 kg/m2. Her mother, who also was slender, suffered a hip fracture at age 74.

When this patient was 53, approximately 8 months after her last menstrual period, she scheduled a problem visit to discuss bothersome hot flushes, which occurred primarily at night. These symptoms were associated with sleep disruption and irritability. At that problem visit, the patient and I discussed the benefits and risks of menopausal hormone therapy (HT), and she elected to initiate it, choosing transdermal estradiol using an 0.05-mg patch, combined with oral micronized progesterone 100-mg (one capsule) at bedtime. Two months later, she telephoned my office to report that she was experiencing only moderate relief of her symptoms. I increased the dose of estradiol to 0.075 mg. On her next well-woman visit, the patient remarked that her symptoms were largely resolved and said that she wished to continue the regimen.

Now, as she presents for her well-woman visit 3 years later, she asks how long she should continue the HT.

How would you counsel such a patient?

Although the duration of HT is still marked by controversy, clinicians often encounter the issue in practice. As the North American Menopause Society (NAMS) notes in a recent Practice Pearl and in its 2012 Position Statement on hormone therapy, the determination of the optimal duration of HT can be a challenge for clinicians and patients.1,2

In this article, I discuss indications for HT and consider variables that may influence its duration. I also offer practical guidance on therapeutic options for women who elect to use HT for an extended duration.

HOT FLUSHES CAN BE A LONG-TERM CONCERN
Moderate to severe vasomotor symptoms (VMS) are the most common indication for systemic combination estrogen-progestin or estrogen-only HT—and HT is the most effective treatment for VMS.2

Some experts have cautioned that “it remains prudent to keep the…duration of treatment short” or that HT “may serve a useful role in short-term symptom management.”3,4 However, for many menopausal women, VMS are a long-term concern. The Penn Ovarian Aging Study was conducted to estimate the duration of moderate-to-severe VMS and found a median duration of such symptoms of more than 10 years. In this landmark cohort study, the median duration of VMS, which began near the time of the menopausal transition, was almost 12 years.5

In a study of older menopausal women (mean age, 67 years; mean time since menopause, 19 years), 11.8% reported “clinically significant” hot flushes and “more than half of these women who complained of significant hot flushes at baseline continued to report bothersome symptoms after 3 years.”6

These observations underscore the fact that, in many women, short-term use (3–5 years) of HT will not be sufficient to control bothersome VMS.

SYSTEMIC HT ALSO BENEFITS BONE
The standard daily dose of HT (eg, conjugated equine estrogens [CEE], 0.625 mg; micronized estradiol, 1.0 mg; or transdermal estradiol, 0.05 mg) for relief of VMS also prevents osteoporosis,2 with many HT formulations approved for prevention of this condition. Randomized trial data from the Women’s Health Initiative (WHI) also have confirmed that a standard dose of HT prevents fractures in menopausal women.2

However, in menopausal women, doses of estrogen therapy substantially lower than those commonly used to treat VMS can still maintain or improve bone mineral density (BMD). For example, serum estradiol levels remain in the menopausal range during use of the weekly estradiol ultra-low-dose patch (0.014 mg).7 In a clinical trial of women (mean age, 66 years) with an intact uterus, use of this ultra-low-dose estradiol patch for 2 years without progestin did not increase the risk of endometrial hyperplasia7—although this patch does appear to increase the incidence of endometrial proliferation.8 For this reason, periodic endometrial monitoring may be appropriate in women using the 0.014-mg estradiol patch over the long term, including vaginal ultrasound assessment of endometrial thickness. Package labeling for this patch recommends that women with an intact uterus be given a progestogen for 14 days every 6 to 12 months.9

Related Article: Update on Osteoporosis Steven R. Goldstein, MD (December 2013)

Although the ultra-low-dose estradiol patch is approved for the prevention of osteoporosis, its efficacy in treating VMS is uncertain. For instance, in a study of this patch in women aged 60 to 80 years, with skeletal health and safety as the primary outcomes, 16% of participants reported VMS at baseline. The 0.014-mg estradiol patch did not reduce VMS more than placebo.10 However, in a trial of the 0.014-mg estradiol patch in which impact on VMS was the primary outcome, the ultra-low-dose patch did relieve VMS.11 (The ultra-low-dose patch currently is not approved to treat VMS.) Low-dose CEE (0.3 mg, 0.45 mg) and low-dose oral estradiol (0.5 mg) have been found to be effective in the treatment of VMS.12,13

 

 

Data on the risk of osteoporotic fractures among women using the ultra-low-dose estradiol patch are not available.

Use of HT to prevent osteoporosis is appropriate for women who have other indications for HT, such as VMS. For women using HT who no longer experience VMS, long-term use of HT for osteoporosis is controversial. However, it may be considered for women at elevated risk for osteoporosis when skeleton-specific treatments (eg, bisphosphonates) are not tolerated or when such women prefer not to use skeleton-specific therapy.

FDA package labeling for systemic HT indicates that, “When prescribing solely for the prevention of postmenopausal osteoporosis, therapy only should be considered for women at significant risk of osteoporosis, and non-estrogen medications should be carefully considered.”14

The NAMS 2012 Position Statement on HT states: “Provided that the woman is well aware of the potential benefits and risks and has clinical supervision, extending [estrogen-progestin therapy] use with the lowest effective dose is acceptable under some circumstances, including 1) for the woman who has determined that the benefits of menopause symptom relief outweigh risks, notably after failing an attempt to stop [estrogen-progestin therapy], and 2) for the woman at high risk of fracture for whom alternate therapies are not appropriate or cause unacceptable adverse effects.”2

A 2014 Practice Bulletin from the American College of Obstetricians and Gynecologists (ACOG) on the management of menopausal symptoms states: “The decision to continue HT should be individualized and be based on a woman’s symptoms and the risk–benefit ratio, regardless of age. Because some women aged 65 years and older may continue to need systemic HT for the management of vasomotor symptoms, ACOG recommends against routine discontinuation of systemic estrogen at age 65. As with younger women, use of HT and estrogen therapy should be individualized based on each woman’s risk–benefit ratio and clinical presentation.”15

As I have detailed, doses of HT that are lower than those used to treat VMS can prevent loss of BMD. Accordingly, clinicians prescribing HT for the sole indication of osteoporosis prevention should use doses lower than those for standard HT. Moreover, clinicians prescribing HT specifically to prevent osteoporosis should recognize the elevated risk of breast cancer with estrogen-progestin therapy. Extended use of estrogen-only therapy is more appropriate for this indication.

Related Article: Your menopausal patient's breast biopsy reveals atypical hyperplasia JoAnn V. Pinkerton, MD (Cases in Menopause, May 2013)

While estrogen-only therapy is common in women following hysterectomy, ultra-low-dose estrogen therapy with regular endometrial monitoring also can be considered in women with an intact uterus.

Also be aware that BMD declines rapidly after discontinuation of HT (in contrast with bisphosphonates), so alternative agents to maintain BMD should be considered when HT is stopped.16

HOW SAFE IS EXTENDED USE OF SYSTEMIC HT?
The incidence of breast cancer and mortality from breast cancer increase after 3 to 5 years of estrogen-progestin therapy, and the risk of stroke remains elevated throughout use of combination as well as estrogen-only HT.2 Women with an intact uterus who choose to extend the duration of combination therapy beyond 5 years for control of VMS or protection against osteoporosis, or both, need to be candidly counseled about these concerns. No increase in the risk of breast cancer was observed in the estrogen-only arm of the WHI randomized, clinical trial (mean duration of CEE therapy of 7.1 years).17

James A. Simon, MD:
Not only was there no increase in the risk of breast cancer in the estrogen-only arm, but the therapy was associated with a decrease in risk that persisted even following discontinuation of the therapy.

 JoAnn V. Pinkerton, MD:
Yes, after 7 years of follow-up, women taking CEE (0.625 mg daily) had a reduction in the risk of breast cancer that translated into a decrease in mortality.17

Related Article: Stop enforcing a 5-year rule for menopausal hormone therapy Robert L. Reid, MD (Stop/Start, December 2013)

Long-term risks of oral estrogen
Among women who initiate HT at the time of menopause, long-term use does not appear to increase the risk of coronary heart disease (CHD), although follow-up in clinical trials has not extended beyond 7 years for estrogen-progestin therapy, and midlife may bring increases in a woman’s baseline cardiovascular risk.2 However, in the WHI, women who initiated oral estrogen-only or estrogen-progestin therapy later in menopause experienced an increased risk of CHD,18 underscoring the need for caution and individualization in this patient population.

Oral HT increases the risk of venous thromboembolism (VTE) and stroke.2 In addition, age is an independent risk factor for these two outcomes. Observational studies suggest that, in contrast with oral estrogen, transdermal HT does not increase the risk of VTE.19–24 Randomized trial data are lacking.

 

 

Similarly, one observational study suggests that low-dose (≤0.05 mg) transdermal estradiol does not appear to increase the risk of stroke,25 but, again, clinical trial data are unavailable.

Given these apparent safety advantages, transdermal estrogen therapy would appear to be preferable to oral estrogen in older, long-term users, a perspective supported by ACOG.26

Related Article: Is one oral estrogen formulation safer than another for menopausal women? Andrew M. Kaunitz, MD (Examining the Evidence, January 2014)

In regard to VTE, oral estradiol appears to be safer than CEE.27 Accordingly, oral estradiol may be preferable for older long-term users who don’t tolerate transdermal estradiol due to local skin reactions or costs. Oral estradiol also is less expensive than CEE.

What to expect when your patient discontinues systemic HT
VMS may recur is as many as 50% of women after they discontinue HT. The likelihood of recurring VMS does not appear to vary between abrupt and tapered discontinuation.2

Some HT users may be reluctant to reduce their dose or discontinue HT, particularly those who experienced severe VMS originally. In my clinical experience, many of these women are receptive to a trial of lower-dose HT, especially when I advise them that they can resume their original (higher) dose should bothersome VMS recur.

JoAnn V. Pinkerton, MD:
I use a similar approach with my patients, advising them to try 3 months off HT with the understanding that they can resume the therapy if they develop bothersome symptoms.

Individualized assessment of HT benefits and risks and shared decision-making play important roles in the management of these patients. As the dose of HT declines, or systemic HT is discontinued, symptoms of genital atrophy may become more prominent and, in the absence of indications for systemic HT (bothersome VMS or prevention of osteoporosis), may best be addressed with vaginal estrogen therapy or ospemifene.

Extended use of vaginal estrogen
Unlike VMS, untreated genital atrophy may continue to progress as women age, ­sometimes necessitating use of vaginal estrogen. Because the clinical trials that served as the basis for FDA approval of vaginal estrogen formulations did not find an elevated risk of endometrial hyperplasia, routine use of a progestin to prevent endometrial proliferation in women with an intact uterus is not recommended.28 However, these trials were too limited in duration to assure long-term endometrial safety. All postmenopausal women using vaginal ET should be advised to report any vaginal bleeding, and that bleeding should be evaluated appropriately.

JoAnn V. Pinkerton, MD:
I recommend transvaginal ultrasound and endometrial biopsy for women using vaginal estrogen who report spotting or bleeding.

Although low-dose local or vaginal estrogen therapy has not been studied in clinical trials beyond 1 year, it is thought to carry significantly fewer risks than systemic HT.28 Several studies have confirmed that serum estrogen levels remain in the postmenopausal range in women using low-dose vaginal estrogen, specifically the 3-month estradiol ring (2 mg) or twice-weekly estradiol tablets (10 µg).28

Besides relieving vaginal dryness and dyspareunia, low-dose vaginal estrogen also may improve overactive bladder and reduce the incidence of recurrent urinary tract infection.29,30

CASE: RESOLVED
The 57-year-old patient has been essentially symptom-free for the past 3 years using an estradiol patch (0.075 mg) with progesterone (100 mg nightly). Now she asks how long she should continue HT, and I explain that the duration of bothersome VMS is different in each woman. I also counsel her that hot flushes do resolve over time in almost all women. When she asks how likely it is that bothersome VMS will recur if she simply stops HT, I explain that bothersome symptoms often last 10 years or longer, and I remind her that her VMS began some 4 years earlier.

I also briefly review HT benefits (treatment of VMS as well as prevention of vulvovaginal atrophy and osteoporosis) and risks (small increased risk of breast cancer and stroke). I suggest a reduction in her HT dose as a reasonable method to determine her ongoing need for HT, telling her that she should know within about 1 month how she feels on the lower dose (0.05-mg patch). I also advise her to call my office if bothersome VMS recur on the lower dose so that I can increase the dose back to its original level.

After her estradiol dose is reduced, the patient reports only minimal VMS, and she opts to continue the estradiol patch (0.05 mg) with nightly progesterone (100 mg) for another 2 years.

At age 59, during her well-woman visit, she decides to lower the estradiol further, transitioning to a 0.0375-mg patch but maintaining the nightly progesterone (100 mg). She reports no VMS on this new regimen.

 

 

At age 61, because of her maternal history of osteoporosis and her own low BMI, the patient undergoes BMD assessment with dual x-ray absorptiometry (DXA). In average-risk women, NAMS recommends that BMD assessment be performed at age 65.31 The results of BMD assessment are normal.

After further discussion, the patient agrees to an even lower dose of estradiol, switching to an 0.025-mg patch, with progesterone (100 mg nightly) administered for 2 weeks in every 3-month interval. She reports no VMS or vaginal bleeding on this lower-dose HT regimen.

After 12 months on this new regimen, the patient undergoes vaginal sonography, revealing an endometrial thickness of 3 mm. She continues this regimen, including annual vaginal ultrasound assessment of the endometrium, without problems until her well-woman visit at age 65.

At that visit, I explain that discontinuation of HT is unlikely to trigger recurred VMS but may cause her to lose BMD rapidly for several years, and may also result in unpleasant symptoms from vulvovaginal atrophy including sexual discomfort. She decides to switch to an 0.014-mg estradiol patch without progesterone, and to undergo ultrasound assessment of her endometrium every 1 to 2 years.

Do you have a troubling case in menopause?
Suggest it to our expert panel. They may address your management dilemma in a future issue!
Email us at obg@frontlinemedcom.com

BOTTOM LINE: INDIVIDUALIZE THE DURATION OF HT
Although published data on extended use of HT are few, many clinicians caring for menopausal women are asked to make a recommendation. Because extended use of estrogen-progestin HT increases the risk of breast cancer, estrogen-only HT has a more favorable benefit-risk ratio. If a patient uses estrogen-progestin HT for an extended duration, periodic discussions about the elevated risk of breast cancer are appropriate.

JoAnn V. Pinkerton, MD:
The risk of breast cancer associated with extended use of estrogen-progestin HT likely is reduced if lower doses are given. Overall, however, the risk appears to be both dose- and duration-dependent.

We lack randomized trial data on CHD and other risks in women who begin HT at the time of menopause and continue it for decades. In older women who use HT for an extended duration, transdermal estrogen may be safer in regard to the risk of VTE and stroke.

As the systemic estrogen dose is lowered, it is possible to reduce the dose of the progestin (the sole function of which is to protect the endometrium from estrogen stimulation). Intermittent dosing can be used, although we lack long-term safety data, and periodic endometrial evaluation should be considered.

Remember also that, with intermittent or daily dosing of a progestin, you are relying on the patient to take this medication to protect the endometrium.

Extended use of low-dose vaginal estrogen HT may be necessary to treat symptoms of vulvovaginal atrophy, which tend to worsen over time. Administration of a progestin is not currently recommended with use of vaginal estrogen, but long-term use may increase the risk of endometrial stimulation.

Read other CASES IN MENOPAUSE
Your postmenopausal patient reports a history of migraine
James A. Simon, MD (October 2013)
Your menopausal patient's breast biopsy reveals atypical hyperplasia
JoAnn V. Pinkerton, MD (May 2013)

WE WANT TO HEAR FROM YOU!
Drop us a line and let us know what you think about current articles, which topics you'd like to see covered in future issues, and what challenges you face in daily practice. Tell us what you think by emailing us at: obg@frontlinemedcom.com

References

  1. Kaunitz AM. NAMS Practice Pearl: Extended duration use of menopausal hormone therapy [published online ahead of print January 6, 2014]. Menopause.
  2. North American Menopause Society. Position Statement: The 2012 hormone therapy position statement of the North American Menopause Society. Menopause. 2012;19(3):257–271.
  3. Nelson HD. Postmenopausal estrogen for treatment of hot flashes: clinical applications. JAMA. 2004;291(13):1621–1625.
  4. Hulley SB, Grady D. The WHI estrogen-alone trial—do things look any better? JAMA. 2004;291(14):1769–1771.
  5. Freeman EW, Sammel MD, Lin H, Liu Z, Gracia CR. Duration of menopausal hot flushes and associated risk factors. Obstet Gynecol. 2011;117(5):1095–1104.
  6. Huang AJ, Grady D, Jacoby VL, Blackwell TL, Bauer DC, Sawaya GF. Persistent hot flushes in older postmenopausal women. Arch Intern Med. 2008;168(8):840–846.
  7. Ettinger B, Ensrud KE, Wallace R, Johnson KC, Cummings SR, Yankov V, Vittinghoff  E, Grady D. Effects of ultralow-dose transdermal estradiol on bone mineral density: A randomized clinical trial. Obstet Gynecol. 2004;104(3):443–451.
  8. Johnson SR, Ettinger B, Macer JL, Ensrud KE, Quan J, Grady D. Uterine and vaginal effects of unopposed ultralow-dose transdermal estradiol. Obstet Gynecol. 2005;105(4):779–787.
  9. Menostar [package insert]. Wayne, NJ: Bayer; 2007.
  10. Diem S, Grady D, Quan J, Vittinghoff  E, Wallace R, Hanes V, Ensrud K. Effects of ultralow-dose transdermal estradiol on postmenopausal symptoms in women aged 60 to 80 years. Menopause. 2006;13(1):130–138.
  11. Bachmann GA, Schaefers M, Uddin A, Utian WH. Lowest effective transdermal 17beta-estradiol dose for relief of hot flushes in postmenopausal women: A randomized controlled trial. Obstet Gynecol. 2007;110(4):771–779.
  12. Honjo H, Taketani Y. Low-dose estradiol for climacteric symptoms in Japanese women: A randomized, controlled trial. Climacteric. 2009;12(4):319–328. 
  13. Utian WH, Shoupe D, Bachmann G, Pinkerton JV, Pickar JH. Relief of vasomotor symptoms and vaginal atrophy with lower doses of conjugated equine estrogens and medroxyprogesterone acetate. Fertil Steril. 2001;75(6):1065–1079.
  14. Premarin [package insert]. Philadelphia, PA: Pfizer; 2012.
  15. American College of Obstetricians and Gynecologists. Practice Bulletin No. 141: Management of menopausal symptoms. Obstet Gynecol. 2014;123(1):202–216.
  16. National Osteoporosis Foundation. Clinician’s Guide to Prevention and Treatment of Osteoporosis. Washington, DC: National Osteoporosis Foundation; 2013.
  17. Anderson GL, Chlebowski RT, Aragaki AK, et al. Conjugated equine oestrogen and breast cancer incidence and mortality in postmenopausal women with hysterectomy: Extended follow-up of the Women’s Health Initiative randomised placebo-controlled trial. Lancet Oncol. 2012;13(5):476–486.
  18. Rossouw JE, Prentice RL, Manson JE, et al. Postmenopausal hormone therapy and risk of cardiovascular disease by age and years since menopause. JAMA. 2007;4:297(13):1465–1477.
  19. Canonico M, Fournier A, Carcaillon L, et al. Postmenopausal hormone therapy and risk of idiopathic venous thromboembolism: results from the E3N cohort study. Arterioscler Thromb Vasc Biol. 2010;30(2):340–345.
  20. Sweetland S, Beral V, Balkwill A, et al; The Million Women Study Collaborators. Venous thromboembolism risk in relation to use of different types of postmenopausal hormone therapy in a large prospective study [published online ahead of print September 10, 2012]. J Thromb Haemost. 2012. doi:10.1111/j.1538-7836.2012.04919.x.
  21. Roach RE, Lijfering WM, Helmerhorst FM, Cannegieter SC, Rosendaal FR, van Hylckama Vlieg A. The risk of venous thrombosis in women over 50 years old using oral contraception or postmenopausal hormone therapy. J Thromb Haemost. 2013;11(1):124–131.
  22. Renoux C, Dell’Aniello S, Suissa S. Hormone replacement therapy and the risk of venous thromboembolism: A population-based study. J Thromb Haemost. 2010;8(5):979–986.
  23. Laliberté F, Dea K, Duh MS, Kahler KH, Rolli M, Lefebvre P. Does the route of administration for estrogen hormone therapy impact the risk of venous thromboembolism? Estradiol transdermal system versus oral estrogen-only hormone therapy. Menopause. 2011;18(10):1052–1059.
  24. Scarabin PY, Oger E, Plu-Bureau G; EStrogen and THromboEmbolism Risk Study Group. Differential association of oral and transdermal oestrogen-replacement therapy with venous thromboembolism risk. Lancet. 2003;362(9382):428–432.
  25. Renoux C, Dell’Aniello S, Garbe E, Suissa S. Transdermal and oral hormone replacement therapy and the risk of stroke: A nested case-control study. BMJ. 2010;340:c2519. doi:10.1136/bmj.c2519.
  26. American College of Obstetricians and Gynecologists. Committee Opinion No. 556: Postmenopausal estrogen therapy: Route of administration and risk of venous thromboembolism. Obstet Gynecol. 2013;121(4):887–890.
  27. Smith NL, Blondon M, Wiggins KL, Harrington LB, van Hylckama Vlieg A, Floyd JS. Lower risk of cardiovascular events in postmenopausal women taking oral estradiol compared with oral conjugated equine estrogens. JAMA Intern Med. 2014;14(1):25–31.
  28. North American Menopause Society. 2013 Position Statement: Management of symptomatic vulvovaginal atrophy. Menopause. 2013;20(9):888–902.
  29. Nelken RS, Ozel BZ, Leegant AR, Felix JC, Mishell DR. Randomized trial of estradiol vaginal ring versus oral oxybutynin for the treatment of overactive bladder. Menopause. 2011;18(9):962–966.
  30. Eriksen BC. A randomized, open, parallel-group study on the preventive effect of an estradiol-releasing vaginal ring (Estring) on recurrent urinary tract infections in postmenopausal women. Am J Obstet Gynecol. 1999;180(5):1072–1079.
  31. North American Menopause Society. 2010 Position Statement: Management of osteoporosis in postmenopausal women. Menopause. 2010;17(1):25–54.
References

  1. Kaunitz AM. NAMS Practice Pearl: Extended duration use of menopausal hormone therapy [published online ahead of print January 6, 2014]. Menopause.
  2. North American Menopause Society. Position Statement: The 2012 hormone therapy position statement of the North American Menopause Society. Menopause. 2012;19(3):257–271.
  3. Nelson HD. Postmenopausal estrogen for treatment of hot flashes: clinical applications. JAMA. 2004;291(13):1621–1625.
  4. Hulley SB, Grady D. The WHI estrogen-alone trial—do things look any better? JAMA. 2004;291(14):1769–1771.
  5. Freeman EW, Sammel MD, Lin H, Liu Z, Gracia CR. Duration of menopausal hot flushes and associated risk factors. Obstet Gynecol. 2011;117(5):1095–1104.
  6. Huang AJ, Grady D, Jacoby VL, Blackwell TL, Bauer DC, Sawaya GF. Persistent hot flushes in older postmenopausal women. Arch Intern Med. 2008;168(8):840–846.
  7. Ettinger B, Ensrud KE, Wallace R, Johnson KC, Cummings SR, Yankov V, Vittinghoff  E, Grady D. Effects of ultralow-dose transdermal estradiol on bone mineral density: A randomized clinical trial. Obstet Gynecol. 2004;104(3):443–451.
  8. Johnson SR, Ettinger B, Macer JL, Ensrud KE, Quan J, Grady D. Uterine and vaginal effects of unopposed ultralow-dose transdermal estradiol. Obstet Gynecol. 2005;105(4):779–787.
  9. Menostar [package insert]. Wayne, NJ: Bayer; 2007.
  10. Diem S, Grady D, Quan J, Vittinghoff  E, Wallace R, Hanes V, Ensrud K. Effects of ultralow-dose transdermal estradiol on postmenopausal symptoms in women aged 60 to 80 years. Menopause. 2006;13(1):130–138.
  11. Bachmann GA, Schaefers M, Uddin A, Utian WH. Lowest effective transdermal 17beta-estradiol dose for relief of hot flushes in postmenopausal women: A randomized controlled trial. Obstet Gynecol. 2007;110(4):771–779.
  12. Honjo H, Taketani Y. Low-dose estradiol for climacteric symptoms in Japanese women: A randomized, controlled trial. Climacteric. 2009;12(4):319–328. 
  13. Utian WH, Shoupe D, Bachmann G, Pinkerton JV, Pickar JH. Relief of vasomotor symptoms and vaginal atrophy with lower doses of conjugated equine estrogens and medroxyprogesterone acetate. Fertil Steril. 2001;75(6):1065–1079.
  14. Premarin [package insert]. Philadelphia, PA: Pfizer; 2012.
  15. American College of Obstetricians and Gynecologists. Practice Bulletin No. 141: Management of menopausal symptoms. Obstet Gynecol. 2014;123(1):202–216.
  16. National Osteoporosis Foundation. Clinician’s Guide to Prevention and Treatment of Osteoporosis. Washington, DC: National Osteoporosis Foundation; 2013.
  17. Anderson GL, Chlebowski RT, Aragaki AK, et al. Conjugated equine oestrogen and breast cancer incidence and mortality in postmenopausal women with hysterectomy: Extended follow-up of the Women’s Health Initiative randomised placebo-controlled trial. Lancet Oncol. 2012;13(5):476–486.
  18. Rossouw JE, Prentice RL, Manson JE, et al. Postmenopausal hormone therapy and risk of cardiovascular disease by age and years since menopause. JAMA. 2007;4:297(13):1465–1477.
  19. Canonico M, Fournier A, Carcaillon L, et al. Postmenopausal hormone therapy and risk of idiopathic venous thromboembolism: results from the E3N cohort study. Arterioscler Thromb Vasc Biol. 2010;30(2):340–345.
  20. Sweetland S, Beral V, Balkwill A, et al; The Million Women Study Collaborators. Venous thromboembolism risk in relation to use of different types of postmenopausal hormone therapy in a large prospective study [published online ahead of print September 10, 2012]. J Thromb Haemost. 2012. doi:10.1111/j.1538-7836.2012.04919.x.
  21. Roach RE, Lijfering WM, Helmerhorst FM, Cannegieter SC, Rosendaal FR, van Hylckama Vlieg A. The risk of venous thrombosis in women over 50 years old using oral contraception or postmenopausal hormone therapy. J Thromb Haemost. 2013;11(1):124–131.
  22. Renoux C, Dell’Aniello S, Suissa S. Hormone replacement therapy and the risk of venous thromboembolism: A population-based study. J Thromb Haemost. 2010;8(5):979–986.
  23. Laliberté F, Dea K, Duh MS, Kahler KH, Rolli M, Lefebvre P. Does the route of administration for estrogen hormone therapy impact the risk of venous thromboembolism? Estradiol transdermal system versus oral estrogen-only hormone therapy. Menopause. 2011;18(10):1052–1059.
  24. Scarabin PY, Oger E, Plu-Bureau G; EStrogen and THromboEmbolism Risk Study Group. Differential association of oral and transdermal oestrogen-replacement therapy with venous thromboembolism risk. Lancet. 2003;362(9382):428–432.
  25. Renoux C, Dell’Aniello S, Garbe E, Suissa S. Transdermal and oral hormone replacement therapy and the risk of stroke: A nested case-control study. BMJ. 2010;340:c2519. doi:10.1136/bmj.c2519.
  26. American College of Obstetricians and Gynecologists. Committee Opinion No. 556: Postmenopausal estrogen therapy: Route of administration and risk of venous thromboembolism. Obstet Gynecol. 2013;121(4):887–890.
  27. Smith NL, Blondon M, Wiggins KL, Harrington LB, van Hylckama Vlieg A, Floyd JS. Lower risk of cardiovascular events in postmenopausal women taking oral estradiol compared with oral conjugated equine estrogens. JAMA Intern Med. 2014;14(1):25–31.
  28. North American Menopause Society. 2013 Position Statement: Management of symptomatic vulvovaginal atrophy. Menopause. 2013;20(9):888–902.
  29. Nelken RS, Ozel BZ, Leegant AR, Felix JC, Mishell DR. Randomized trial of estradiol vaginal ring versus oral oxybutynin for the treatment of overactive bladder. Menopause. 2011;18(9):962–966.
  30. Eriksen BC. A randomized, open, parallel-group study on the preventive effect of an estradiol-releasing vaginal ring (Estring) on recurrent urinary tract infections in postmenopausal women. Am J Obstet Gynecol. 1999;180(5):1072–1079.
  31. North American Menopause Society. 2010 Position Statement: Management of osteoporosis in postmenopausal women. Menopause. 2010;17(1):25–54.
Issue
OBG Management - 26(2)
Issue
OBG Management - 26(2)
Page Number
59-65
Page Number
59-65
Publications
Publications
Topics
Article Type
Display Headline
When should a menopausal woman discontinue hormone therapy?
Display Headline
When should a menopausal woman discontinue hormone therapy?
Legacy Keywords
Andrew Kaunitz,James Simon,JoAnn Pinkerton,menopause,hormone therapy,menopausal symptoms,hot flushes,HT,transdermal estradiol,oral micronized progesterone,North American Menopause Society,NAMS,optimal duration of HT,vasomotor symptoms,estrogen-only,estrogen-progestin,Women's Health Initiative,WHI,VMS,osteoporosis,hysterectomy,bone mineral density,BMD,
Legacy Keywords
Andrew Kaunitz,James Simon,JoAnn Pinkerton,menopause,hormone therapy,menopausal symptoms,hot flushes,HT,transdermal estradiol,oral micronized progesterone,North American Menopause Society,NAMS,optimal duration of HT,vasomotor symptoms,estrogen-only,estrogen-progestin,Women's Health Initiative,WHI,VMS,osteoporosis,hysterectomy,bone mineral density,BMD,
Sections
Article Source

PURLs Copyright

Inside the Article

Article PDF Media

Your postmenopausal patient reports a history of migraine

Article Type
Changed
Tue, 08/28/2018 - 11:02
Display Headline
Your postmenopausal patient reports a history of migraine

CASES IN MENOPAUSE
Brought to you by the menopause experts

Andrew M. Kaunitz, MD
Professor and Associate Chairman, Department of Obstetrics and Gynecology, University of Florida College of Medicine–Jacksonville. Dr. Kaunitz is a NAMS Board member and certified menopause practitioner. He also serves on the OBG Management Board of Editors.

JoAnn V. Pinkerton, MD
Professor, Department of Obstetrics and Gynecology, and Director, Division of Midlife Women’s Health, University of Virginia. Dr. Pinkerton is a North American Menopause Society (NAMS) past president and certified menopause practitioner. She also serves on the OBG Management Board of Editors.

James A. Simon, MD
Clinical Professor, Department of Obstetrics and Gynecology, George Washington University, and Medical Director, Women’s Health & Research Consultants, Washington, DC. Dr. Simon is a NAMS past president, a certified menopause practitioner, and a certified clinical densitometrist. He also serves on the OBG Management Board of Editors.

Disclosures
Dr. Kaunitz reports that his institution receives grant or research support from Bayer, Teva, Medical Diagnostic Laboratories, and Noven, and that he is a consultant to Bayer, Actavis, and Teva.

Dr. Pinkerton reports that her institution receives consulting fees from Pfizer, DepoMed, Shionogi, and Noven and multicenter research fees from DepoMed, Endoceutics, and Bionova.

Dr. Simon reports being a consultant to or on the advisory boards of Abbott Laboratories, Agile Therapeutics, Amgen, Ascend Therapeutics, BioSante, Depomed, Lelo, MD Therapeutics, Meda Pharmaceuticals, Merck, Noven, Novo Nordisk, Novogyne, Pfizer, Shionogi, Shippan Point Advisors LLC, Slate Pharmaceuticals, Sprout Pharmaceuticals, Teva, Warner Chilcott, and Watson. He also reports receiving (currently or in the past year) grant/research support from BioSante, EndoCeutics, Novo Nordisk, Novogyne, Palatin Technologies, Teva, and Warner Chilcott. He reports serving on the speakers bureaus of Amgen, Merck, Novartis, Noven, Novo Nordisk, Novogyne, Teva, and Warner Chilcott. Dr. Simon is currently the Chief Medical Officer for Sprout Pharmaceuticals.

CASE: Menopausal symptoms and a history of migraine with aura

Your new patient is a 52-year-old woman (G2P2) who reports a long history of two types of migraine: menstrually related migraine without aura and nonmenstrually related migraine with aura (usually involving visual scotomata). Other than the history of migraine, her health is good. Now postmenopausal, she has been referred to you by her primary care physician (PCP) for management of severe vasomotor symptoms and sleep disturbance.

Because of this patient’s history of migraine, her PCP declined to prescribe oral contraceptives (OCs) in the past over concern of increasing her risk of stroke. For her vasomotor symptoms, her PCP prescribed a trial of venlafaxine (Effexor) 75 mg daily, but her orgasms, which always had been difficult to achieve, became impossible. In addition, the patient began to perspire heavily unrelated to her hot flashes. As a result, she describes her mood as “terrible,” her energy level as “miniscule,” and she reports losing interest in sex completely (“I am just too tired”). She and her referring physician wonder whether it would be safe to try hormone therapy (HT).

A physical examination, including funduscopic assessment, reveals no abnormalities. Her blood pressure is 126/70 mm Hg, and blood chemistry results, including C-reactive protein, 25-hydroxy vitamin D, a complete blood count, and lipid profile, are all normal.

Would you offer this patient the option of HT?

Migraine affects roughly twice as many women as men.1 During the reproductive years, rapid fluctuations in ovarian hormones—both increases at midcycle and, to a greater extent, decreases during the premenstrual phase—are believed to be migraine “triggers.” Women who experience menstrually related migraine before menopause typically have an increased risk of migraine during perimenopause, with a significant reduction of migraine symptoms following menopause.2

Side effects of SSRIs and SNRIs
Most providers are aware that selective serotonin reuptake inhibitors (SSRIs) cause sexual side effects in as many as 80% of users. There is a dose-related pattern to reports of sexual problems among SSRI users, with higher doses causing more problems. The most common sexual symptoms associated with SSRIs are delayed ejaculation and absent or delayed orgasm.3

What is not as widely known is that even serotonin-norepinephrine reuptake inhibitors (SNRIs) can cause sexual dysfunction. For example, in a prospective, multicenter study from Spain involving more than
1,000 outpatients, all of whom were taking an antidepressant, the overall rate of sexual dysfunction was 59%.4 Sexual dysfunction was most common among users of SSRIs and venlafaxine, an SNRI.5

Another common side effect of venlafaxine, sweating, is unrelated to hot flashes.5 So two of this patient’s concerns—orgasmic difficulties and profuse sweating unrelated to hot flashes—may have been caused or worsened by her antidepressant.

Another nonhormonal option
In June 2013, the US Food and Drug Administration (FDA) approved paroxetine mesylate (Brisdelle), an SSRI, for the treatment of moderate to severe menopausal vasomotor symptoms. Because the drug is a strong CYP2D6 inhibitor, it should not be given to women taking another medication that is metabolized by CYP2D6—most notably, tamoxifen.

Preliminary data on this drug suggest that, at the recommended dose (7.5 mg/d), it has no effect on sexual function.6 For this reason, it is another option for our patient to consider.

 

 

Related article: The gynecologist's role in managing menstrual migraine Anne H. Calhoun, MD 

Migraine and the risk of stroke
Migraine with aura has been associated with an increased risk of stroke and other cerebral vascular events,7 and that risk is further elevated in patients treated with OCs.8 Although migraine without aura also may be associated with an elevated risk of stroke, OCs do not further increase that risk.

Andrew M. Kaunitz, MD:
The elevated risk of stroke associated with use of OCs by women with migraine with aura appears to relate, in particular, to older, higher-dose OC formulations.9,10

Some practitioners assume that the data on the risk of stroke associated with OC use also applies to hormone therapy, but there is no evidence that HT, in which doses of estrogen are far lower than in OCs, increases the risk of stroke in migraineurs to any greater degree than would be expected in unselected populations (ie, as noted in the Women’s Health Initiative, Nurses Health Study, or other large investigations). Therefore, HT would be an appropriate option for this patient if her very slight risk of stroke on HT would be acceptable to the practitioner and patient.

JoAnn V. Pinkerton, MD:
The route of administration is critical here. In relatively healthy postmenopausal women (average age, 63), combined continuous oral HT significantly increased the risk of stroke. After 3 years of use, the absolute risk was 18 cases of stroke per 1,000 HT users (95% confidence interval [CI], 14–23). And oral estrogen-only therapy increased the risk of stroke after 7 years of use, with an absolute risk of 32 cases per 1,000 HT users (95% CI, 25–40).11

The limited clinical evidence available on the effects of tramsdermal estradiol on stroke risk indicates that the risk is not increased.12

Choosing an HT formulation

Consider the pharmacokinetic profile. Many oral estrogen HT products have rapid-release characteristics that make them likely to contribute to rapid rises and falls in the user’s estrogen level. Oral estrogens also are associated with procoagulant properties that may increase the risk of thrombosis and thromboembolism. Nonoral estrogens do not appear to increase these risks.13

Nonoral estrogens (patches, gels, sprays, lotions, and vaginal rings) provide a more stable pharmacokinetic profile, as do some oral products with controlled-release properties.

As for progestins, some formulations (medroxyprogesterone acetate) tend to cause vasoconstriction, whereas others (micronized progesterone) tend to be vasodilators. Whether these properties affect the rate of migraine or risk of stroke is unclear.

My management approach for this patient
In the absence of any systematic data on the use of HT in this clinical setting, and without any concrete suggestions from migraine experts, I would take the following three-step approach:

1. I would begin with a low-dose nonoral estradiol formulation, prescribing it without a progestin even in a woman who still has a uterus. My aim: to determine the lowest effective dose of HT for this particular patient. I would follow the patient on this dose for 3 months.

JoAnn V. Pinkerton, MD:
Another goal is to determine whether transdermal estradiol increases headaches. Before settling on a therapy, however, I would ask how long this patient has been postmenopausal, how long she has been experiencing vasomotor symptoms, and how severe those symptoms are. For example, is she having 7 or 8 hot flashes per day and waking from night sweats once or twice per night? I also would ask her how long she remained on the venlafaxine. The additional information would allow me to fine-tune her treatment.

 
2. If this formulation is tolerated, I would add micronized progesterone (oral or vaginal) for endometrial protection.

JoAnn V. Pinkerton, MD:
I would give oral progesterone if it is FDA approved for postmenopausal use, vaginal progesterone if it isn’t.

3. I would follow the patient’s clinical response—specifically, her vasomotor symptoms and rate of migraine with or without aura.

Hormone therapy is one option for postmenopausal migraineurs with bothersome vasomotor symptoms
Many women with a history of migraine move into menopause expecting their condition to improve, says headache expert Anne H. Calhoun, MD, a founder of the Carolina Headache Institute in Chapel Hill, North Carolina.

“Over the years, these women have heard that things get better with menopause.”

For women with a history of episodic migraine, that expectation is realistic, Calhoun says. “But for women with chronic migraine, who may experience a low-grade headache on a daily, or almost daily basis, with 10 or 12 severe headaches in a month, things usually get worse after menopause because the sleep issues of menopause are superimposed on the migraine.”

Dr. Calhoun observes that hormone therapy (HT) has never been contraindicated in women with migraine, although many neurologists are hesitant to prescribe any hormones for this population.

Before prescribing HT to a postmenopausal migraineur, Dr. Calhoun considers a range of variables, including sleep patterns, current medications, anxiety, frequency and severity of vasomotor symptoms, and any other problems the patient may be experiencing.

“It’s basically the same assessment as with any postmenopausal patient—to determine whether HT is a reasonable option,” she says.

And when she determines that HT is appropriate, “I almost exclusively use transdermal HT. I also am more likely to prescribe continuous use of a transdermal patch or skin gel, as I want to achieve very consistent hormone levels, day in and day out,” she says.

 

 

My bottom line

No systematic data on the use of HT in migraineurs has been published. In the absence of such data, some practitioners have extrapolated data on the use of OCs in this population and decline to prescribe HT to women with migraine. However, HT and OCs are vastly different in formulation, dose, and risks. Rather than make assumptions on the basis of irrelevant data, we should conduct studies of HT use in migraineurs.

Related article: Update on Menopause Andrew M. Kaunitz, MD (June 2013)

Women who have menstrually related migraine typically have an increased risk of migraine during perimenopause and a significant reduction in migraine following menopause. If hot flashes are bothersome, these women certainly can use HT. I recommend prescribing HT in a continuous fashion that maintains stable hormone levels in the blood, as fluctuating hormones tend to trigger migraines.

Andrew M. Kaunitz, MD:
I would just add that transdermal estradiol is preferred, to be given at the lowest effective dose.

Do you have a troubling case in menopause? Suggest it to the expert panel: obg@frontlinemedcom.com. They may address your management dilemma in a future issue.

We want to hear from you! Send us your Letter to the Editor 

References

1. Shuster LT, Faubion SS. Sood R, Casey PM. Hormonal manipulation strategies in the management of menstrual migraine and other hormonally related headaches. Curr Neurol Neurosci Rep. 2011;11(2):131–138.

2. Loder E, Rizzoli P, Golub J. Hormonal management of migraine associated with menses and the menopause: a clinical review. Headache. 2007;47(2):329–340.

3. Balon R. SSRI-associated sexual dysfunction. Am J Psychiatry. 2006;163:1504–1509.

4. Taylor MJ. Strategies for managing antidepressant-induced sexual dysfunction: a review. Curr Psychiatry Rep. 2006;8(6):431–436.

5. Effexor [package insert]. New York, NY: Pfizer; 2012.

6. Brisdelle [package insert]. Miami, FL: Noven Therapeutics; 2013.

7. Etminan M, Takkouche B, Isorna FC, Samli A. Risk of ischaemic stroke in people with migraine: systematic review and meta-analysis of observational studies. BMJ. 2005;330(7482):63–65.

8. Becker WJ. Use of oral contraceptives in patients with migraine. Neurology. 1999;53(4 Suppl 1):S19–S25.

9. WHO Collaborative Study of Cardiovascular Disease and Steroid Hormone Contraception. Ischaemic stroke and combined oral contraceptives: results of an international, multicentre, case-control study. Lancet. 1996;348(9026): 498–505.

10. Petitti DB, Sidney S, Bernstein A, Wolf S, Quesenberry C, Ziel HK. Stroke in users of low-dose oral contraceptives. N Engl J Med. 1996;335(1):8-15.

11. Marjoribanks J, Farquhar C, Roberts H, Lethaby A. Long-term hormone therapy for perimenopausal and postmenopausal women. Cochrane Database Syst Rev. 2012;7:CD004143. doi: 10.1002/14651858.CD004143.pub4

12. Renoux C, Dell’aniello S, Garbe E, Suissa S. Transdermal and oral hormone replacement therapy and the risk of stroke: a nested case-control study. BMJ. 2010;340:c2519.

13. Kaunitz AM. Update on Menopause. OBG Manag. 2013;25(6):36–43, 49.

Article PDF
Author and Disclosure Information

James A. Simon, MD

Dr. Andrew M. Kaunitz and Dr. JoAnn V. Pinkerton provided peer review and comments for Dr. Simon’s case study.

Issue
OBG Management - 25(10)
Publications
Topics
Page Number
18-24
Legacy Keywords
James A. Simon MD, JoAnn V. Pinkerton MD, Andrew M. Kaunitz MD, cases in menopause, your postmenopausal patient reports a history of migraine, migraine with aura, migraine, hormone therapy, HT, formulation, postmenopause, postmenopausal, stroke, SSRIa, SNRIs
Sections
Author and Disclosure Information

James A. Simon, MD

Dr. Andrew M. Kaunitz and Dr. JoAnn V. Pinkerton provided peer review and comments for Dr. Simon’s case study.

Author and Disclosure Information

James A. Simon, MD

Dr. Andrew M. Kaunitz and Dr. JoAnn V. Pinkerton provided peer review and comments for Dr. Simon’s case study.

Article PDF
Article PDF
Related Articles

CASES IN MENOPAUSE
Brought to you by the menopause experts

Andrew M. Kaunitz, MD
Professor and Associate Chairman, Department of Obstetrics and Gynecology, University of Florida College of Medicine–Jacksonville. Dr. Kaunitz is a NAMS Board member and certified menopause practitioner. He also serves on the OBG Management Board of Editors.

JoAnn V. Pinkerton, MD
Professor, Department of Obstetrics and Gynecology, and Director, Division of Midlife Women’s Health, University of Virginia. Dr. Pinkerton is a North American Menopause Society (NAMS) past president and certified menopause practitioner. She also serves on the OBG Management Board of Editors.

James A. Simon, MD
Clinical Professor, Department of Obstetrics and Gynecology, George Washington University, and Medical Director, Women’s Health & Research Consultants, Washington, DC. Dr. Simon is a NAMS past president, a certified menopause practitioner, and a certified clinical densitometrist. He also serves on the OBG Management Board of Editors.

Disclosures
Dr. Kaunitz reports that his institution receives grant or research support from Bayer, Teva, Medical Diagnostic Laboratories, and Noven, and that he is a consultant to Bayer, Actavis, and Teva.

Dr. Pinkerton reports that her institution receives consulting fees from Pfizer, DepoMed, Shionogi, and Noven and multicenter research fees from DepoMed, Endoceutics, and Bionova.

Dr. Simon reports being a consultant to or on the advisory boards of Abbott Laboratories, Agile Therapeutics, Amgen, Ascend Therapeutics, BioSante, Depomed, Lelo, MD Therapeutics, Meda Pharmaceuticals, Merck, Noven, Novo Nordisk, Novogyne, Pfizer, Shionogi, Shippan Point Advisors LLC, Slate Pharmaceuticals, Sprout Pharmaceuticals, Teva, Warner Chilcott, and Watson. He also reports receiving (currently or in the past year) grant/research support from BioSante, EndoCeutics, Novo Nordisk, Novogyne, Palatin Technologies, Teva, and Warner Chilcott. He reports serving on the speakers bureaus of Amgen, Merck, Novartis, Noven, Novo Nordisk, Novogyne, Teva, and Warner Chilcott. Dr. Simon is currently the Chief Medical Officer for Sprout Pharmaceuticals.

CASE: Menopausal symptoms and a history of migraine with aura

Your new patient is a 52-year-old woman (G2P2) who reports a long history of two types of migraine: menstrually related migraine without aura and nonmenstrually related migraine with aura (usually involving visual scotomata). Other than the history of migraine, her health is good. Now postmenopausal, she has been referred to you by her primary care physician (PCP) for management of severe vasomotor symptoms and sleep disturbance.

Because of this patient’s history of migraine, her PCP declined to prescribe oral contraceptives (OCs) in the past over concern of increasing her risk of stroke. For her vasomotor symptoms, her PCP prescribed a trial of venlafaxine (Effexor) 75 mg daily, but her orgasms, which always had been difficult to achieve, became impossible. In addition, the patient began to perspire heavily unrelated to her hot flashes. As a result, she describes her mood as “terrible,” her energy level as “miniscule,” and she reports losing interest in sex completely (“I am just too tired”). She and her referring physician wonder whether it would be safe to try hormone therapy (HT).

A physical examination, including funduscopic assessment, reveals no abnormalities. Her blood pressure is 126/70 mm Hg, and blood chemistry results, including C-reactive protein, 25-hydroxy vitamin D, a complete blood count, and lipid profile, are all normal.

Would you offer this patient the option of HT?

Migraine affects roughly twice as many women as men.1 During the reproductive years, rapid fluctuations in ovarian hormones—both increases at midcycle and, to a greater extent, decreases during the premenstrual phase—are believed to be migraine “triggers.” Women who experience menstrually related migraine before menopause typically have an increased risk of migraine during perimenopause, with a significant reduction of migraine symptoms following menopause.2

Side effects of SSRIs and SNRIs
Most providers are aware that selective serotonin reuptake inhibitors (SSRIs) cause sexual side effects in as many as 80% of users. There is a dose-related pattern to reports of sexual problems among SSRI users, with higher doses causing more problems. The most common sexual symptoms associated with SSRIs are delayed ejaculation and absent or delayed orgasm.3

What is not as widely known is that even serotonin-norepinephrine reuptake inhibitors (SNRIs) can cause sexual dysfunction. For example, in a prospective, multicenter study from Spain involving more than
1,000 outpatients, all of whom were taking an antidepressant, the overall rate of sexual dysfunction was 59%.4 Sexual dysfunction was most common among users of SSRIs and venlafaxine, an SNRI.5

Another common side effect of venlafaxine, sweating, is unrelated to hot flashes.5 So two of this patient’s concerns—orgasmic difficulties and profuse sweating unrelated to hot flashes—may have been caused or worsened by her antidepressant.

Another nonhormonal option
In June 2013, the US Food and Drug Administration (FDA) approved paroxetine mesylate (Brisdelle), an SSRI, for the treatment of moderate to severe menopausal vasomotor symptoms. Because the drug is a strong CYP2D6 inhibitor, it should not be given to women taking another medication that is metabolized by CYP2D6—most notably, tamoxifen.

Preliminary data on this drug suggest that, at the recommended dose (7.5 mg/d), it has no effect on sexual function.6 For this reason, it is another option for our patient to consider.

 

 

Related article: The gynecologist's role in managing menstrual migraine Anne H. Calhoun, MD 

Migraine and the risk of stroke
Migraine with aura has been associated with an increased risk of stroke and other cerebral vascular events,7 and that risk is further elevated in patients treated with OCs.8 Although migraine without aura also may be associated with an elevated risk of stroke, OCs do not further increase that risk.

Andrew M. Kaunitz, MD:
The elevated risk of stroke associated with use of OCs by women with migraine with aura appears to relate, in particular, to older, higher-dose OC formulations.9,10

Some practitioners assume that the data on the risk of stroke associated with OC use also applies to hormone therapy, but there is no evidence that HT, in which doses of estrogen are far lower than in OCs, increases the risk of stroke in migraineurs to any greater degree than would be expected in unselected populations (ie, as noted in the Women’s Health Initiative, Nurses Health Study, or other large investigations). Therefore, HT would be an appropriate option for this patient if her very slight risk of stroke on HT would be acceptable to the practitioner and patient.

JoAnn V. Pinkerton, MD:
The route of administration is critical here. In relatively healthy postmenopausal women (average age, 63), combined continuous oral HT significantly increased the risk of stroke. After 3 years of use, the absolute risk was 18 cases of stroke per 1,000 HT users (95% confidence interval [CI], 14–23). And oral estrogen-only therapy increased the risk of stroke after 7 years of use, with an absolute risk of 32 cases per 1,000 HT users (95% CI, 25–40).11

The limited clinical evidence available on the effects of tramsdermal estradiol on stroke risk indicates that the risk is not increased.12

Choosing an HT formulation

Consider the pharmacokinetic profile. Many oral estrogen HT products have rapid-release characteristics that make them likely to contribute to rapid rises and falls in the user’s estrogen level. Oral estrogens also are associated with procoagulant properties that may increase the risk of thrombosis and thromboembolism. Nonoral estrogens do not appear to increase these risks.13

Nonoral estrogens (patches, gels, sprays, lotions, and vaginal rings) provide a more stable pharmacokinetic profile, as do some oral products with controlled-release properties.

As for progestins, some formulations (medroxyprogesterone acetate) tend to cause vasoconstriction, whereas others (micronized progesterone) tend to be vasodilators. Whether these properties affect the rate of migraine or risk of stroke is unclear.

My management approach for this patient
In the absence of any systematic data on the use of HT in this clinical setting, and without any concrete suggestions from migraine experts, I would take the following three-step approach:

1. I would begin with a low-dose nonoral estradiol formulation, prescribing it without a progestin even in a woman who still has a uterus. My aim: to determine the lowest effective dose of HT for this particular patient. I would follow the patient on this dose for 3 months.

JoAnn V. Pinkerton, MD:
Another goal is to determine whether transdermal estradiol increases headaches. Before settling on a therapy, however, I would ask how long this patient has been postmenopausal, how long she has been experiencing vasomotor symptoms, and how severe those symptoms are. For example, is she having 7 or 8 hot flashes per day and waking from night sweats once or twice per night? I also would ask her how long she remained on the venlafaxine. The additional information would allow me to fine-tune her treatment.

 
2. If this formulation is tolerated, I would add micronized progesterone (oral or vaginal) for endometrial protection.

JoAnn V. Pinkerton, MD:
I would give oral progesterone if it is FDA approved for postmenopausal use, vaginal progesterone if it isn’t.

3. I would follow the patient’s clinical response—specifically, her vasomotor symptoms and rate of migraine with or without aura.

Hormone therapy is one option for postmenopausal migraineurs with bothersome vasomotor symptoms
Many women with a history of migraine move into menopause expecting their condition to improve, says headache expert Anne H. Calhoun, MD, a founder of the Carolina Headache Institute in Chapel Hill, North Carolina.

“Over the years, these women have heard that things get better with menopause.”

For women with a history of episodic migraine, that expectation is realistic, Calhoun says. “But for women with chronic migraine, who may experience a low-grade headache on a daily, or almost daily basis, with 10 or 12 severe headaches in a month, things usually get worse after menopause because the sleep issues of menopause are superimposed on the migraine.”

Dr. Calhoun observes that hormone therapy (HT) has never been contraindicated in women with migraine, although many neurologists are hesitant to prescribe any hormones for this population.

Before prescribing HT to a postmenopausal migraineur, Dr. Calhoun considers a range of variables, including sleep patterns, current medications, anxiety, frequency and severity of vasomotor symptoms, and any other problems the patient may be experiencing.

“It’s basically the same assessment as with any postmenopausal patient—to determine whether HT is a reasonable option,” she says.

And when she determines that HT is appropriate, “I almost exclusively use transdermal HT. I also am more likely to prescribe continuous use of a transdermal patch or skin gel, as I want to achieve very consistent hormone levels, day in and day out,” she says.

 

 

My bottom line

No systematic data on the use of HT in migraineurs has been published. In the absence of such data, some practitioners have extrapolated data on the use of OCs in this population and decline to prescribe HT to women with migraine. However, HT and OCs are vastly different in formulation, dose, and risks. Rather than make assumptions on the basis of irrelevant data, we should conduct studies of HT use in migraineurs.

Related article: Update on Menopause Andrew M. Kaunitz, MD (June 2013)

Women who have menstrually related migraine typically have an increased risk of migraine during perimenopause and a significant reduction in migraine following menopause. If hot flashes are bothersome, these women certainly can use HT. I recommend prescribing HT in a continuous fashion that maintains stable hormone levels in the blood, as fluctuating hormones tend to trigger migraines.

Andrew M. Kaunitz, MD:
I would just add that transdermal estradiol is preferred, to be given at the lowest effective dose.

Do you have a troubling case in menopause? Suggest it to the expert panel: obg@frontlinemedcom.com. They may address your management dilemma in a future issue.

We want to hear from you! Send us your Letter to the Editor 

CASES IN MENOPAUSE
Brought to you by the menopause experts

Andrew M. Kaunitz, MD
Professor and Associate Chairman, Department of Obstetrics and Gynecology, University of Florida College of Medicine–Jacksonville. Dr. Kaunitz is a NAMS Board member and certified menopause practitioner. He also serves on the OBG Management Board of Editors.

JoAnn V. Pinkerton, MD
Professor, Department of Obstetrics and Gynecology, and Director, Division of Midlife Women’s Health, University of Virginia. Dr. Pinkerton is a North American Menopause Society (NAMS) past president and certified menopause practitioner. She also serves on the OBG Management Board of Editors.

James A. Simon, MD
Clinical Professor, Department of Obstetrics and Gynecology, George Washington University, and Medical Director, Women’s Health & Research Consultants, Washington, DC. Dr. Simon is a NAMS past president, a certified menopause practitioner, and a certified clinical densitometrist. He also serves on the OBG Management Board of Editors.

Disclosures
Dr. Kaunitz reports that his institution receives grant or research support from Bayer, Teva, Medical Diagnostic Laboratories, and Noven, and that he is a consultant to Bayer, Actavis, and Teva.

Dr. Pinkerton reports that her institution receives consulting fees from Pfizer, DepoMed, Shionogi, and Noven and multicenter research fees from DepoMed, Endoceutics, and Bionova.

Dr. Simon reports being a consultant to or on the advisory boards of Abbott Laboratories, Agile Therapeutics, Amgen, Ascend Therapeutics, BioSante, Depomed, Lelo, MD Therapeutics, Meda Pharmaceuticals, Merck, Noven, Novo Nordisk, Novogyne, Pfizer, Shionogi, Shippan Point Advisors LLC, Slate Pharmaceuticals, Sprout Pharmaceuticals, Teva, Warner Chilcott, and Watson. He also reports receiving (currently or in the past year) grant/research support from BioSante, EndoCeutics, Novo Nordisk, Novogyne, Palatin Technologies, Teva, and Warner Chilcott. He reports serving on the speakers bureaus of Amgen, Merck, Novartis, Noven, Novo Nordisk, Novogyne, Teva, and Warner Chilcott. Dr. Simon is currently the Chief Medical Officer for Sprout Pharmaceuticals.

CASE: Menopausal symptoms and a history of migraine with aura

Your new patient is a 52-year-old woman (G2P2) who reports a long history of two types of migraine: menstrually related migraine without aura and nonmenstrually related migraine with aura (usually involving visual scotomata). Other than the history of migraine, her health is good. Now postmenopausal, she has been referred to you by her primary care physician (PCP) for management of severe vasomotor symptoms and sleep disturbance.

Because of this patient’s history of migraine, her PCP declined to prescribe oral contraceptives (OCs) in the past over concern of increasing her risk of stroke. For her vasomotor symptoms, her PCP prescribed a trial of venlafaxine (Effexor) 75 mg daily, but her orgasms, which always had been difficult to achieve, became impossible. In addition, the patient began to perspire heavily unrelated to her hot flashes. As a result, she describes her mood as “terrible,” her energy level as “miniscule,” and she reports losing interest in sex completely (“I am just too tired”). She and her referring physician wonder whether it would be safe to try hormone therapy (HT).

A physical examination, including funduscopic assessment, reveals no abnormalities. Her blood pressure is 126/70 mm Hg, and blood chemistry results, including C-reactive protein, 25-hydroxy vitamin D, a complete blood count, and lipid profile, are all normal.

Would you offer this patient the option of HT?

Migraine affects roughly twice as many women as men.1 During the reproductive years, rapid fluctuations in ovarian hormones—both increases at midcycle and, to a greater extent, decreases during the premenstrual phase—are believed to be migraine “triggers.” Women who experience menstrually related migraine before menopause typically have an increased risk of migraine during perimenopause, with a significant reduction of migraine symptoms following menopause.2

Side effects of SSRIs and SNRIs
Most providers are aware that selective serotonin reuptake inhibitors (SSRIs) cause sexual side effects in as many as 80% of users. There is a dose-related pattern to reports of sexual problems among SSRI users, with higher doses causing more problems. The most common sexual symptoms associated with SSRIs are delayed ejaculation and absent or delayed orgasm.3

What is not as widely known is that even serotonin-norepinephrine reuptake inhibitors (SNRIs) can cause sexual dysfunction. For example, in a prospective, multicenter study from Spain involving more than
1,000 outpatients, all of whom were taking an antidepressant, the overall rate of sexual dysfunction was 59%.4 Sexual dysfunction was most common among users of SSRIs and venlafaxine, an SNRI.5

Another common side effect of venlafaxine, sweating, is unrelated to hot flashes.5 So two of this patient’s concerns—orgasmic difficulties and profuse sweating unrelated to hot flashes—may have been caused or worsened by her antidepressant.

Another nonhormonal option
In June 2013, the US Food and Drug Administration (FDA) approved paroxetine mesylate (Brisdelle), an SSRI, for the treatment of moderate to severe menopausal vasomotor symptoms. Because the drug is a strong CYP2D6 inhibitor, it should not be given to women taking another medication that is metabolized by CYP2D6—most notably, tamoxifen.

Preliminary data on this drug suggest that, at the recommended dose (7.5 mg/d), it has no effect on sexual function.6 For this reason, it is another option for our patient to consider.

 

 

Related article: The gynecologist's role in managing menstrual migraine Anne H. Calhoun, MD 

Migraine and the risk of stroke
Migraine with aura has been associated with an increased risk of stroke and other cerebral vascular events,7 and that risk is further elevated in patients treated with OCs.8 Although migraine without aura also may be associated with an elevated risk of stroke, OCs do not further increase that risk.

Andrew M. Kaunitz, MD:
The elevated risk of stroke associated with use of OCs by women with migraine with aura appears to relate, in particular, to older, higher-dose OC formulations.9,10

Some practitioners assume that the data on the risk of stroke associated with OC use also applies to hormone therapy, but there is no evidence that HT, in which doses of estrogen are far lower than in OCs, increases the risk of stroke in migraineurs to any greater degree than would be expected in unselected populations (ie, as noted in the Women’s Health Initiative, Nurses Health Study, or other large investigations). Therefore, HT would be an appropriate option for this patient if her very slight risk of stroke on HT would be acceptable to the practitioner and patient.

JoAnn V. Pinkerton, MD:
The route of administration is critical here. In relatively healthy postmenopausal women (average age, 63), combined continuous oral HT significantly increased the risk of stroke. After 3 years of use, the absolute risk was 18 cases of stroke per 1,000 HT users (95% confidence interval [CI], 14–23). And oral estrogen-only therapy increased the risk of stroke after 7 years of use, with an absolute risk of 32 cases per 1,000 HT users (95% CI, 25–40).11

The limited clinical evidence available on the effects of tramsdermal estradiol on stroke risk indicates that the risk is not increased.12

Choosing an HT formulation

Consider the pharmacokinetic profile. Many oral estrogen HT products have rapid-release characteristics that make them likely to contribute to rapid rises and falls in the user’s estrogen level. Oral estrogens also are associated with procoagulant properties that may increase the risk of thrombosis and thromboembolism. Nonoral estrogens do not appear to increase these risks.13

Nonoral estrogens (patches, gels, sprays, lotions, and vaginal rings) provide a more stable pharmacokinetic profile, as do some oral products with controlled-release properties.

As for progestins, some formulations (medroxyprogesterone acetate) tend to cause vasoconstriction, whereas others (micronized progesterone) tend to be vasodilators. Whether these properties affect the rate of migraine or risk of stroke is unclear.

My management approach for this patient
In the absence of any systematic data on the use of HT in this clinical setting, and without any concrete suggestions from migraine experts, I would take the following three-step approach:

1. I would begin with a low-dose nonoral estradiol formulation, prescribing it without a progestin even in a woman who still has a uterus. My aim: to determine the lowest effective dose of HT for this particular patient. I would follow the patient on this dose for 3 months.

JoAnn V. Pinkerton, MD:
Another goal is to determine whether transdermal estradiol increases headaches. Before settling on a therapy, however, I would ask how long this patient has been postmenopausal, how long she has been experiencing vasomotor symptoms, and how severe those symptoms are. For example, is she having 7 or 8 hot flashes per day and waking from night sweats once or twice per night? I also would ask her how long she remained on the venlafaxine. The additional information would allow me to fine-tune her treatment.

 
2. If this formulation is tolerated, I would add micronized progesterone (oral or vaginal) for endometrial protection.

JoAnn V. Pinkerton, MD:
I would give oral progesterone if it is FDA approved for postmenopausal use, vaginal progesterone if it isn’t.

3. I would follow the patient’s clinical response—specifically, her vasomotor symptoms and rate of migraine with or without aura.

Hormone therapy is one option for postmenopausal migraineurs with bothersome vasomotor symptoms
Many women with a history of migraine move into menopause expecting their condition to improve, says headache expert Anne H. Calhoun, MD, a founder of the Carolina Headache Institute in Chapel Hill, North Carolina.

“Over the years, these women have heard that things get better with menopause.”

For women with a history of episodic migraine, that expectation is realistic, Calhoun says. “But for women with chronic migraine, who may experience a low-grade headache on a daily, or almost daily basis, with 10 or 12 severe headaches in a month, things usually get worse after menopause because the sleep issues of menopause are superimposed on the migraine.”

Dr. Calhoun observes that hormone therapy (HT) has never been contraindicated in women with migraine, although many neurologists are hesitant to prescribe any hormones for this population.

Before prescribing HT to a postmenopausal migraineur, Dr. Calhoun considers a range of variables, including sleep patterns, current medications, anxiety, frequency and severity of vasomotor symptoms, and any other problems the patient may be experiencing.

“It’s basically the same assessment as with any postmenopausal patient—to determine whether HT is a reasonable option,” she says.

And when she determines that HT is appropriate, “I almost exclusively use transdermal HT. I also am more likely to prescribe continuous use of a transdermal patch or skin gel, as I want to achieve very consistent hormone levels, day in and day out,” she says.

 

 

My bottom line

No systematic data on the use of HT in migraineurs has been published. In the absence of such data, some practitioners have extrapolated data on the use of OCs in this population and decline to prescribe HT to women with migraine. However, HT and OCs are vastly different in formulation, dose, and risks. Rather than make assumptions on the basis of irrelevant data, we should conduct studies of HT use in migraineurs.

Related article: Update on Menopause Andrew M. Kaunitz, MD (June 2013)

Women who have menstrually related migraine typically have an increased risk of migraine during perimenopause and a significant reduction in migraine following menopause. If hot flashes are bothersome, these women certainly can use HT. I recommend prescribing HT in a continuous fashion that maintains stable hormone levels in the blood, as fluctuating hormones tend to trigger migraines.

Andrew M. Kaunitz, MD:
I would just add that transdermal estradiol is preferred, to be given at the lowest effective dose.

Do you have a troubling case in menopause? Suggest it to the expert panel: obg@frontlinemedcom.com. They may address your management dilemma in a future issue.

We want to hear from you! Send us your Letter to the Editor 

References

1. Shuster LT, Faubion SS. Sood R, Casey PM. Hormonal manipulation strategies in the management of menstrual migraine and other hormonally related headaches. Curr Neurol Neurosci Rep. 2011;11(2):131–138.

2. Loder E, Rizzoli P, Golub J. Hormonal management of migraine associated with menses and the menopause: a clinical review. Headache. 2007;47(2):329–340.

3. Balon R. SSRI-associated sexual dysfunction. Am J Psychiatry. 2006;163:1504–1509.

4. Taylor MJ. Strategies for managing antidepressant-induced sexual dysfunction: a review. Curr Psychiatry Rep. 2006;8(6):431–436.

5. Effexor [package insert]. New York, NY: Pfizer; 2012.

6. Brisdelle [package insert]. Miami, FL: Noven Therapeutics; 2013.

7. Etminan M, Takkouche B, Isorna FC, Samli A. Risk of ischaemic stroke in people with migraine: systematic review and meta-analysis of observational studies. BMJ. 2005;330(7482):63–65.

8. Becker WJ. Use of oral contraceptives in patients with migraine. Neurology. 1999;53(4 Suppl 1):S19–S25.

9. WHO Collaborative Study of Cardiovascular Disease and Steroid Hormone Contraception. Ischaemic stroke and combined oral contraceptives: results of an international, multicentre, case-control study. Lancet. 1996;348(9026): 498–505.

10. Petitti DB, Sidney S, Bernstein A, Wolf S, Quesenberry C, Ziel HK. Stroke in users of low-dose oral contraceptives. N Engl J Med. 1996;335(1):8-15.

11. Marjoribanks J, Farquhar C, Roberts H, Lethaby A. Long-term hormone therapy for perimenopausal and postmenopausal women. Cochrane Database Syst Rev. 2012;7:CD004143. doi: 10.1002/14651858.CD004143.pub4

12. Renoux C, Dell’aniello S, Garbe E, Suissa S. Transdermal and oral hormone replacement therapy and the risk of stroke: a nested case-control study. BMJ. 2010;340:c2519.

13. Kaunitz AM. Update on Menopause. OBG Manag. 2013;25(6):36–43, 49.

References

1. Shuster LT, Faubion SS. Sood R, Casey PM. Hormonal manipulation strategies in the management of menstrual migraine and other hormonally related headaches. Curr Neurol Neurosci Rep. 2011;11(2):131–138.

2. Loder E, Rizzoli P, Golub J. Hormonal management of migraine associated with menses and the menopause: a clinical review. Headache. 2007;47(2):329–340.

3. Balon R. SSRI-associated sexual dysfunction. Am J Psychiatry. 2006;163:1504–1509.

4. Taylor MJ. Strategies for managing antidepressant-induced sexual dysfunction: a review. Curr Psychiatry Rep. 2006;8(6):431–436.

5. Effexor [package insert]. New York, NY: Pfizer; 2012.

6. Brisdelle [package insert]. Miami, FL: Noven Therapeutics; 2013.

7. Etminan M, Takkouche B, Isorna FC, Samli A. Risk of ischaemic stroke in people with migraine: systematic review and meta-analysis of observational studies. BMJ. 2005;330(7482):63–65.

8. Becker WJ. Use of oral contraceptives in patients with migraine. Neurology. 1999;53(4 Suppl 1):S19–S25.

9. WHO Collaborative Study of Cardiovascular Disease and Steroid Hormone Contraception. Ischaemic stroke and combined oral contraceptives: results of an international, multicentre, case-control study. Lancet. 1996;348(9026): 498–505.

10. Petitti DB, Sidney S, Bernstein A, Wolf S, Quesenberry C, Ziel HK. Stroke in users of low-dose oral contraceptives. N Engl J Med. 1996;335(1):8-15.

11. Marjoribanks J, Farquhar C, Roberts H, Lethaby A. Long-term hormone therapy for perimenopausal and postmenopausal women. Cochrane Database Syst Rev. 2012;7:CD004143. doi: 10.1002/14651858.CD004143.pub4

12. Renoux C, Dell’aniello S, Garbe E, Suissa S. Transdermal and oral hormone replacement therapy and the risk of stroke: a nested case-control study. BMJ. 2010;340:c2519.

13. Kaunitz AM. Update on Menopause. OBG Manag. 2013;25(6):36–43, 49.

Issue
OBG Management - 25(10)
Issue
OBG Management - 25(10)
Page Number
18-24
Page Number
18-24
Publications
Publications
Topics
Article Type
Display Headline
Your postmenopausal patient reports a history of migraine
Display Headline
Your postmenopausal patient reports a history of migraine
Legacy Keywords
James A. Simon MD, JoAnn V. Pinkerton MD, Andrew M. Kaunitz MD, cases in menopause, your postmenopausal patient reports a history of migraine, migraine with aura, migraine, hormone therapy, HT, formulation, postmenopause, postmenopausal, stroke, SSRIa, SNRIs
Legacy Keywords
James A. Simon MD, JoAnn V. Pinkerton MD, Andrew M. Kaunitz MD, cases in menopause, your postmenopausal patient reports a history of migraine, migraine with aura, migraine, hormone therapy, HT, formulation, postmenopause, postmenopausal, stroke, SSRIa, SNRIs
Sections
Article Source

PURLs Copyright

Inside the Article
Article PDF Media

Your menopausal patient’s breast biopsy reveals atypical hyperplasia

Article Type
Changed
Tue, 08/28/2018 - 11:01
Display Headline
Your menopausal patient’s breast biopsy reveals atypical hyperplasia

CASES IN MENOPAUSE

A new series brought to you by the menopause experts

Andrew M. Kaunitz, MD
Professor and Associate Chairman, Department of Obstetrics and Gynecology, University of Florida College of Medicine–Jacksonville. Dr. Kaunitz is a NAMS Board member and certified menopause practitioner. He also serves on the OBG Management Board of Editors.
JoAnn V. Pinkerton, MD
Professor, Department of Obstetrics and Gynecology, and Director, Division of Midlife Women’s Health, University of Virginia. Dr. Pinkerton is a North American Menopause Society (NAMS) past president and certified menopause practitioner. She also serves on the OBG Management Board of Editors.
James A. Simon, MD
Clinical Professor, Department of Obstetrics and Gynecology, George Washington University, and Medical Director, Women’s Health & Research Consultants, Washington, DC. Dr. Simon is a NAMS past president, a certified menopause practitioner, and a certified clinical densitometrist. He also serves on the OBG Management Board of Editors.

Disclosures
Dr. Kaunitz reports that his institution receives grant or research support from Agile, Bayer, Endoceutics, Teva, Medical Diagnostic Laboratories, and Noven, and that he is a consultant to Bayer, Merck, and Teva.

Dr. Pinkerton reports that her institution receives consulting fees from Pfizer, DepoMed, Shionogi, and Noven and multicenter research fees from DepoMed, Endoceutics, and Bionova.

Dr. Simon reports being a consultant to or on the advisory boards of Abbott Laboratories, Agile Therapeutics, Amgen, Ascend Therapeutics, BioSante, Depomed, Lelo, MD Therapeutics, Meda Pharmaceuticals, Merck, Noven, Novo Nordisk, Novogyne, Pfizer, Shionogi, Shippan Point Advisors LLC, Slate Pharmaceuticals, Sprout Pharmaceuticals, Teva, Warner Chilcott, and Watson. He also reports receiving (currently or in the past year) grant/research support from BioSante, EndoCeutics, Novo Nordisk, Novogyne, Palatin Technologies, Teva, and Warner Chilcott. He reports serving on the speakers bureaus of Amgen, Merck, Novartis, Noven, Novo Nordisk, Novogyne, Teva, and Warner Chilcott. Dr. Simon is currently the Chief Medical Officer for Sprout Pharmaceuticals.

CASE: Atypical ductal hyperplasia

Your 56-year-old, married, white patient has been on hormone therapy (HT) since age 52 for the treatment of vasomotor symptoms. She is taking a low-dose oral estrogen and micronized progesterone combination as she has an intact uterus. Her family history is positive for breast cancer, as her mother was diagnosed at age 68.

Her most recent annual screening mammogram shows linear calcifications. Because fine, linear, branching or casting calcifications are worrisome for atypical ductal hyperplasia (ADH) or ductal carcinoma in situ, a biopsy is recommended.

She elects to wean off and discontinue HT during the evaluation of her abnormal mammogram. The mammographic-guided stereotactic biopsy reveals ADH. She undergoes an open excisional biopsy, the results of which reveal extensive ADH with negative margins.

Six weeks after a lumpectomy she returns to your office reporting moderate to severe hot flashes that occur seven to 10 times per day and impair her sleep, leading to fatigue and “brain fog.” In addition, she is noticing vaginal dryness and dyspareunia despite use of lubricants. She requests treatment for her symptoms and wonders if she can restart HT systemically or vaginally.

How do you manage her hot flashes?

What are the alternatives to HT for hot flashes?

Certain lifestyle changes have been reported to provide relief for hot flushes.1 These include:

  • use of layered clothing
  • maintenance of cool ambient temperature (particularly during sleep)
  • consumption of cool foods or beverages
  • relaxation techniques (such as deep breathing, or paced respirations, for 20 min three times per day).

Despite sparse data, avoiding triggers such as spicy or hot foods or alcohol may be helpful.

Therapies such as evening primrose oil, dong quai, ginseng, wild yam, magnet therapy, reflexology, and homeopathy have not been found more effective in treating hot flashes than placebo.2

Andrew M. Kaunitz, MD:
I would add soy isoflavones, red clover, and black cohosh to this list of therapies that have not been shown to be more effective than placebo.3

Phytoestrogens (such as equol), acupuncture, yoga, and hypnosis continue to be tested in randomized trials with mixed results.

Off-label drug options offer modest help. There are currently no FDA-approved nonhormonal pharmaceutical options for relief of hot flashes; the gold standard for treatment remains estrogen therapy. For moderate to severe bothersome hot flashes, potentially effective drug therapies used off label include clonidine, selective serotonin reuptake inhibitors (SSRIs), selective norepinephrine reuptake inhibitors (SNRIs), and gabapentin (TABLE 1).2,4 In large, randomized, controlled trials, the following agents were modestly more effective than placebo: desvenlafaxine,5 low-dose paroxetine salt,6 escitalopram,7 and gastroretentive gabapentin.8 Participants in these trials included women with both spontaneous and surgically induced menopause.

Although sponsors have applied for approval for three of these agents, the FDA so far has declined to approve these agents for vasomotor treatment due to concerns about risks versus benefits. Benefits of these nonhormonal prescription therapies need to be weighed carefully against side effects, because the reduction in absolute hot flushes is modest.

 

 

Many small trials have assessed other medications and complementary and alternative therapies regarding management of menopausal symptoms. Most, however, are limited by small numbers of enrolled participants and shorter study duration (≤12 weeks). In addition, enrolled participants have variable numbers of hot flashes, often less than 14 per week.2,4

TABLE 1

Nonhormonal treatment of vasomotor symptoms

TreatmentStudy Design*Findings
Complementary/alternative medicines (black cohosh, St. John’s Wort, red clover, acupuncture, exercise)Duration: 4–52 wk; OL and RPL trials; entry criteria for most trials: >14 hot flashes/wkMixed results, mostly with no sustained improvement
SSRIs** (paroxetine, fluoxetine, sertraline, citalopram, escitalopram)Duration: 4–36 wk; RPL trials with all agents; N = 20–90 in active arms; entry criteria for most trials: >14 hot flashes/wkReduction in vasomotor symptoms (frequency, composite scores): 28%–55%
SNRIs** (venlafaxine, desvenlafaxine)Duration, 12–52 wk; RPL trials with all agents; N = 20–65 in VEN; N = 120–200 in DVS; Entry criteria >14 hot flashes/wk for VEN; >50/wk for DVSReduction in VMS (frequency, composite scores): 35%–58% for VEN, 55%–68% for DVS
Gabapentin**Duration: 4–12 wk; RPL trials; N = 20–100; entry criteria for most trials: >14–50 hot flashes/wkReduction in vasomotor symptoms (frequency, composite scores): 50%–70%
*All studies of menopausal, nondepressed women.
**Treatment is off label.
Hall E, et al. Drugs. 2011;71:287-304.
Reprinted with permission. Pinkerton JV, Shapiro M. The North American Menopause Society. Overview of available treatment options for VMS and VVA. Medscape Education Web site. http://www.medscape.org/viewarticle/763413. Published May 24, 2012. Accessed February 23, 2013.19

Can your patient restart HT? If so, should HT be offered vaginally or systemically?

Non-HT may be enough for vaginal dryness. Benefit has been shown with the use of vaginal moisturizers twice weekly and lubricants as needed for sexual activity.9 Therefore, the local application of daily lubricants, such as olive oil, along with the use of moisturizers with regular sexual intercourse may be enough to maintain vaginal health and function.

In randomized trials, phytoestrogens lack benefit data for vaginal atrophy. Small pilot studies of the effect of oral/ vaginal phytoestrogens on vaginal atrophy do not show any benefit on vaginal pH or vaginal maturation index and mixed improvement in vaginal dryness. In addition, no clear effect of these agents has been seen compared with placebo, except that there may be less progression of vaginal atrophy over time with phytoestrogens.10 It is possible that the benefits of phytoestrogens may take longer to take effect than the 12 weeks required to see an effect with HT.

Vaginal estrogen: limited safety data. No published clinical trials have assessed the impact of topical vaginal estrogen on risk of recurrence in breast cancer survivors, and concern exists because detectable estradiol levels have been reported in women who take aromatase inhibitors and have very atrophic vaginal mucosa.11 NAMS recommends that the discussion about vaginal estrogen be individualized between the patient, her provider, and her oncologist.12

Vaginal estrogen creams and tablets (Vagifem 10 μg per tablet) are often started daily for 2 weeks for a “priming dose” then dosed twice per week. To minimize systemic absorption, creams may be used externally or with smaller doses vaginally. The higher the dose or more frequent the use, the greater the risk of significant systemic absorption, particularly when the vagina is atrophic.13

Another option is the vaginal estradiol ring, which delivers a low dose (7.5 μg per day) for 90 days.14,15

James A. Simon, MD:
When you want the lowest dose of vaginal estradiol, consider Vagifem; the total dose of estrogen is lower over 90 days than with the vaginal ring.16

Progestogen therapy is generally not needed when low-dose estrogen is administered locally to treat vaginal atrophy.12

A new oral option. In February 2013, ospemifene, a selective estrogen receptor modulator (SERM), was approved for pain with intercourse and vaginal dryness.

Related article: “New treatment option for vulvar and vaginal atrophy,” by Andrew M. Kaunitz, MD (May 2013)

There is concern with systemic estrogen use

If her hot flashes remain persistent and bothersome, low-dose estrogen could be considered. However, data from the Breast Cancer Surveillance Consortium9 showed that as postmenopausal HT use decreased (from 35% to 11% between 1999 and 2005), rates of ADH decreased from a peak of 5.5 per 10,000 mammograms in 1999 to 2.4 per 10,000 mammograms in 2005. Similarly, rates of invasive cancer with ADH decreased from a peak of 4.3 per 10,000 mammograms in 2003 to 3.3 per 10,000 mammograms in 2005. This finding—that rates of ADH and invasive breast cancer were significantly linked to postmenopausal use of HT—raises concern about using HT in women with prior ADH. Of note, the cancers linked with ADH were of lower grade and stage and more estrogen receptor–positive than cancers not linked with ADH.17

 

 

How do you manage your patient’s increased risk of breast cancer?

In examining the answer to this management point, we need to first ask and answer, “How does ADH develop?” and “What is her risk of developing breast cancer?

The development of invasive breast cancer is believed to involve a complex, multistep process. Initially, there is disruption of normal cell development and growth, with overproduction of normal-looking cells (hyperplasia). These excess cells stack up and/or become abnormal. Then, there is continued change in appearance and multiplication, becoming ductal carcinoma in situ (noninvasive). If left untreated, the cells may develop into invasive cancer.18 See TABLE 2 for the relative risk of a patient with ADH developing breast cancer.10

Now, we can address, “How do you manage this patient’s increased risk of breast cancer?

TABLE 2. Relative risk of developing breast cancer

RiskRelative Risk
Atypical ductal hyperplasia4-5
Atypical ductal hyperplasia and positive family history6-8

More frequent breast screening!

  • Clinical breast exams twice per year
  • Screening mammograms annually
  • Screening tomosynthesis (These are additional digital screening views which provide almost a 3D view.)
  • Screening breast ultrasound
  • Screening breast MRI — if she has a 20% lifetime risk of breast cancer (family history or genetic predisposition) (TABLE 3).

TABLE 3. ACS guidelines for screening breast MRI

RiskRecommendation
<15% lifetime riskMRI not recommended
15% to 20% lifetime riskTalk about benefits and limitations of MRI screening
>20% lifetime riskAnnual mammogram and annual MRI alternating every 6 months

Careful consideration of medications

Since it is possible that estrogen may fuel the growth of some breast cancers, avoiding systemic menopausal HT may be safest.

Counsel her about strategies to reduce breast cancer risk

These include:

  • Lifestyle changes, including weight loss, exercise, and avoiding excess alcohol intake.
  • Preventive medications. Tamoxifen or raloxifene (Evista) can be used for 5 years. These medications block estrogen from binding to the breast estrogen receptors. Another option is an aromatase inhibitor, which decreases estrogen production.
  • Risk-reducing (prophylactic) mastectomy.

Andrew M. Kaunitz, MD:
It is important to note that both of the SERMs just mentioned (tamoxifen and raloxifene), as well as aromatase inhibitors, may cause women to experience vasomotor symptoms. SERMS also increase the risk of venous thromboembolism.

Management approach for this patient

This patient has had her ADH surgically excised. She will remain at higher risk for breast cancer and should consider strategies to decrease her risk, including lifestyle changes and the possible initiation of medications such as tamoxifen or raloxifene. New screening modalities, such as tomosynthesis or breast ultrasound, may be used to screen for breast cancer, and she may be a candidate for alternating mammograms and MRIs at 6-month intervals.

For her vaginal dryness, over-the-counter lubricants and moisturizers may be helpful. If not, topical or vaginal estrogen is available (as creams, tablets, or a ring) and provides primarily local benefit with limited systemic absorption.

For her bothersome hot flashes, if lifestyle changes don’t work, nonhormonal therapies can be offered off label, such as effexor, desvenlafaxine, gabapentin, or any of the SSRIs—including those tested in large, randomized, controlled trials, such as escitalopram and low-dose paroxetine salt, at low doses.

Andrew M. Kaunitz, MD:
Sertraline seems to be more likely to cause sweating than other SSRIs.3 For this reason, I don’t prescribe sertraline off label to treat vasomotor symptoms.

If she is taking tamoxifen, however, SSRIs such as paroxetine should be avoided due to P450 interaction.

If her hot flashes remain persistent and bothersome, low-dose estrogen could be considered, with education about the potential risks, as she is already at higher risk for breast cancer.

Acknowledgment

The author would like to thank Andrew M. Kaunitz, MD, for his forward thinking in helping to establish this new series on menopause.

DO YOU HAVE A TROUBLING CASE IN MENOPAUSE?

Suggest it to our expert panel. They may address your management dilemma in a future issue!

Email us at obg@frontlinemedcom.com or send a Letter to the Editor to rbarbieri@frontlinemedcom.com

References

1. North American Menopause Society. Treatment of menopause-associated vasomotor symptoms: position statement of The North American Menopause Society. Menopause. 2004;11(1):11-33.

2. Pinkerton JV, Stovall DW, Kightlinger RS. Advances in the treatment of menopausal symptoms. Womens Health (Lond Engl). 2009;5(4):361-384.

3. Ghaleiha A, Jahangard L, Sherafat Z, et al. Oxybutynin reduces sweating in depressed patients treated with sertraline: a double-blink, placebo-controlled, clinical study. Neuropsychiatr Dis Treat. 2012;8:407-412.

4. Hall E, Frey BN, Soares CN. Non-hormonal treatment strategies for vasomotor symptoms: a critical review. Drugs. 2011;71(3):287-304.

5. Pinkerton JV, Archer DF, Guico-Pabia CJ, Hwang E, Cheng RF. Maintenance of the efficacy of desvenlafaxine in menopausal vasomotor symptoms: a 1-year randomized controlled trial. Menopause. 2013;20(1):38-46.

6. Simon JA, Sanacora G, Bhaskar S, Lippman J. Safety and efficacy of low-dose mesylate salt of paroxetine (LDMP) for the treatment of vasomotor symptoms (VMS) associated with menopause: a 24-week randomized, placebo-controlled phase 3 study. Menopause. 2012;19(12):1371.-

7. Freeman EW, Guthrie KA, Caan B, et al. Efficacy of escitalopram for hot flashes in healthy menopausal women: a randomized controlled trial. JAMA. 2011;305(3):267-274.

8. Pinkerton JV, Kagan R, Portman D, Sathyanarayan RK, Sweeney M. Efficacy of gabapentin extended release in the treatment of menopausal hot flashes: results of the Breeze 3 study. Menopause. 2012;19(12):1377.-

9. MacBride MB, Rhodes DJ, Shuster LT. Vulvovaginal atrophy. Mayo Clin Proc. 2010;85(1):87-94.

10. Bedell S, Nachtigall M, Naftolin F. The pros and cons of plant estrogens for menopause [published online ahead of print December 25 2012]. J Steroid Biochem Mol Biol. 2012. doi: 10.1016/j.jsbmb.2012.12.004.

11. Moegele M, Buchholz S, Seitz S, Ortmann O. Vaginal estrogen therapy in postmenopausal breast cancer patients treated with aromatase inhibitors. Arch Gynecol Obstet. 2012;285(5):1397-402.

12. North American Menopause Society. The role of local vaginal estrogen for treatment of vaginal atrophy in postmenopausal women: 2007 position statement of The North American Menopause Society. Menopause. 2007;14(3 Pt 1):355-371.

13. Archer DF. Efficacy and tolerability of local estrogen therapy for urogenital atrophy. Menopause. 2010;17(1):194-203.

14. Simon JA. Vulvovaginal atrophy: new and upcoming approaches. Menopause. 2009;16(1):5-7.

15. Kaunitz AM. Transdermal and vaginal estradiol for the treatment of menopausal symptoms: the nuts and bolts. Menopause. 2012;19(6):602-603.

16. Pruthi S, Simon JA, Early AP. Current overview of the management of urogenital atrophy in women with breast cancer. Breast J. 2011;17(4):403-408.

17. Menes TS, Kerlikowske K, Jaffer S, Seger D, Miglioretti D. Rates of atypical ductal hyperplasia declined with less use of postmenopausal hormone treatment: findings from the Breast Cancer Surveillance Consortium. Cancer Epidemiol Biomarkers Prev. 2009;18(11):2822-2828.

18. Santen RJ, Mansel R. Benign breast disorders. N Engl J Med. 2005;353(3):275-285.

19. Pinkerton JV, Shapiro M. The North American Menopause Society. Overview of available treatment options for VMS and VVA. Medscape Education Web site. http://www.medscape.org/viewarticle/763413. Published May 24 2012. Accessed February 23, 2013.

Article PDF
Author and Disclosure Information

JoAnn V. Pinkerton, MD

Dr. Andrew M. Kaunitz and Dr. James A Simon provided peer review and comments for Dr. Pinkerton’s case study.

Dr. Pinkerton discusses her approach to screening patients who are at increased risk for breast cancer

Issue
OBG Management - 25(5)
Publications
Topics
Page Number
36-41
Legacy Keywords
JoAnn V. Pinkerton MD;Andrew M. Kaunitz MD;James A. Simon MD;cases in menopause;atypical hyperplasia;menopausal symptoms;hot flashes;mammogram;ADH;stereotactic biopsy;vaginal dryness;dyspareunia;vaginal lubricants;hormone therapy;lifestyle changes;phytoestrogens;nonhormonal therapy;SNRIs;SSRIs;black cohosh;St. John’s Wort;red clover;gabapentin;vaginal estrogen;desvenlafaxine;vaginal estradiol;Vagifem;vaginal ring;vaginal moisturizer;paroxetine;fluoxetine;sertraline;citalopram;escitalopram;
Sections
Author and Disclosure Information

JoAnn V. Pinkerton, MD

Dr. Andrew M. Kaunitz and Dr. James A Simon provided peer review and comments for Dr. Pinkerton’s case study.

Dr. Pinkerton discusses her approach to screening patients who are at increased risk for breast cancer

Author and Disclosure Information

JoAnn V. Pinkerton, MD

Dr. Andrew M. Kaunitz and Dr. James A Simon provided peer review and comments for Dr. Pinkerton’s case study.

Dr. Pinkerton discusses her approach to screening patients who are at increased risk for breast cancer

Article PDF
Article PDF

CASES IN MENOPAUSE

A new series brought to you by the menopause experts

Andrew M. Kaunitz, MD
Professor and Associate Chairman, Department of Obstetrics and Gynecology, University of Florida College of Medicine–Jacksonville. Dr. Kaunitz is a NAMS Board member and certified menopause practitioner. He also serves on the OBG Management Board of Editors.
JoAnn V. Pinkerton, MD
Professor, Department of Obstetrics and Gynecology, and Director, Division of Midlife Women’s Health, University of Virginia. Dr. Pinkerton is a North American Menopause Society (NAMS) past president and certified menopause practitioner. She also serves on the OBG Management Board of Editors.
James A. Simon, MD
Clinical Professor, Department of Obstetrics and Gynecology, George Washington University, and Medical Director, Women’s Health & Research Consultants, Washington, DC. Dr. Simon is a NAMS past president, a certified menopause practitioner, and a certified clinical densitometrist. He also serves on the OBG Management Board of Editors.

Disclosures
Dr. Kaunitz reports that his institution receives grant or research support from Agile, Bayer, Endoceutics, Teva, Medical Diagnostic Laboratories, and Noven, and that he is a consultant to Bayer, Merck, and Teva.

Dr. Pinkerton reports that her institution receives consulting fees from Pfizer, DepoMed, Shionogi, and Noven and multicenter research fees from DepoMed, Endoceutics, and Bionova.

Dr. Simon reports being a consultant to or on the advisory boards of Abbott Laboratories, Agile Therapeutics, Amgen, Ascend Therapeutics, BioSante, Depomed, Lelo, MD Therapeutics, Meda Pharmaceuticals, Merck, Noven, Novo Nordisk, Novogyne, Pfizer, Shionogi, Shippan Point Advisors LLC, Slate Pharmaceuticals, Sprout Pharmaceuticals, Teva, Warner Chilcott, and Watson. He also reports receiving (currently or in the past year) grant/research support from BioSante, EndoCeutics, Novo Nordisk, Novogyne, Palatin Technologies, Teva, and Warner Chilcott. He reports serving on the speakers bureaus of Amgen, Merck, Novartis, Noven, Novo Nordisk, Novogyne, Teva, and Warner Chilcott. Dr. Simon is currently the Chief Medical Officer for Sprout Pharmaceuticals.

CASE: Atypical ductal hyperplasia

Your 56-year-old, married, white patient has been on hormone therapy (HT) since age 52 for the treatment of vasomotor symptoms. She is taking a low-dose oral estrogen and micronized progesterone combination as she has an intact uterus. Her family history is positive for breast cancer, as her mother was diagnosed at age 68.

Her most recent annual screening mammogram shows linear calcifications. Because fine, linear, branching or casting calcifications are worrisome for atypical ductal hyperplasia (ADH) or ductal carcinoma in situ, a biopsy is recommended.

She elects to wean off and discontinue HT during the evaluation of her abnormal mammogram. The mammographic-guided stereotactic biopsy reveals ADH. She undergoes an open excisional biopsy, the results of which reveal extensive ADH with negative margins.

Six weeks after a lumpectomy she returns to your office reporting moderate to severe hot flashes that occur seven to 10 times per day and impair her sleep, leading to fatigue and “brain fog.” In addition, she is noticing vaginal dryness and dyspareunia despite use of lubricants. She requests treatment for her symptoms and wonders if she can restart HT systemically or vaginally.

How do you manage her hot flashes?

What are the alternatives to HT for hot flashes?

Certain lifestyle changes have been reported to provide relief for hot flushes.1 These include:

  • use of layered clothing
  • maintenance of cool ambient temperature (particularly during sleep)
  • consumption of cool foods or beverages
  • relaxation techniques (such as deep breathing, or paced respirations, for 20 min three times per day).

Despite sparse data, avoiding triggers such as spicy or hot foods or alcohol may be helpful.

Therapies such as evening primrose oil, dong quai, ginseng, wild yam, magnet therapy, reflexology, and homeopathy have not been found more effective in treating hot flashes than placebo.2

Andrew M. Kaunitz, MD:
I would add soy isoflavones, red clover, and black cohosh to this list of therapies that have not been shown to be more effective than placebo.3

Phytoestrogens (such as equol), acupuncture, yoga, and hypnosis continue to be tested in randomized trials with mixed results.

Off-label drug options offer modest help. There are currently no FDA-approved nonhormonal pharmaceutical options for relief of hot flashes; the gold standard for treatment remains estrogen therapy. For moderate to severe bothersome hot flashes, potentially effective drug therapies used off label include clonidine, selective serotonin reuptake inhibitors (SSRIs), selective norepinephrine reuptake inhibitors (SNRIs), and gabapentin (TABLE 1).2,4 In large, randomized, controlled trials, the following agents were modestly more effective than placebo: desvenlafaxine,5 low-dose paroxetine salt,6 escitalopram,7 and gastroretentive gabapentin.8 Participants in these trials included women with both spontaneous and surgically induced menopause.

Although sponsors have applied for approval for three of these agents, the FDA so far has declined to approve these agents for vasomotor treatment due to concerns about risks versus benefits. Benefits of these nonhormonal prescription therapies need to be weighed carefully against side effects, because the reduction in absolute hot flushes is modest.

 

 

Many small trials have assessed other medications and complementary and alternative therapies regarding management of menopausal symptoms. Most, however, are limited by small numbers of enrolled participants and shorter study duration (≤12 weeks). In addition, enrolled participants have variable numbers of hot flashes, often less than 14 per week.2,4

TABLE 1

Nonhormonal treatment of vasomotor symptoms

TreatmentStudy Design*Findings
Complementary/alternative medicines (black cohosh, St. John’s Wort, red clover, acupuncture, exercise)Duration: 4–52 wk; OL and RPL trials; entry criteria for most trials: >14 hot flashes/wkMixed results, mostly with no sustained improvement
SSRIs** (paroxetine, fluoxetine, sertraline, citalopram, escitalopram)Duration: 4–36 wk; RPL trials with all agents; N = 20–90 in active arms; entry criteria for most trials: >14 hot flashes/wkReduction in vasomotor symptoms (frequency, composite scores): 28%–55%
SNRIs** (venlafaxine, desvenlafaxine)Duration, 12–52 wk; RPL trials with all agents; N = 20–65 in VEN; N = 120–200 in DVS; Entry criteria >14 hot flashes/wk for VEN; >50/wk for DVSReduction in VMS (frequency, composite scores): 35%–58% for VEN, 55%–68% for DVS
Gabapentin**Duration: 4–12 wk; RPL trials; N = 20–100; entry criteria for most trials: >14–50 hot flashes/wkReduction in vasomotor symptoms (frequency, composite scores): 50%–70%
*All studies of menopausal, nondepressed women.
**Treatment is off label.
Hall E, et al. Drugs. 2011;71:287-304.
Reprinted with permission. Pinkerton JV, Shapiro M. The North American Menopause Society. Overview of available treatment options for VMS and VVA. Medscape Education Web site. http://www.medscape.org/viewarticle/763413. Published May 24, 2012. Accessed February 23, 2013.19

Can your patient restart HT? If so, should HT be offered vaginally or systemically?

Non-HT may be enough for vaginal dryness. Benefit has been shown with the use of vaginal moisturizers twice weekly and lubricants as needed for sexual activity.9 Therefore, the local application of daily lubricants, such as olive oil, along with the use of moisturizers with regular sexual intercourse may be enough to maintain vaginal health and function.

In randomized trials, phytoestrogens lack benefit data for vaginal atrophy. Small pilot studies of the effect of oral/ vaginal phytoestrogens on vaginal atrophy do not show any benefit on vaginal pH or vaginal maturation index and mixed improvement in vaginal dryness. In addition, no clear effect of these agents has been seen compared with placebo, except that there may be less progression of vaginal atrophy over time with phytoestrogens.10 It is possible that the benefits of phytoestrogens may take longer to take effect than the 12 weeks required to see an effect with HT.

Vaginal estrogen: limited safety data. No published clinical trials have assessed the impact of topical vaginal estrogen on risk of recurrence in breast cancer survivors, and concern exists because detectable estradiol levels have been reported in women who take aromatase inhibitors and have very atrophic vaginal mucosa.11 NAMS recommends that the discussion about vaginal estrogen be individualized between the patient, her provider, and her oncologist.12

Vaginal estrogen creams and tablets (Vagifem 10 μg per tablet) are often started daily for 2 weeks for a “priming dose” then dosed twice per week. To minimize systemic absorption, creams may be used externally or with smaller doses vaginally. The higher the dose or more frequent the use, the greater the risk of significant systemic absorption, particularly when the vagina is atrophic.13

Another option is the vaginal estradiol ring, which delivers a low dose (7.5 μg per day) for 90 days.14,15

James A. Simon, MD:
When you want the lowest dose of vaginal estradiol, consider Vagifem; the total dose of estrogen is lower over 90 days than with the vaginal ring.16

Progestogen therapy is generally not needed when low-dose estrogen is administered locally to treat vaginal atrophy.12

A new oral option. In February 2013, ospemifene, a selective estrogen receptor modulator (SERM), was approved for pain with intercourse and vaginal dryness.

Related article: “New treatment option for vulvar and vaginal atrophy,” by Andrew M. Kaunitz, MD (May 2013)

There is concern with systemic estrogen use

If her hot flashes remain persistent and bothersome, low-dose estrogen could be considered. However, data from the Breast Cancer Surveillance Consortium9 showed that as postmenopausal HT use decreased (from 35% to 11% between 1999 and 2005), rates of ADH decreased from a peak of 5.5 per 10,000 mammograms in 1999 to 2.4 per 10,000 mammograms in 2005. Similarly, rates of invasive cancer with ADH decreased from a peak of 4.3 per 10,000 mammograms in 2003 to 3.3 per 10,000 mammograms in 2005. This finding—that rates of ADH and invasive breast cancer were significantly linked to postmenopausal use of HT—raises concern about using HT in women with prior ADH. Of note, the cancers linked with ADH were of lower grade and stage and more estrogen receptor–positive than cancers not linked with ADH.17

 

 

How do you manage your patient’s increased risk of breast cancer?

In examining the answer to this management point, we need to first ask and answer, “How does ADH develop?” and “What is her risk of developing breast cancer?

The development of invasive breast cancer is believed to involve a complex, multistep process. Initially, there is disruption of normal cell development and growth, with overproduction of normal-looking cells (hyperplasia). These excess cells stack up and/or become abnormal. Then, there is continued change in appearance and multiplication, becoming ductal carcinoma in situ (noninvasive). If left untreated, the cells may develop into invasive cancer.18 See TABLE 2 for the relative risk of a patient with ADH developing breast cancer.10

Now, we can address, “How do you manage this patient’s increased risk of breast cancer?

TABLE 2. Relative risk of developing breast cancer

RiskRelative Risk
Atypical ductal hyperplasia4-5
Atypical ductal hyperplasia and positive family history6-8

More frequent breast screening!

  • Clinical breast exams twice per year
  • Screening mammograms annually
  • Screening tomosynthesis (These are additional digital screening views which provide almost a 3D view.)
  • Screening breast ultrasound
  • Screening breast MRI — if she has a 20% lifetime risk of breast cancer (family history or genetic predisposition) (TABLE 3).

TABLE 3. ACS guidelines for screening breast MRI

RiskRecommendation
<15% lifetime riskMRI not recommended
15% to 20% lifetime riskTalk about benefits and limitations of MRI screening
>20% lifetime riskAnnual mammogram and annual MRI alternating every 6 months

Careful consideration of medications

Since it is possible that estrogen may fuel the growth of some breast cancers, avoiding systemic menopausal HT may be safest.

Counsel her about strategies to reduce breast cancer risk

These include:

  • Lifestyle changes, including weight loss, exercise, and avoiding excess alcohol intake.
  • Preventive medications. Tamoxifen or raloxifene (Evista) can be used for 5 years. These medications block estrogen from binding to the breast estrogen receptors. Another option is an aromatase inhibitor, which decreases estrogen production.
  • Risk-reducing (prophylactic) mastectomy.

Andrew M. Kaunitz, MD:
It is important to note that both of the SERMs just mentioned (tamoxifen and raloxifene), as well as aromatase inhibitors, may cause women to experience vasomotor symptoms. SERMS also increase the risk of venous thromboembolism.

Management approach for this patient

This patient has had her ADH surgically excised. She will remain at higher risk for breast cancer and should consider strategies to decrease her risk, including lifestyle changes and the possible initiation of medications such as tamoxifen or raloxifene. New screening modalities, such as tomosynthesis or breast ultrasound, may be used to screen for breast cancer, and she may be a candidate for alternating mammograms and MRIs at 6-month intervals.

For her vaginal dryness, over-the-counter lubricants and moisturizers may be helpful. If not, topical or vaginal estrogen is available (as creams, tablets, or a ring) and provides primarily local benefit with limited systemic absorption.

For her bothersome hot flashes, if lifestyle changes don’t work, nonhormonal therapies can be offered off label, such as effexor, desvenlafaxine, gabapentin, or any of the SSRIs—including those tested in large, randomized, controlled trials, such as escitalopram and low-dose paroxetine salt, at low doses.

Andrew M. Kaunitz, MD:
Sertraline seems to be more likely to cause sweating than other SSRIs.3 For this reason, I don’t prescribe sertraline off label to treat vasomotor symptoms.

If she is taking tamoxifen, however, SSRIs such as paroxetine should be avoided due to P450 interaction.

If her hot flashes remain persistent and bothersome, low-dose estrogen could be considered, with education about the potential risks, as she is already at higher risk for breast cancer.

Acknowledgment

The author would like to thank Andrew M. Kaunitz, MD, for his forward thinking in helping to establish this new series on menopause.

DO YOU HAVE A TROUBLING CASE IN MENOPAUSE?

Suggest it to our expert panel. They may address your management dilemma in a future issue!

Email us at obg@frontlinemedcom.com or send a Letter to the Editor to rbarbieri@frontlinemedcom.com

CASES IN MENOPAUSE

A new series brought to you by the menopause experts

Andrew M. Kaunitz, MD
Professor and Associate Chairman, Department of Obstetrics and Gynecology, University of Florida College of Medicine–Jacksonville. Dr. Kaunitz is a NAMS Board member and certified menopause practitioner. He also serves on the OBG Management Board of Editors.
JoAnn V. Pinkerton, MD
Professor, Department of Obstetrics and Gynecology, and Director, Division of Midlife Women’s Health, University of Virginia. Dr. Pinkerton is a North American Menopause Society (NAMS) past president and certified menopause practitioner. She also serves on the OBG Management Board of Editors.
James A. Simon, MD
Clinical Professor, Department of Obstetrics and Gynecology, George Washington University, and Medical Director, Women’s Health & Research Consultants, Washington, DC. Dr. Simon is a NAMS past president, a certified menopause practitioner, and a certified clinical densitometrist. He also serves on the OBG Management Board of Editors.

Disclosures
Dr. Kaunitz reports that his institution receives grant or research support from Agile, Bayer, Endoceutics, Teva, Medical Diagnostic Laboratories, and Noven, and that he is a consultant to Bayer, Merck, and Teva.

Dr. Pinkerton reports that her institution receives consulting fees from Pfizer, DepoMed, Shionogi, and Noven and multicenter research fees from DepoMed, Endoceutics, and Bionova.

Dr. Simon reports being a consultant to or on the advisory boards of Abbott Laboratories, Agile Therapeutics, Amgen, Ascend Therapeutics, BioSante, Depomed, Lelo, MD Therapeutics, Meda Pharmaceuticals, Merck, Noven, Novo Nordisk, Novogyne, Pfizer, Shionogi, Shippan Point Advisors LLC, Slate Pharmaceuticals, Sprout Pharmaceuticals, Teva, Warner Chilcott, and Watson. He also reports receiving (currently or in the past year) grant/research support from BioSante, EndoCeutics, Novo Nordisk, Novogyne, Palatin Technologies, Teva, and Warner Chilcott. He reports serving on the speakers bureaus of Amgen, Merck, Novartis, Noven, Novo Nordisk, Novogyne, Teva, and Warner Chilcott. Dr. Simon is currently the Chief Medical Officer for Sprout Pharmaceuticals.

CASE: Atypical ductal hyperplasia

Your 56-year-old, married, white patient has been on hormone therapy (HT) since age 52 for the treatment of vasomotor symptoms. She is taking a low-dose oral estrogen and micronized progesterone combination as she has an intact uterus. Her family history is positive for breast cancer, as her mother was diagnosed at age 68.

Her most recent annual screening mammogram shows linear calcifications. Because fine, linear, branching or casting calcifications are worrisome for atypical ductal hyperplasia (ADH) or ductal carcinoma in situ, a biopsy is recommended.

She elects to wean off and discontinue HT during the evaluation of her abnormal mammogram. The mammographic-guided stereotactic biopsy reveals ADH. She undergoes an open excisional biopsy, the results of which reveal extensive ADH with negative margins.

Six weeks after a lumpectomy she returns to your office reporting moderate to severe hot flashes that occur seven to 10 times per day and impair her sleep, leading to fatigue and “brain fog.” In addition, she is noticing vaginal dryness and dyspareunia despite use of lubricants. She requests treatment for her symptoms and wonders if she can restart HT systemically or vaginally.

How do you manage her hot flashes?

What are the alternatives to HT for hot flashes?

Certain lifestyle changes have been reported to provide relief for hot flushes.1 These include:

  • use of layered clothing
  • maintenance of cool ambient temperature (particularly during sleep)
  • consumption of cool foods or beverages
  • relaxation techniques (such as deep breathing, or paced respirations, for 20 min three times per day).

Despite sparse data, avoiding triggers such as spicy or hot foods or alcohol may be helpful.

Therapies such as evening primrose oil, dong quai, ginseng, wild yam, magnet therapy, reflexology, and homeopathy have not been found more effective in treating hot flashes than placebo.2

Andrew M. Kaunitz, MD:
I would add soy isoflavones, red clover, and black cohosh to this list of therapies that have not been shown to be more effective than placebo.3

Phytoestrogens (such as equol), acupuncture, yoga, and hypnosis continue to be tested in randomized trials with mixed results.

Off-label drug options offer modest help. There are currently no FDA-approved nonhormonal pharmaceutical options for relief of hot flashes; the gold standard for treatment remains estrogen therapy. For moderate to severe bothersome hot flashes, potentially effective drug therapies used off label include clonidine, selective serotonin reuptake inhibitors (SSRIs), selective norepinephrine reuptake inhibitors (SNRIs), and gabapentin (TABLE 1).2,4 In large, randomized, controlled trials, the following agents were modestly more effective than placebo: desvenlafaxine,5 low-dose paroxetine salt,6 escitalopram,7 and gastroretentive gabapentin.8 Participants in these trials included women with both spontaneous and surgically induced menopause.

Although sponsors have applied for approval for three of these agents, the FDA so far has declined to approve these agents for vasomotor treatment due to concerns about risks versus benefits. Benefits of these nonhormonal prescription therapies need to be weighed carefully against side effects, because the reduction in absolute hot flushes is modest.

 

 

Many small trials have assessed other medications and complementary and alternative therapies regarding management of menopausal symptoms. Most, however, are limited by small numbers of enrolled participants and shorter study duration (≤12 weeks). In addition, enrolled participants have variable numbers of hot flashes, often less than 14 per week.2,4

TABLE 1

Nonhormonal treatment of vasomotor symptoms

TreatmentStudy Design*Findings
Complementary/alternative medicines (black cohosh, St. John’s Wort, red clover, acupuncture, exercise)Duration: 4–52 wk; OL and RPL trials; entry criteria for most trials: >14 hot flashes/wkMixed results, mostly with no sustained improvement
SSRIs** (paroxetine, fluoxetine, sertraline, citalopram, escitalopram)Duration: 4–36 wk; RPL trials with all agents; N = 20–90 in active arms; entry criteria for most trials: >14 hot flashes/wkReduction in vasomotor symptoms (frequency, composite scores): 28%–55%
SNRIs** (venlafaxine, desvenlafaxine)Duration, 12–52 wk; RPL trials with all agents; N = 20–65 in VEN; N = 120–200 in DVS; Entry criteria >14 hot flashes/wk for VEN; >50/wk for DVSReduction in VMS (frequency, composite scores): 35%–58% for VEN, 55%–68% for DVS
Gabapentin**Duration: 4–12 wk; RPL trials; N = 20–100; entry criteria for most trials: >14–50 hot flashes/wkReduction in vasomotor symptoms (frequency, composite scores): 50%–70%
*All studies of menopausal, nondepressed women.
**Treatment is off label.
Hall E, et al. Drugs. 2011;71:287-304.
Reprinted with permission. Pinkerton JV, Shapiro M. The North American Menopause Society. Overview of available treatment options for VMS and VVA. Medscape Education Web site. http://www.medscape.org/viewarticle/763413. Published May 24, 2012. Accessed February 23, 2013.19

Can your patient restart HT? If so, should HT be offered vaginally or systemically?

Non-HT may be enough for vaginal dryness. Benefit has been shown with the use of vaginal moisturizers twice weekly and lubricants as needed for sexual activity.9 Therefore, the local application of daily lubricants, such as olive oil, along with the use of moisturizers with regular sexual intercourse may be enough to maintain vaginal health and function.

In randomized trials, phytoestrogens lack benefit data for vaginal atrophy. Small pilot studies of the effect of oral/ vaginal phytoestrogens on vaginal atrophy do not show any benefit on vaginal pH or vaginal maturation index and mixed improvement in vaginal dryness. In addition, no clear effect of these agents has been seen compared with placebo, except that there may be less progression of vaginal atrophy over time with phytoestrogens.10 It is possible that the benefits of phytoestrogens may take longer to take effect than the 12 weeks required to see an effect with HT.

Vaginal estrogen: limited safety data. No published clinical trials have assessed the impact of topical vaginal estrogen on risk of recurrence in breast cancer survivors, and concern exists because detectable estradiol levels have been reported in women who take aromatase inhibitors and have very atrophic vaginal mucosa.11 NAMS recommends that the discussion about vaginal estrogen be individualized between the patient, her provider, and her oncologist.12

Vaginal estrogen creams and tablets (Vagifem 10 μg per tablet) are often started daily for 2 weeks for a “priming dose” then dosed twice per week. To minimize systemic absorption, creams may be used externally or with smaller doses vaginally. The higher the dose or more frequent the use, the greater the risk of significant systemic absorption, particularly when the vagina is atrophic.13

Another option is the vaginal estradiol ring, which delivers a low dose (7.5 μg per day) for 90 days.14,15

James A. Simon, MD:
When you want the lowest dose of vaginal estradiol, consider Vagifem; the total dose of estrogen is lower over 90 days than with the vaginal ring.16

Progestogen therapy is generally not needed when low-dose estrogen is administered locally to treat vaginal atrophy.12

A new oral option. In February 2013, ospemifene, a selective estrogen receptor modulator (SERM), was approved for pain with intercourse and vaginal dryness.

Related article: “New treatment option for vulvar and vaginal atrophy,” by Andrew M. Kaunitz, MD (May 2013)

There is concern with systemic estrogen use

If her hot flashes remain persistent and bothersome, low-dose estrogen could be considered. However, data from the Breast Cancer Surveillance Consortium9 showed that as postmenopausal HT use decreased (from 35% to 11% between 1999 and 2005), rates of ADH decreased from a peak of 5.5 per 10,000 mammograms in 1999 to 2.4 per 10,000 mammograms in 2005. Similarly, rates of invasive cancer with ADH decreased from a peak of 4.3 per 10,000 mammograms in 2003 to 3.3 per 10,000 mammograms in 2005. This finding—that rates of ADH and invasive breast cancer were significantly linked to postmenopausal use of HT—raises concern about using HT in women with prior ADH. Of note, the cancers linked with ADH were of lower grade and stage and more estrogen receptor–positive than cancers not linked with ADH.17

 

 

How do you manage your patient’s increased risk of breast cancer?

In examining the answer to this management point, we need to first ask and answer, “How does ADH develop?” and “What is her risk of developing breast cancer?

The development of invasive breast cancer is believed to involve a complex, multistep process. Initially, there is disruption of normal cell development and growth, with overproduction of normal-looking cells (hyperplasia). These excess cells stack up and/or become abnormal. Then, there is continued change in appearance and multiplication, becoming ductal carcinoma in situ (noninvasive). If left untreated, the cells may develop into invasive cancer.18 See TABLE 2 for the relative risk of a patient with ADH developing breast cancer.10

Now, we can address, “How do you manage this patient’s increased risk of breast cancer?

TABLE 2. Relative risk of developing breast cancer

RiskRelative Risk
Atypical ductal hyperplasia4-5
Atypical ductal hyperplasia and positive family history6-8

More frequent breast screening!

  • Clinical breast exams twice per year
  • Screening mammograms annually
  • Screening tomosynthesis (These are additional digital screening views which provide almost a 3D view.)
  • Screening breast ultrasound
  • Screening breast MRI — if she has a 20% lifetime risk of breast cancer (family history or genetic predisposition) (TABLE 3).

TABLE 3. ACS guidelines for screening breast MRI

RiskRecommendation
<15% lifetime riskMRI not recommended
15% to 20% lifetime riskTalk about benefits and limitations of MRI screening
>20% lifetime riskAnnual mammogram and annual MRI alternating every 6 months

Careful consideration of medications

Since it is possible that estrogen may fuel the growth of some breast cancers, avoiding systemic menopausal HT may be safest.

Counsel her about strategies to reduce breast cancer risk

These include:

  • Lifestyle changes, including weight loss, exercise, and avoiding excess alcohol intake.
  • Preventive medications. Tamoxifen or raloxifene (Evista) can be used for 5 years. These medications block estrogen from binding to the breast estrogen receptors. Another option is an aromatase inhibitor, which decreases estrogen production.
  • Risk-reducing (prophylactic) mastectomy.

Andrew M. Kaunitz, MD:
It is important to note that both of the SERMs just mentioned (tamoxifen and raloxifene), as well as aromatase inhibitors, may cause women to experience vasomotor symptoms. SERMS also increase the risk of venous thromboembolism.

Management approach for this patient

This patient has had her ADH surgically excised. She will remain at higher risk for breast cancer and should consider strategies to decrease her risk, including lifestyle changes and the possible initiation of medications such as tamoxifen or raloxifene. New screening modalities, such as tomosynthesis or breast ultrasound, may be used to screen for breast cancer, and she may be a candidate for alternating mammograms and MRIs at 6-month intervals.

For her vaginal dryness, over-the-counter lubricants and moisturizers may be helpful. If not, topical or vaginal estrogen is available (as creams, tablets, or a ring) and provides primarily local benefit with limited systemic absorption.

For her bothersome hot flashes, if lifestyle changes don’t work, nonhormonal therapies can be offered off label, such as effexor, desvenlafaxine, gabapentin, or any of the SSRIs—including those tested in large, randomized, controlled trials, such as escitalopram and low-dose paroxetine salt, at low doses.

Andrew M. Kaunitz, MD:
Sertraline seems to be more likely to cause sweating than other SSRIs.3 For this reason, I don’t prescribe sertraline off label to treat vasomotor symptoms.

If she is taking tamoxifen, however, SSRIs such as paroxetine should be avoided due to P450 interaction.

If her hot flashes remain persistent and bothersome, low-dose estrogen could be considered, with education about the potential risks, as she is already at higher risk for breast cancer.

Acknowledgment

The author would like to thank Andrew M. Kaunitz, MD, for his forward thinking in helping to establish this new series on menopause.

DO YOU HAVE A TROUBLING CASE IN MENOPAUSE?

Suggest it to our expert panel. They may address your management dilemma in a future issue!

Email us at obg@frontlinemedcom.com or send a Letter to the Editor to rbarbieri@frontlinemedcom.com

References

1. North American Menopause Society. Treatment of menopause-associated vasomotor symptoms: position statement of The North American Menopause Society. Menopause. 2004;11(1):11-33.

2. Pinkerton JV, Stovall DW, Kightlinger RS. Advances in the treatment of menopausal symptoms. Womens Health (Lond Engl). 2009;5(4):361-384.

3. Ghaleiha A, Jahangard L, Sherafat Z, et al. Oxybutynin reduces sweating in depressed patients treated with sertraline: a double-blink, placebo-controlled, clinical study. Neuropsychiatr Dis Treat. 2012;8:407-412.

4. Hall E, Frey BN, Soares CN. Non-hormonal treatment strategies for vasomotor symptoms: a critical review. Drugs. 2011;71(3):287-304.

5. Pinkerton JV, Archer DF, Guico-Pabia CJ, Hwang E, Cheng RF. Maintenance of the efficacy of desvenlafaxine in menopausal vasomotor symptoms: a 1-year randomized controlled trial. Menopause. 2013;20(1):38-46.

6. Simon JA, Sanacora G, Bhaskar S, Lippman J. Safety and efficacy of low-dose mesylate salt of paroxetine (LDMP) for the treatment of vasomotor symptoms (VMS) associated with menopause: a 24-week randomized, placebo-controlled phase 3 study. Menopause. 2012;19(12):1371.-

7. Freeman EW, Guthrie KA, Caan B, et al. Efficacy of escitalopram for hot flashes in healthy menopausal women: a randomized controlled trial. JAMA. 2011;305(3):267-274.

8. Pinkerton JV, Kagan R, Portman D, Sathyanarayan RK, Sweeney M. Efficacy of gabapentin extended release in the treatment of menopausal hot flashes: results of the Breeze 3 study. Menopause. 2012;19(12):1377.-

9. MacBride MB, Rhodes DJ, Shuster LT. Vulvovaginal atrophy. Mayo Clin Proc. 2010;85(1):87-94.

10. Bedell S, Nachtigall M, Naftolin F. The pros and cons of plant estrogens for menopause [published online ahead of print December 25 2012]. J Steroid Biochem Mol Biol. 2012. doi: 10.1016/j.jsbmb.2012.12.004.

11. Moegele M, Buchholz S, Seitz S, Ortmann O. Vaginal estrogen therapy in postmenopausal breast cancer patients treated with aromatase inhibitors. Arch Gynecol Obstet. 2012;285(5):1397-402.

12. North American Menopause Society. The role of local vaginal estrogen for treatment of vaginal atrophy in postmenopausal women: 2007 position statement of The North American Menopause Society. Menopause. 2007;14(3 Pt 1):355-371.

13. Archer DF. Efficacy and tolerability of local estrogen therapy for urogenital atrophy. Menopause. 2010;17(1):194-203.

14. Simon JA. Vulvovaginal atrophy: new and upcoming approaches. Menopause. 2009;16(1):5-7.

15. Kaunitz AM. Transdermal and vaginal estradiol for the treatment of menopausal symptoms: the nuts and bolts. Menopause. 2012;19(6):602-603.

16. Pruthi S, Simon JA, Early AP. Current overview of the management of urogenital atrophy in women with breast cancer. Breast J. 2011;17(4):403-408.

17. Menes TS, Kerlikowske K, Jaffer S, Seger D, Miglioretti D. Rates of atypical ductal hyperplasia declined with less use of postmenopausal hormone treatment: findings from the Breast Cancer Surveillance Consortium. Cancer Epidemiol Biomarkers Prev. 2009;18(11):2822-2828.

18. Santen RJ, Mansel R. Benign breast disorders. N Engl J Med. 2005;353(3):275-285.

19. Pinkerton JV, Shapiro M. The North American Menopause Society. Overview of available treatment options for VMS and VVA. Medscape Education Web site. http://www.medscape.org/viewarticle/763413. Published May 24 2012. Accessed February 23, 2013.

References

1. North American Menopause Society. Treatment of menopause-associated vasomotor symptoms: position statement of The North American Menopause Society. Menopause. 2004;11(1):11-33.

2. Pinkerton JV, Stovall DW, Kightlinger RS. Advances in the treatment of menopausal symptoms. Womens Health (Lond Engl). 2009;5(4):361-384.

3. Ghaleiha A, Jahangard L, Sherafat Z, et al. Oxybutynin reduces sweating in depressed patients treated with sertraline: a double-blink, placebo-controlled, clinical study. Neuropsychiatr Dis Treat. 2012;8:407-412.

4. Hall E, Frey BN, Soares CN. Non-hormonal treatment strategies for vasomotor symptoms: a critical review. Drugs. 2011;71(3):287-304.

5. Pinkerton JV, Archer DF, Guico-Pabia CJ, Hwang E, Cheng RF. Maintenance of the efficacy of desvenlafaxine in menopausal vasomotor symptoms: a 1-year randomized controlled trial. Menopause. 2013;20(1):38-46.

6. Simon JA, Sanacora G, Bhaskar S, Lippman J. Safety and efficacy of low-dose mesylate salt of paroxetine (LDMP) for the treatment of vasomotor symptoms (VMS) associated with menopause: a 24-week randomized, placebo-controlled phase 3 study. Menopause. 2012;19(12):1371.-

7. Freeman EW, Guthrie KA, Caan B, et al. Efficacy of escitalopram for hot flashes in healthy menopausal women: a randomized controlled trial. JAMA. 2011;305(3):267-274.

8. Pinkerton JV, Kagan R, Portman D, Sathyanarayan RK, Sweeney M. Efficacy of gabapentin extended release in the treatment of menopausal hot flashes: results of the Breeze 3 study. Menopause. 2012;19(12):1377.-

9. MacBride MB, Rhodes DJ, Shuster LT. Vulvovaginal atrophy. Mayo Clin Proc. 2010;85(1):87-94.

10. Bedell S, Nachtigall M, Naftolin F. The pros and cons of plant estrogens for menopause [published online ahead of print December 25 2012]. J Steroid Biochem Mol Biol. 2012. doi: 10.1016/j.jsbmb.2012.12.004.

11. Moegele M, Buchholz S, Seitz S, Ortmann O. Vaginal estrogen therapy in postmenopausal breast cancer patients treated with aromatase inhibitors. Arch Gynecol Obstet. 2012;285(5):1397-402.

12. North American Menopause Society. The role of local vaginal estrogen for treatment of vaginal atrophy in postmenopausal women: 2007 position statement of The North American Menopause Society. Menopause. 2007;14(3 Pt 1):355-371.

13. Archer DF. Efficacy and tolerability of local estrogen therapy for urogenital atrophy. Menopause. 2010;17(1):194-203.

14. Simon JA. Vulvovaginal atrophy: new and upcoming approaches. Menopause. 2009;16(1):5-7.

15. Kaunitz AM. Transdermal and vaginal estradiol for the treatment of menopausal symptoms: the nuts and bolts. Menopause. 2012;19(6):602-603.

16. Pruthi S, Simon JA, Early AP. Current overview of the management of urogenital atrophy in women with breast cancer. Breast J. 2011;17(4):403-408.

17. Menes TS, Kerlikowske K, Jaffer S, Seger D, Miglioretti D. Rates of atypical ductal hyperplasia declined with less use of postmenopausal hormone treatment: findings from the Breast Cancer Surveillance Consortium. Cancer Epidemiol Biomarkers Prev. 2009;18(11):2822-2828.

18. Santen RJ, Mansel R. Benign breast disorders. N Engl J Med. 2005;353(3):275-285.

19. Pinkerton JV, Shapiro M. The North American Menopause Society. Overview of available treatment options for VMS and VVA. Medscape Education Web site. http://www.medscape.org/viewarticle/763413. Published May 24 2012. Accessed February 23, 2013.

Issue
OBG Management - 25(5)
Issue
OBG Management - 25(5)
Page Number
36-41
Page Number
36-41
Publications
Publications
Topics
Article Type
Display Headline
Your menopausal patient’s breast biopsy reveals atypical hyperplasia
Display Headline
Your menopausal patient’s breast biopsy reveals atypical hyperplasia
Legacy Keywords
JoAnn V. Pinkerton MD;Andrew M. Kaunitz MD;James A. Simon MD;cases in menopause;atypical hyperplasia;menopausal symptoms;hot flashes;mammogram;ADH;stereotactic biopsy;vaginal dryness;dyspareunia;vaginal lubricants;hormone therapy;lifestyle changes;phytoestrogens;nonhormonal therapy;SNRIs;SSRIs;black cohosh;St. John’s Wort;red clover;gabapentin;vaginal estrogen;desvenlafaxine;vaginal estradiol;Vagifem;vaginal ring;vaginal moisturizer;paroxetine;fluoxetine;sertraline;citalopram;escitalopram;
Legacy Keywords
JoAnn V. Pinkerton MD;Andrew M. Kaunitz MD;James A. Simon MD;cases in menopause;atypical hyperplasia;menopausal symptoms;hot flashes;mammogram;ADH;stereotactic biopsy;vaginal dryness;dyspareunia;vaginal lubricants;hormone therapy;lifestyle changes;phytoestrogens;nonhormonal therapy;SNRIs;SSRIs;black cohosh;St. John’s Wort;red clover;gabapentin;vaginal estrogen;desvenlafaxine;vaginal estradiol;Vagifem;vaginal ring;vaginal moisturizer;paroxetine;fluoxetine;sertraline;citalopram;escitalopram;
Sections
Article Source

PURLs Copyright

Inside the Article

Article PDF Media